[obm-l] Re: [obm-l] Re: [obm-l] Re: [obm-l] Re: [obm-l] Combinatória

2024-03-16 Por tôpico Anderson Torres
Em qua., 13 de mar. de 2024 às 13:07, Claudio Buffara
 escreveu:
>
> Mas este caso tem 7 pessoas. E o enunciado fala em 3 A e 3 C.
>
> On Wed, Mar 13, 2024 at 9:28 AM Pedro Júnior  
> wrote:
>>
>> Eu pensei sim, mas e os casos do tipo ACCACAC. Esse caso não entra na conta 
>> 6! - 2* 3!* 3!.
>>
>> Em qua., 13 de mar. de 2024 às 09:09, Claudio Buffara 
>>  escreveu:
>>>
>>> Pense no oposto: de quantas maneiras as crianças e adultos podem se sentar 
>>> separados uns dos outros.
>>>
>>> On Wed, Mar 13, 2024 at 8:39 AM Pedro Júnior  
>>> wrote:

 Olá pessoal, bom dia.
 Alguém poderia me ajudar nesse problema?

 Seis poltronas enfileiradas em um cinema e entram 3 adultos e 3 crianças. 
 De quantas maneiras podem sentar-se 2 crianças juntas e dois adultos 
 juntos?


Eu pensei numa maneira muito feia: enumeração na raça!

AAA CCC +

AACACC +
AACCAC +
AACCCA +

ACAACC +
ACACAC -
ACACCA -

CAAACC +
CAACAC -
CAACCC +

ACCAAC +
ACCACA -
ACCCAA +

CACAAC -
CACACA -
CACCAA +

CCAAAC +
CCAACA +
CCACAA +

CCC AAA +

20 possibilidades. Apenas 14 servem.
Cada uma nos permite permutar as crianças seis vezes, os adultos outras seis.
6x6x14, que dá meh.


 Desde já fico grato!

 --
 Esta mensagem foi verificada pelo sistema de antivírus e
 acredita-se estar livre de perigo.
>>>
>>>
>>> --
>>> Esta mensagem foi verificada pelo sistema de antivírus e
>>> acredita-se estar livre de perigo.
>>
>>
>>
>> --
>>
>> Pedro Jerônimo S. de O. Júnior
>>
>> Professor de Matemática
>>
>> Geo João Pessoa – PB
>>
>>
>> --
>> Esta mensagem foi verificada pelo sistema de antivírus e
>> acredita-se estar livre de perigo.
>
>
> --
> Esta mensagem foi verificada pelo sistema de antivírus e
> acredita-se estar livre de perigo.

-- 
Esta mensagem foi verificada pelo sistema de antiv�rus e
 acredita-se estar livre de perigo.


=
Instru��es para entrar na lista, sair da lista e usar a lista em
http://www.mat.puc-rio.br/~obmlistas/obm-l.html
=


[obm-l] Re: [obm-l] Re: [obm-l] Re: [obm-l] Re: [obm-l] Re: [obm-l] Questão de probabilidade

2022-06-24 Por tôpico Rogerio Ponce
Otima explicacao!
Obrigado, Ralph!

PS: e sim, a provocacao foi pra voce mesmo!
:)

[]'s
Rogerio Ponce


On Wed, Jun 22, 2022 at 1:00 PM Ralph Costa Teixeira  wrote:
>
> Ponce está provocando a gente... senti que esta flecha tinha um bocado a 
> minha direção...  :D :D :D
>
> Olha, tem duas "visões" sobre o que "probabilidade" significa.
>
> A primeira vai na linha de que só podemos falar de probabilidade sobre coisas 
> que ainda não aconteceram. Vai nessa linha: se os eventos estão no passado, 
> então já aconteceram, já estão definidos, e não faz sentido dizer que tinha x 
> de chance de ser assim ou y de ser assado. Se você já jogou a moeda justa, 
> não é mais 50/50 --  é 100% de ser cara, ou 100% de ser coroa, dependendo do 
> que ocorreu. Quem pensa assim vai dizer que dado um certo evento (sempre no 
> futuro), ele tem uma probabilidade dada; se duas pessoas diferentes derem 
> duas probabilidades diferentes para o mesmo evento, uma delas errou.
>
> Outra linha diz que podemos falar de probabilidade sempre que houver 
> incerteza; não interessa o que aconteceu ou o que vai acontecer, o que 
> interessa é o que você SABE sobre o acontecimento. Se você jogou a moeda 
> justa mas eu não sei nada mais sobre o lançamento, continua sendo 50/50 
> **PARA MIM**. Probabilidade passa a ser um conceito sobre INFORMAÇÃO, não 
> sobre os fatos em si (a probabilidade não está na moeda, está no que você 
> sabe sobre a moeda). Quem pensa assim vai dizer que a probabilidade do evento 
> depende não apenas do evento em si, mas da informação que se tem em mãos. 
> Quem pensa assim admite que duas pessoas diferentes podem dar probabilidades 
> diferentes ao mesmo evento SE SOUBEREM FATOS DIFERENTES a respeito do evento, 
> ou seja, probabilidade passa a ser bastante "subjetivo".
>
> Eu talvez tenha descrito mal a primeira interpretação, pois sou ferrenho 
> defensor da segunda. Ela simplesmente engloba a primeira, porque você pode 
> ter informação parcial sobre fatos que ocorrem no futuro. E falar de 
> probabilidade para descrever incerteza presente ou passada é MUITO útil! Eu 
> quero poder expressar incerteza sobre fatos passados com frases do tipo 
> "fulano tem x% de probabilidade de ter cometido tal crime", ou "tem y% de 
> probabilidade de ter petróleo nesse poço", ou "tem z% de chance de eu ter 
> COVID"... Se você tem uma reação negativa a essas frases, lembre o que elas 
> realmente significam (na segunda interpretação): claro que ou o cara cometeu 
> o crime ou não, não faz sentido dizer que ele cometeu o crime x% das vezes em 
> que fizermos um experimento de ele cometer o crime... mas o que aquilo 
> significa é "com a informação que eu tenho, numa escala de 0 a 1, eu tenho 
> x/100 de certeza que fulano cometeu o crime". E "certeza baseada em informaç�!
 �es" é sim quantificável -- e satisfaz exatamente as leis das probabilidades 
com as quais concordamos. "Subjetivo" não significa "posso falar qualquer 
coisa", significa apenas que a conta pode variar de pessoa para pessoa... mas, 
de novo, SE ESSAS PESSOAS TIVEREM INFORMAÇÕES DIFERENTES sobre o evento.
>
> Abraço, Ralph.
>
> On Wed, Jun 22, 2022 at 12:09 PM Rogerio Ponce  wrote:
>>
>> Olá Pedro e pessoal da lista!
>>
>> Segundo a opinião do Pedro, nao faz sentido perguntar qual a probabilidade 
>> de Jose ter conseguido um 6 ao jogar o dado ontem, pois isso ja' aconteceu, 
>> e, portanto, ja' esta' definido.
>>
>> Sera' que e' isso mesmo?
>>
>> []'s
>> Rogerio Ponce
>>
>>
>> On Mon, Jun 20, 2022 at 9:45 PM Pedro José  wrote:
>>>
>>> Eu na minha humilde opinião creio que a probabilidade exista quando pode 
>>> ser uma coisa ou outra. No caso já é definido o que os animais são. Então 
>>> já está tudo errado. A questão seria viável se dessem esses limitantes para 
>>> uma criança que pintaria os desenhos dos animais. Aí sim há probabilidade.
>>>
>>> Em sáb., 18 de jun. de 2022 03:33, Rogerio Ponce  
>>> escreveu:

 Ola' Vanderlei e pessoal da lista!

 Sem perda de generalidade, podemos imaginar que vamos fazer o seguinte:

 - uma pintura preta em um dos caes, escolhido aleatoriamente

 - uma pintura "malhada" em um dos animais, escolhido aleatoriamente entre 
 os 7 animais nao pintados

 - duas pintura pretas, em dois animais, escolhidos aleatoriamente entre os 
 6 animais restantes,

 - quatro pinturas brancas nos 4 animais restantes


 Analisando a afirmacao 04, por exemplo, verificamos que, no segundo passo 
 (pintura malhada) existem 4 opcoes de cachorro e 3 opcoes de gato.

 Assim, a probabilidade de haver um cachorro malhado (4/7) e' maior que a 
 probabilidade de haver um gato malhado (3/7).
 Portanto, a afirmacao 04 esta' correta.
 (e o gabarito esta' errado).

 []'s
 Rogerio Ponce



 On Wed, Mar 16, 2022 at 8:08 AM Professor Vanderlei Nemitz 
  wrote:
>
> Bom dia!
> Na questão a seguir, do vestibular da UEM, 

[obm-l] Re: [obm-l] Re: [obm-l] Re: [obm-l] Re: [obm-l] Questão de probabilidade

2022-06-22 Por tôpico Ralph Costa Teixeira
Ponce está provocando a gente... senti que esta flecha tinha um bocado a
minha direção...  :D :D :D

Olha, tem duas "visões" sobre o que "probabilidade" significa.

A primeira vai na linha de que só podemos falar de probabilidade sobre
coisas que ainda não aconteceram. Vai nessa linha: se os eventos estão no
passado, então já aconteceram, já estão definidos, e não faz sentido dizer
que tinha x de chance de ser assim ou y de ser assado. Se você já jogou a
moeda justa, não é mais 50/50 --  é 100% de ser cara, ou 100% de ser coroa,
dependendo do que ocorreu. Quem pensa assim vai dizer que dado um certo
evento (sempre no futuro), ele tem uma probabilidade dada; se duas pessoas
diferentes derem duas probabilidades diferentes para o mesmo evento, uma
delas errou.

Outra linha diz que podemos falar de probabilidade sempre que houver
incerteza; não interessa o que aconteceu ou o que vai acontecer, o que
interessa é o que você SABE sobre o acontecimento. Se você jogou a moeda
justa mas eu não sei nada mais sobre o lançamento, continua sendo 50/50
**PARA MIM**. Probabilidade passa a ser um conceito sobre INFORMAÇÃO, não
sobre os fatos em si (a probabilidade não está na moeda, está no que você
sabe sobre a moeda). Quem pensa assim vai dizer que a probabilidade do
evento depende não apenas do evento em si, mas da informação que se tem em
mãos. Quem pensa assim admite que duas pessoas diferentes podem dar
probabilidades diferentes ao mesmo evento SE SOUBEREM FATOS DIFERENTES a
respeito do evento, ou seja, probabilidade passa a ser bastante "subjetivo".

Eu talvez tenha descrito mal a primeira interpretação, pois sou ferrenho
defensor da segunda. Ela simplesmente engloba a primeira, porque você pode
ter informação parcial sobre fatos que ocorrem no futuro. E falar de
probabilidade para descrever incerteza presente ou passada é MUITO útil! Eu
quero poder expressar incerteza sobre fatos passados com frases do tipo
"fulano tem x% de probabilidade de ter cometido tal crime", ou "tem y% de
probabilidade de ter petróleo nesse poço", ou "tem z% de chance de eu ter
COVID"... Se você tem uma reação negativa a essas frases, lembre o que elas
realmente significam (na segunda interpretação): claro que ou o cara
cometeu o crime ou não, não faz sentido dizer que ele cometeu o crime x%
das vezes em que fizermos um experimento de ele cometer o crime... mas o
que aquilo significa é "com a informação que eu tenho, numa escala de 0 a
1, eu tenho x/100 de certeza que fulano cometeu o crime". E "certeza
baseada em informações" é sim quantificável -- e satisfaz exatamente as
leis das probabilidades com as quais concordamos. "Subjetivo" não significa
"posso falar qualquer coisa", significa apenas que a conta pode variar de
pessoa para pessoa... mas, de novo, SE ESSAS PESSOAS TIVEREM INFORMAÇÕES
DIFERENTES sobre o evento.

Abraço, Ralph.

On Wed, Jun 22, 2022 at 12:09 PM Rogerio Ponce  wrote:

> Olá Pedro e pessoal da lista!
>
> Segundo a opinião do Pedro, nao faz sentido perguntar qual a probabilidade
> de Jose ter conseguido um 6 ao jogar o dado ontem, pois isso ja' aconteceu,
> e, portanto, ja' esta' definido.
>
> Sera' que e' isso mesmo?
>
> []'s
> Rogerio Ponce
>
>
> On Mon, Jun 20, 2022 at 9:45 PM Pedro José  wrote:
>
>> Eu na minha humilde opinião creio que a probabilidade exista quando pode
>> ser uma coisa ou outra. No caso já é definido o que os animais são. Então
>> já está tudo errado. A questão seria viável se dessem esses limitantes para
>> uma criança que pintaria os desenhos dos animais. Aí sim há probabilidade.
>>
>> Em sáb., 18 de jun. de 2022 03:33, Rogerio Ponce da Silva <
>> abrlw...@gmail.com> escreveu:
>>
>>> Ola' Vanderlei e pessoal da lista!
>>>
>>> Sem perda de generalidade, podemos imaginar que vamos fazer o seguinte:
>>>
>>> - uma pintura preta em um dos caes, escolhido aleatoriamente
>>>
>>> - uma pintura "malhada" em um dos animais, escolhido aleatoriamente
>>> entre os 7 animais nao pintados
>>>
>>> - duas pintura pretas, em dois animais, escolhidos aleatoriamente entre
>>> os 6 animais restantes,
>>>
>>> - quatro pinturas brancas nos 4 animais restantes
>>>
>>>
>>> Analisando a afirmacao 04, por exemplo, verificamos que, no segundo
>>> passo (pintura malhada) existem 4 opcoes de cachorro e 3 opcoes de gato.
>>>
>>> Assim, a probabilidade de haver um cachorro malhado (4/7) e' maior que a
>>> probabilidade de haver um gato malhado (3/7).
>>> Portanto, a afirmacao 04 esta' correta.
>>> (e o gabarito esta' errado).
>>>
>>> []'s
>>> Rogerio Ponce
>>>
>>>
>>>
>>> On Wed, Mar 16, 2022 at 8:08 AM Professor Vanderlei Nemitz <
>>> vanderma...@gmail.com> wrote:
>>>
 Bom dia!
 Na questão a seguir, do vestibular da UEM, penso que o espaço amostral
 tem 105 elementos, pois um cachorro é preto (desconsideramos esse). Porém,
 com esse pensamento, não consigo obter o gabarito, que diz que 02 e 16 são
 corretas.
 Alguém poderia ajudar?
 Muito obrigado!

 *Em um pet shop há 3 gatos e 5 cães. 

[obm-l] Re: [obm-l] Re: [obm-l] Re: [obm-l] Re: matemática discreta

2021-09-20 Por tôpico Anderson Torres
Não consegui entender esse texto.

Em seg., 20 de set. de 2021 às 22:37, Israel Meireles Chrisostomo
 escreveu:
>
> Obrigado
>
> Em seg, 20 de set de 2021 22:00, Israel Meireles Chrisostomo 
>  escreveu:
>>
>> Tome n maior que n
>>
>> Em seg, 20 de set de 2021 20:49, Marcelo Salhab Brogliato 
>>  escreveu:
>>>
>>> Oi Israel,
>>>
>>> Não consegui entender a questão.
>>>
>>> Exemplo:
>>>
>>> n = 10, m = 3, Fib(10 - 3 + 1) = Fib(8) = 21
>>>
>>> (alpha**(2*n)) / (alpha**(n - m)) = alpha**(n + m) = 521.0019193787257
>>>
>>> Pela sua igualdade, alpha**(n + m) deveria ser 1/21, correto?
>>>
>>> Abraços,
>>> Marcelo
>>>
>>> Il giorno lun 20 set 2021 alle ore 15:54 Israel Meireles Chrisostomo 
>>>  ha scritto:

 já tentei de tudo, por favor me ajudem.

 Em seg., 20 de set. de 2021 às 19:39, Israel Meireles Chrisostomo 
  escreveu:
>
> Alguém poderia resolver o problema no link abaixo?
>
> https://mathoverflow.net/questions/404417/alpha2n-fracf-n-m1-alphan-m-1-how-to-prove-that-equality-is-true
>
> --
> Israel Meireles Chrisostomo



 --
 Israel Meireles Chrisostomo

 --
 Esta mensagem foi verificada pelo sistema de antivírus e
 acredita-se estar livre de perigo.
>>>
>>>
>>> --
>>> Esta mensagem foi verificada pelo sistema de antivírus e
>>> acredita-se estar livre de perigo.
>
>
> --
> Esta mensagem foi verificada pelo sistema de antivírus e
> acredita-se estar livre de perigo.

-- 
Esta mensagem foi verificada pelo sistema de antiv�rus e
 acredita-se estar livre de perigo.


=
Instru��es para entrar na lista, sair da lista e usar a lista em
http://www.mat.puc-rio.br/~obmlistas/obm-l.html
=


[obm-l] Re: [obm-l] Re: [obm-l] Re: [obm-l] Re: [obm-l] Função

2021-05-29 Por tôpico Israel Meireles Chrisostomo
obrigado


Livre
de vírus. www.avast.com
.
<#DAB4FAD8-2DD7-40BB-A1B8-4E2AA1F9FDF2>

Em qua., 19 de mai. de 2021 às 10:56, Anderson Torres <
torres.anderson...@gmail.com> escreveu:

> Em seg., 26 de abr. de 2021 às 17:18, Israel Meireles Chrisostomo
>  escreveu:
> >
> > Mas aí então a+bi e b+ai são os mesmos números
>
> Não são.
>
> 4+5i e 5+4i são diferentes, e 4+5i < 5+4i por essas regras.
>
> >
> > Em seg, 26 de abr de 2021 13:36, Anderson Torres <
> torres.anderson...@gmail.com> escreveu:
> >>
> >> Em qui., 22 de abr. de 2021 às 07:19, Israel Meireles Chrisostomo
> >>  escreveu:
> >> >
> >> > Me desculpem se eu estou falando bobagem, mas considere uma função
> com domínio complexo, então essa função não pode ser bijetora, pois toda
> função bijetora ou é crescente ou é decrescente, mas não há ordem nos
> complexos
>
> Você não entendeu nada aqui, suponho. Primeiramente, funções não são
> coisas limitadas a números.
>
> Segundamente, quando usamos esse teorema de que funções contínuas são
> monótonas, é óbvio que estamos supondo de antemão que estamos
> trabalhando com um sistema numérico que admita a ideia de ordem.
> Especialmente, a de um corpo ordenado completo.
>
> Por exemplo, não faz sentido falar de "continuidade" quando se fala de
> funções de naturais para naturais, porque números naturais não formam
> um sistema numérico contínuo.
>
> >>
> >> Não é correto dizer que não existe ordem nos complexos. É só atribuir
> >> o seguinte: o complexo A é maior que o complexo B se e somente se ou o
> >> módulo de A é maior que o de B ou os módulos são iguais mas o
> >> argumento de A é maior que o de B (tomando este módulo no intervalo de
> >> 0 a tau).
> >>
> >> >
> >> > --
> >> > Esta mensagem foi verificada pelo sistema de antivírus e
> >> > acredita-se estar livre de perigo.
> >>
> >> --
> >> Esta mensagem foi verificada pelo sistema de antivírus e
> >>  acredita-se estar livre de perigo.
> >>
> >>
> >>
> =
> >> Instru�ões para entrar na lista, sair da lista e usar a lista em
> >> http://www.mat.puc-rio.br/~obmlistas/obm-l.html
> >>
> =
> >
> >
> > --
> > Esta mensagem foi verificada pelo sistema de antivírus e
> > acredita-se estar livre de perigo.
>
> --
> Esta mensagem foi verificada pelo sistema de antivírus e
>  acredita-se estar livre de perigo.
>
>
> =
> Instru�ões para entrar na lista, sair da lista e usar a lista em
> http://www.mat.puc-rio.br/~obmlistas/obm-l.html
> =
>


-- 
Israel Meireles Chrisostomo

-- 
Esta mensagem foi verificada pelo sistema de antiv�rus e
 acredita-se estar livre de perigo.



[obm-l] Re: [obm-l] Re: [obm-l] Re: [obm-l] Re: [obm-l] Re: [obm-l] Re: [obm-l] Re: [obm-l] Re: Equações funcionais

2021-02-16 Por tôpico Anderson Torres
Em ter., 16 de fev. de 2021 às 21:26, joao pedro b menezes
 escreveu:
>
> Eu sei, temos f(-1)= 0, f(0) = 1, e f é bijetora. Após trabalhar a equação 
> que cheguei na expressão:
> f( x + f(x) ) - f( f(x)) = x.  Queria saber se essa identidade, junto com a 
> do enunciado, é suficiente para provar a linearidade de f.
>

Seriosamente, não me parece útil perder tempo provando que isso é
linear. O processo que você levaria provando que f(x)=Ax+B basicamente
se resumiria a finalizar o problema.

Outra identidade que pode ser útil para você é provar que f(f(x)) -
f(x) = f(x) -x.
Essa, junto com a identidade acima que você provou, te deixam em 70%
do problema.

> --
> Esta mensagem foi verificada pelo sistema de antivírus e
> acredita-se estar livre de perigo.

-- 
Esta mensagem foi verificada pelo sistema de antiv�rus e
 acredita-se estar livre de perigo.


=
Instru��es para entrar na lista, sair da lista e usar a lista em
http://www.mat.puc-rio.br/~obmlistas/obm-l.html
=


[obm-l] Re: [obm-l] Re: [obm-l] Re: [obm-l] Re: [obm-l] Re: [obm-l] Re: [obm-l] Re: Equações funcionais

2021-02-16 Por tôpico joao pedro b menezes
Eu sei, temos f(-1)= 0, f(0) = 1, e f é bijetora. Após trabalhar a equação
que cheguei na expressão:
f( x + f(x) ) - f( f(x)) = x.  Queria saber se essa identidade, junto com a
do enunciado, é suficiente para provar a linearidade de f.

-- 
Esta mensagem foi verificada pelo sistema de antiv�rus e
 acredita-se estar livre de perigo.



[obm-l] Re: [obm-l] Re: [obm-l] Re: [obm-l] Re: [obm-l] Re: [obm-l] Re: Equações funcionais

2021-02-16 Por tôpico Anderson Torres
Em ter., 16 de fev. de 2021 às 20:43, joao pedro b menezes
 escreveu:
>
> Foi da OBM 2006, nível 3,  3° fase:
> “Determine todas as funções f: R -> R tais que
> f( xf(y) + f(x) ) = 2f(x) + xy

Isso dá bem mais informação!

Por exemplo essa função é sobrejetora. Afinal, qualquer número pode
ser escrito na forma 2f(x)+xy - faça por exemplo x=1 e y=z-2f(1).

Daí a ideia é resolver as equacoes f(A)=0 e f(B)=1.

> para todos x,y reais”
>
> --
> Esta mensagem foi verificada pelo sistema de antivírus e
> acredita-se estar livre de perigo.

-- 
Esta mensagem foi verificada pelo sistema de antiv�rus e
 acredita-se estar livre de perigo.


=
Instru��es para entrar na lista, sair da lista e usar a lista em
http://www.mat.puc-rio.br/~obmlistas/obm-l.html
=


[obm-l] Re: [obm-l] Re: [obm-l] Re: [obm-l] Re: [obm-l] Sequência Injetiva

2021-02-16 Por tôpico Anderson Torres
Nada como uma bijeção N -> Q para encerrar o dia!

Se pensar nas operacoes INC e REV, podemos usar um algoritmo assim:

- Se o número é maior que 1, usa DEC (inversa de INC)
- Se o número é menor que 1, usa INV
- Se o número é 1, pare

Como demonstrar que este procedimento sempre encerrará em 1, não
importando que número racional começou? Acho que no fundo isso é só
uma maneira de encodar fracoes continuas mesmo.

Em ter., 16 de fev. de 2021 às 20:35, Matheus Secco
 escreveu:
>
> Esse problema caiu na Olimpíada Iberoamericana de 2009 que eu participei. Foi 
> o problema 5 da prova e lá pedia para provar injetividade e sobrejetividade.
>
> Em qua, 17 de fev de 2021 00:16, Anderson Torres 
>  escreveu:
>>
>> Em dom., 14 de fev. de 2021 às 17:20, Claudio Buffara
>>  escreveu:
>> >
>> > Será que essa sequência é sobrejetiva (sobre os racionais positivos)?
>> > Porque como a(2^n) = n+1, ela certamente atinge todos os naturais, de modo 
>> > que é ilimitada, superiormente e inferiormente (já que a(2^n + 1) = 
>> > 1/(n+1) ).
>> > Mesmo que não seja, seria interessante descobrir que racionais positivos 
>> > ela não atinge.
>> > É suficiente provar que todos os racionais entre 0 e 1 são atingidos (no 
>> > caso, pelos termos de ordem ímpar), mas não sei se isso facilita.
>> > Vale uma exploração numérica, talvez com uma planilha.
>>
>>
>> Se eu não errei as contas, acredito que sim. Afinal basta reverter a
>> fracao continua.
>>
>> As operacoes parecem ser bem limitadas, contudo nao e necessario muito
>> mais que isso para gerar um racional qualquer:
>>
>> - Função INC: x -> x+1
>> - Função REV: x -> 1/x
>>
>> Talvez haja algum invariante que permita prever que cada operacao esta
>> fadada a cair em 1
>>
>> >
>> >
>> > Abs,
>> > Claudio.
>> >
>> > Enviado do meu iPhone
>> >
>> > Em 14 de fev. de 2021, à(s) 13:57, Anderson Torres 
>> >  escreveu:
>> >
>> > 
>> >
>> >
>> > Em sáb., 13 de fev. de 2021 às 17:56, Jeferson Almir 
>> >  escreveu:
>> >>
>> >> Amigos, peço ajuda em provar a injetividade dessa sequência que seria 
>> >> uma saída para provar a unica ocorrência do racional que aparece nela. 
>> >> Estou andando em círculos tentando montar uma possível indução.
>> >>
>> >>
>> >> Dado a sequência a_1 = 1 e a_2n = a_n  + 1 e a_2n+1 = 1/a_2n.
>> >>
>> >> Prove que para todo racional positivo que ocorre na sequência, ocorre 
>> >> uma única vez.
>> >>
>> >>
>> >
>> > Acho que e uma boa usar fracao continua aqui.
>> >
>> > Se a_n = [c0; c1, c2, ..., ck], temos entao a_1 = [1] e
>> >
>> > a_2n =Â [(1+c0); c1, c2, ..., ck] (chamemos isso de operacao E)
>> > a_2n+1 = [0; (1+c0), c1, c2, ..., ck] (chamemos isso de operacao O)
>> >
>> >
>> > A partir disso, acredito que a bijecao fica quase obvia, bastando 
>> > formalizar algumas inducoes marotas.
>> >
>> > Primeiramente, nenhuma representacao da forma [...,N,1] vai surgir dai a 
>> > partir de a_2. Isso pode ser demonstrado por inducao mesmo: ck=1 somente 
>> > no caso [1], e depois dele a funcao a_n so modifica o comeco da cadeia, 
>> > nunca o final dela.
>> >
>> > Assim sendo, temos certeza que nao tem como um racional aparecer uma vez 
>> > na forma canonica e outra na forma alternativa. E, por conseguinte, se 
>> > duas fracoes tem comprimentos diferentes, elas devem ser diferentes. E 
>> > fracoes com comprimentos iguais diferem se e somente se pelo menos um dos 
>> > componentes diferir.
>> >
>> > Agora, a funcao recursiva age de duas formas. Uma delas altera o 
>> > comprimento em 1, e a outra mantém. A que altera, só altera 
>> > acrescentando o 0 na cabeceira. A que não altera, incrementa a cabeceira.
>> >
>> > Desta forma, é possível gerar de maneira unica qualquer numero racional 
>> > comecando do 1.
>> >
>> > - Qualquer fracao de comprimento 1 pode ser gerada simplesmente aplicando 
>> > a operacao E tantas vezes quantas forem necessarias. E tambem nao e 
>> > possivel fazer isso de outra maneira, pois a operacao O aumentara o 
>> > comprimento de maneira irreversivel.
>> >
>> > - Dada uma fracao com comprimento K, temos duas sub inducoes para fazer:
>> >
>> > + A fracao tem comprimento K e comeca com 0.
>> >
>> > Â  Entao ela foi gerada por uma operacao O. O elemento que a gerou tinha 
>> > menos componentes, os quais satisfazem a hipotese de inducao.
>> >
>> > + A fracao tem comprimento K e comeca com algo maior que 0.
>> >
>> > Entao ela foi gerada por uma operacao E. A fracao da qual ela foi gerada 
>> > difere unicamente no primeiro elemento, o qual antes era menor. Assim 
>> > sendo, e possivel reduzir isso ate chegar no caso anterior.
>> >
>> > E isso demonstra recursivamente a unicidade e existencia!
>> >
>> >
>> >
>> >> --
>> >> Esta mensagem foi verificada pelo sistema de antivírus e
>> >> acredita-se estar livre de perigo.
>> >
>> >
>> > --
>> > Esta mensagem foi verificada pelo sistema de antivírus e
>> > acredita-se estar livre de perigo.
>> >
>> >
>> > --
>> > Esta mensagem foi verificada pelo sistema de 

[obm-l] Re: [obm-l] Re: [obm-l] Re: [obm-l] Re: [obm-l] Re: Equações funcionais

2021-02-16 Por tôpico joao pedro b menezes
Foi da OBM 2006, nível 3,  3° fase:
“Determine todas as funções f: R -> R tais que
f( xf(y) + f(x) ) = 2f(x) + xy
para todos x,y reais”

-- 
Esta mensagem foi verificada pelo sistema de antiv�rus e
 acredita-se estar livre de perigo.



[obm-l] Re: [obm-l] Re: [obm-l] Re: [obm-l] Re: Equações funcionais

2021-02-16 Por tôpico Anderson Torres
Eu gostaria de saber da origem desse problema...

Em dom., 14 de fev. de 2021 às 14:32, joao pedro b menezes <
joaopedrobmene...@gmail.com> escreveu:

> Obrigado pela resposta, mas ainda tenho umas dúvidas. Poderia dar um
> exemplo de tal função ou explicar como construí-la? E se f fosse somente
> injetora, mudaria alguma coisa?
>
>
> --
> Esta mensagem foi verificada pelo sistema de antivírus e
> acredita-se estar livre de perigo.

-- 
Esta mensagem foi verificada pelo sistema de antiv�rus e
 acredita-se estar livre de perigo.



[obm-l] Re: [obm-l] Re: [obm-l] Re: [obm-l] Re: [obm-l] Sequência Injetiva

2021-02-14 Por tôpico Claudio Buffara
a(1) = 1
a(2n) = a(2n-1) + 1
a(2n+1) = 1/a(2n)

Fazendo a(n) = p(n)/q(n), obtemos duas sequências: p(n) e q(n).
E elas são tais que:
p(1) = q(1) = 1
p(2n) = p(2n-1) + q(2n-1)
q(2n) = q(2n-1)
p(2n+1) = q(2n)
q(2n+1) = p(2n)

Como as sequências começam com 1 e 1, que são primos entre si, e como
mdc(p,q) = mdc(q,p) = mdc(p+q,q), p(n) e q(n) sempre serão primos entre si.

Usando a definição de p e q:
p(2n+1) = q(2n) = q(2n-1) = p(2n-2) = p(2n-3) + q(2n-3) = p(2n-3) + q(2n-2)
= p(2n-3) + p(2n-1)
e
p(2n+2) = p(2n+1) + q(2n+1) = q(2n) + p(2n) = q(2n-1) + p(2n) = p(2n-2) +
p(2n)

Ou seja, os termos p(n) de ordem ímpar e de ordem par realmente formam uma
sequência de Fibonacci.
Os de ordem ímpar começam com p(1) = p(3) = 1 e os de ordem par com p(2) =
2 e p(4) = 3.

[]s,
Claudio.

On Sun, Feb 14, 2021 at 10:03 AM Claudio Buffara 
wrote:

> Ué!  Continua sendo. Só que é outra questão...
>
>
> On Sun, Feb 14, 2021 at 3:34 AM Ralph Costa Teixeira 
> wrote:
>
>> Sim, voce tem razao -- eu achei que era a_2n = a_{2n-1} +1. Que pena, era
>> uma boa questao com Fibonacci. :)
>>
>> On Sun, Feb 14, 2021 at 12:35 AM Claudio Buffara <
>> claudio.buff...@gmail.com> wrote:
>>
>>> Oi, Ralph:
>>>
>>> Eu posso ter entendido errado a definição da sequência, mas achei termos
>>> diferentes dos seus:
>>> 1:  1
>>> 2:  2
>>> 3:  1/2
>>> 4:  3
>>> 5:  1/3
>>> 6:  3/2
>>> 7:  2/3
>>> 8:  4
>>> 9:  1/4
>>> 10:  4/3
>>> 11:  3/4
>>> 12:  5/2
>>> 13:  2/5
>>> 14:  5/3
>>> 15:  3/5
>>> 16:  5
>>> ...
>>>
>>> []s,
>>> Claudio.
>>>
>>>
>>> On Sat, Feb 13, 2021 at 7:59 PM Ralph Costa Teixeira 
>>> wrote:
>>>
 Meio enrolado, vou escrever meio vagamente.

 Eu sugiro olhar primeiro para os caras com indice impar. Sao eles:
 a1=1/1
 a3=1/2
 a5=2/3
 a7=3/5
 a8=5/8
 ...
 Ou seja, mostre que eles sao quocientes de numeros de Fibonacci
 consecutivos (os caras de indice par sao os inversos desses). Agora tem
 varias maneiras de continuar:

 -- Voce pode mostrar que os numeros de Fibonacci consecutivos sao
 primos entre si; portanto cada fracao dessas fica unicamente determinada
 por numerador e denominador, e (como os numeros de Fibonacci formam uma
 sequencia crescente) vao ser distintos entre si;
 -- Se voce nao quiser entrar no merito do Fibonacci, tente mostrar
 (pode ser por inducao) que a3 < a7 < a11 <...>>> a_(4k+1) < ... < a13 < a9 < a5 < 1  (phi ali seria (raiz(5)-1) / 2, acho).

 De qualquer forma, como a_(2n+1)<1, a1=1 e os "a_2n" sao os inversos
 dos "a_2n+1, vao ser todos diferentes.

 Abraco, Ralph.


 On Sat, Feb 13, 2021 at 5:56 PM Jeferson Almir <
 jefersonram...@gmail.com> wrote:

> Amigos, peço ajuda em provar a injetividade dessa sequência que seria
> uma saída para provar a unica ocorrência do racional que aparece nela.
> Estou andando em círculos tentando montar uma possível indução.
>
>
> Dado a sequência a_1 = 1 e a_2n = a_n  + 1 e a_2n+1 = 1/a_2n.
>
> Prove que para todo racional positivo que ocorre na sequência, ocorre
> uma única vez.
>
> --
> Esta mensagem foi verificada pelo sistema de antivírus e
> acredita-se estar livre de perigo.


 --
 Esta mensagem foi verificada pelo sistema de antivírus e
 acredita-se estar livre de perigo.
>>>
>>>
>>> --
>>> Esta mensagem foi verificada pelo sistema de antivírus e
>>> acredita-se estar livre de perigo.
>>
>>
>> --
>> Esta mensagem foi verificada pelo sistema de antivírus e
>> acredita-se estar livre de perigo.
>
>

-- 
Esta mensagem foi verificada pelo sistema de antiv�rus e
 acredita-se estar livre de perigo.



[obm-l] Re: [obm-l] Re: [obm-l] Re: [obm-l] Re: [obm-l] Sequência Injetiva

2021-02-14 Por tôpico Claudio Buffara
Ué!  Continua sendo. Só que é outra questão...


On Sun, Feb 14, 2021 at 3:34 AM Ralph Costa Teixeira 
wrote:

> Sim, voce tem razao -- eu achei que era a_2n = a_{2n-1} +1. Que pena, era
> uma boa questao com Fibonacci. :)
>
> On Sun, Feb 14, 2021 at 12:35 AM Claudio Buffara <
> claudio.buff...@gmail.com> wrote:
>
>> Oi, Ralph:
>>
>> Eu posso ter entendido errado a definição da sequência, mas achei termos
>> diferentes dos seus:
>> 1:  1
>> 2:  2
>> 3:  1/2
>> 4:  3
>> 5:  1/3
>> 6:  3/2
>> 7:  2/3
>> 8:  4
>> 9:  1/4
>> 10:  4/3
>> 11:  3/4
>> 12:  5/2
>> 13:  2/5
>> 14:  5/3
>> 15:  3/5
>> 16:  5
>> ...
>>
>> []s,
>> Claudio.
>>
>>
>> On Sat, Feb 13, 2021 at 7:59 PM Ralph Costa Teixeira 
>> wrote:
>>
>>> Meio enrolado, vou escrever meio vagamente.
>>>
>>> Eu sugiro olhar primeiro para os caras com indice impar. Sao eles:
>>> a1=1/1
>>> a3=1/2
>>> a5=2/3
>>> a7=3/5
>>> a8=5/8
>>> ...
>>> Ou seja, mostre que eles sao quocientes de numeros de Fibonacci
>>> consecutivos (os caras de indice par sao os inversos desses). Agora tem
>>> varias maneiras de continuar:
>>>
>>> -- Voce pode mostrar que os numeros de Fibonacci consecutivos sao primos
>>> entre si; portanto cada fracao dessas fica unicamente determinada por
>>> numerador e denominador, e (como os numeros de Fibonacci formam uma
>>> sequencia crescente) vao ser distintos entre si;
>>> -- Se voce nao quiser entrar no merito do Fibonacci, tente mostrar (pode
>>> ser por inducao) que a3 < a7 < a11 <...>> < ... < a13 < a9 < a5 < 1  (phi ali seria (raiz(5)-1) / 2, acho).
>>>
>>> De qualquer forma, como a_(2n+1)<1, a1=1 e os "a_2n" sao os inversos dos
>>> "a_2n+1, vao ser todos diferentes.
>>>
>>> Abraco, Ralph.
>>>
>>>
>>> On Sat, Feb 13, 2021 at 5:56 PM Jeferson Almir 
>>> wrote:
>>>
 Amigos, peço ajuda em provar a injetividade dessa sequência que seria
 uma saída para provar a unica ocorrência do racional que aparece nela.
 Estou andando em círculos tentando montar uma possível indução.


 Dado a sequência a_1 = 1 e a_2n = a_n  + 1 e a_2n+1 = 1/a_2n.

 Prove que para todo racional positivo que ocorre na sequência, ocorre
 uma única vez.

 --
 Esta mensagem foi verificada pelo sistema de antivírus e
 acredita-se estar livre de perigo.
>>>
>>>
>>> --
>>> Esta mensagem foi verificada pelo sistema de antivírus e
>>> acredita-se estar livre de perigo.
>>
>>
>> --
>> Esta mensagem foi verificada pelo sistema de antivírus e
>> acredita-se estar livre de perigo.
>
>
> --
> Esta mensagem foi verificada pelo sistema de antivírus e
> acredita-se estar livre de perigo.

-- 
Esta mensagem foi verificada pelo sistema de antiv�rus e
 acredita-se estar livre de perigo.



[obm-l] Re: [obm-l] Re: [obm-l] Re: [obm-l] Re: [obm-l] Re: [obm-l] Re: [obm-l] Relação de girard

2020-11-16 Por tôpico Claudio Buffara
Sugestão: proponha pra eles o problema de determinar se é possível atribuir
sinais "+" ou "-" a cada um dos números:
1  2  3  4  5  6  7  8  9  10
de modo que a soma algébrica (com sinal) destes números seja igual a zero.
Isso é um desafio e é razoavelmente lúdico, apesar de envolver conceitos
que uma criança de 8 anos entenderia.

On Sat, Nov 14, 2020 at 4:22 PM Israel Meireles Chrisostomo <
israelmchrisost...@gmail.com> wrote:

> Desculpe é q eu queria propor algo q fosse lúdico, mais um desafio,
> voltada para jovens adolescentes, algo descompromissado, sem muitas
> complicações com formalidades
>
> Em qui, 12 de nov de 2020 09:10, Anderson Torres <
> torres.anderson...@gmail.com> escreveu:
>
>>
>>
>> Em sáb., 7 de nov. de 2020 às 16:44, Israel Meireles Chrisostomo <
>> israelmchrisost...@gmail.com> escreveu:
>>
>>>   o objetivo dessa proposta é recriar o ambiente vivido por Euler na
>>> época.
>>>
>>
>> E naquele tempo eles não usavam indução? Formalização é algo bem recente
>> na matemática.
>>
>> Sua exigência me parece algo tão surreal quanto exigir rigor na geometria
>> do tempo de Euclides.
>>
>>
>>
>>>
>>> Em sáb., 7 de nov. de 2020 às 15:10, Israel Meireles Chrisostomo <
>>> israelmchrisost...@gmail.com> escreveu:
>>>
 Na verdade eu estava elaborando um problema que dependia disso.O
 problema é esse aqui:

 Desafio do ano: resolver o problema da Basiléia sem usar derivadas,
 integrais, série de potências, produto infinito do seno ou cosseno, ou
 mesmo indução ou números complexos.

 Em sáb., 7 de nov. de 2020 às 15:07, Israel Meireles Chrisostomo <
 israelmchrisost...@gmail.com> escreveu:

> Na verdade eu estava elaborando um problema que dependia disso.O
> problema é esse aqui:
>
> Desafio do ano: resolver o problema da Basiléia sem usar derivadas,
> integrais, série de potências, produto infinito do seno ou cosseno, ou
> mesmo indução.
>
> Em sáb., 7 de nov. de 2020 às 14:47, Israel Meireles Chrisostomo <
> israelmchrisost...@gmail.com> escreveu:
>
>> conheço uma que usa o teorema de d'lambert
>>
>> Em sáb., 7 de nov. de 2020 às 12:50, Bernardo Freitas Paulo da Costa <
>> bernardo...@gmail.com> escreveu:
>>
>>> On Thu, Nov 5, 2020 at 9:26 PM Artur Costa Steiner
>>>  wrote:
>>> >
>>> > Para facilitar, suponhamos que o polinômio de grau n P seja
>>> mônico. Sejam z_1, , z_n suas n raízes não necessariamente 
>>> distintas.
>>> Para todo complexo z, temos que
>>> >
>>> > P(z)  = ( z - z_1) (z - z_n)
>>> >
>>> > Desenvolvendo e aplicando o chamado produto de Stevin, vc tem as
>>> relações de Girard.
>>>
>>> Eu não conhecia o produto de Stevin, mas de forma geral quando você
>>> usa "..." tem, muitas vezes, um argumento por indução que está
>>> subentendido.  Pode ser que o produto de Stevin "faça a indução pra
>>> você" (calculando os termos \sum \prod z_i que vão aparecer como
>>> coeficientes dos monômios z^k), mas é "quase" como se você estivesse
>>> empurrando a indução um andar abaixo ;-)
>>>
>>>
>>> Israel: qual a demonstração por indução que você conhece?  E porque
>>> você gostaria de outra??
>>>
>>> Abraços,
>>> --
>>> Bernardo Freitas Paulo da Costa
>>>
>>>
>>> =
>>> Instru�ões para entrar na lista, sair da lista e usar a lista em
>>> http://www.mat.puc-rio.br/~obmlistas/obm-l.html
>>>
>>> =
>>>
>>
>>
>> --
>> Israel Meireles Chrisostomo
>>
>
>
> --
> Israel Meireles Chrisostomo
>


 --
 Israel Meireles Chrisostomo

>>>
>>>
>>> --
>>> Israel Meireles Chrisostomo
>>>
>>


[obm-l] Re: [obm-l] Re: [obm-l] Re: [obm-l] Re: [obm-l] Re: [obm-l] Relação de girard

2020-11-14 Por tôpico Israel Meireles Chrisostomo
Desculpe é q eu queria propor algo q fosse lúdico, mais um desafio,
voltada para jovens adolescentes, algo descompromissado, sem muitas
complicações com formalidades

Em qui, 12 de nov de 2020 09:10, Anderson Torres <
torres.anderson...@gmail.com> escreveu:

>
>
> Em sáb., 7 de nov. de 2020 às 16:44, Israel Meireles Chrisostomo <
> israelmchrisost...@gmail.com> escreveu:
>
>>   o objetivo dessa proposta é recriar o ambiente vivido por Euler na
>> época.
>>
>
> E naquele tempo eles não usavam indução? Formalização é algo bem recente
> na matemática.
>
> Sua exigência me parece algo tão surreal quanto exigir rigor na geometria
> do tempo de Euclides.
>
>
>
>>
>> Em sáb., 7 de nov. de 2020 às 15:10, Israel Meireles Chrisostomo <
>> israelmchrisost...@gmail.com> escreveu:
>>
>>> Na verdade eu estava elaborando um problema que dependia disso.O
>>> problema é esse aqui:
>>>
>>> Desafio do ano: resolver o problema da Basiléia sem usar derivadas,
>>> integrais, série de potências, produto infinito do seno ou cosseno, ou
>>> mesmo indução ou números complexos.
>>>
>>> Em sáb., 7 de nov. de 2020 às 15:07, Israel Meireles Chrisostomo <
>>> israelmchrisost...@gmail.com> escreveu:
>>>
 Na verdade eu estava elaborando um problema que dependia disso.O
 problema é esse aqui:

 Desafio do ano: resolver o problema da Basiléia sem usar derivadas,
 integrais, série de potências, produto infinito do seno ou cosseno, ou
 mesmo indução.

 Em sáb., 7 de nov. de 2020 às 14:47, Israel Meireles Chrisostomo <
 israelmchrisost...@gmail.com> escreveu:

> conheço uma que usa o teorema de d'lambert
>
> Em sáb., 7 de nov. de 2020 às 12:50, Bernardo Freitas Paulo da Costa <
> bernardo...@gmail.com> escreveu:
>
>> On Thu, Nov 5, 2020 at 9:26 PM Artur Costa Steiner
>>  wrote:
>> >
>> > Para facilitar, suponhamos que o polinômio de grau n P seja mônico.
>> Sejam z_1, , z_n suas n raízes não necessariamente distintas. Para 
>> todo
>> complexo z, temos que
>> >
>> > P(z)  = ( z - z_1) (z - z_n)
>> >
>> > Desenvolvendo e aplicando o chamado produto de Stevin, vc tem as
>> relações de Girard.
>>
>> Eu não conhecia o produto de Stevin, mas de forma geral quando você
>> usa "..." tem, muitas vezes, um argumento por indução que está
>> subentendido.  Pode ser que o produto de Stevin "faça a indução pra
>> você" (calculando os termos \sum \prod z_i que vão aparecer como
>> coeficientes dos monômios z^k), mas é "quase" como se você estivesse
>> empurrando a indução um andar abaixo ;-)
>>
>>
>> Israel: qual a demonstração por indução que você conhece?  E porque
>> você gostaria de outra??
>>
>> Abraços,
>> --
>> Bernardo Freitas Paulo da Costa
>>
>>
>> =
>> Instru�ões para entrar na lista, sair da lista e usar a lista em
>> http://www.mat.puc-rio.br/~obmlistas/obm-l.html
>>
>> =
>>
>
>
> --
> Israel Meireles Chrisostomo
>


 --
 Israel Meireles Chrisostomo

>>>
>>>
>>> --
>>> Israel Meireles Chrisostomo
>>>
>>
>>
>> --
>> Israel Meireles Chrisostomo
>>
>


[obm-l] Re: [obm-l] Re: [obm-l] Re: [obm-l] Re: [obm-l] Relação de girard

2020-11-12 Por tôpico Anderson Torres
Em sáb., 7 de nov. de 2020 às 16:44, Israel Meireles Chrisostomo <
israelmchrisost...@gmail.com> escreveu:

>   o objetivo dessa proposta é recriar o ambiente vivido por Euler na
> época.
>

E naquele tempo eles não usavam indução? Formalização é algo bem recente na
matemática.

Sua exigência me parece algo tão surreal quanto exigir rigor na geometria
do tempo de Euclides.



>
> Em sáb., 7 de nov. de 2020 às 15:10, Israel Meireles Chrisostomo <
> israelmchrisost...@gmail.com> escreveu:
>
>> Na verdade eu estava elaborando um problema que dependia disso.O problema
>> é esse aqui:
>>
>> Desafio do ano: resolver o problema da Basiléia sem usar derivadas,
>> integrais, série de potências, produto infinito do seno ou cosseno, ou
>> mesmo indução ou números complexos.
>>
>> Em sáb., 7 de nov. de 2020 às 15:07, Israel Meireles Chrisostomo <
>> israelmchrisost...@gmail.com> escreveu:
>>
>>> Na verdade eu estava elaborando um problema que dependia disso.O
>>> problema é esse aqui:
>>>
>>> Desafio do ano: resolver o problema da Basiléia sem usar derivadas,
>>> integrais, série de potências, produto infinito do seno ou cosseno, ou
>>> mesmo indução.
>>>
>>> Em sáb., 7 de nov. de 2020 às 14:47, Israel Meireles Chrisostomo <
>>> israelmchrisost...@gmail.com> escreveu:
>>>
 conheço uma que usa o teorema de d'lambert

 Em sáb., 7 de nov. de 2020 às 12:50, Bernardo Freitas Paulo da Costa <
 bernardo...@gmail.com> escreveu:

> On Thu, Nov 5, 2020 at 9:26 PM Artur Costa Steiner
>  wrote:
> >
> > Para facilitar, suponhamos que o polinômio de grau n P seja mônico.
> Sejam z_1, , z_n suas n raízes não necessariamente distintas. Para 
> todo
> complexo z, temos que
> >
> > P(z)  = ( z - z_1) (z - z_n)
> >
> > Desenvolvendo e aplicando o chamado produto de Stevin, vc tem as
> relações de Girard.
>
> Eu não conhecia o produto de Stevin, mas de forma geral quando você
> usa "..." tem, muitas vezes, um argumento por indução que está
> subentendido.  Pode ser que o produto de Stevin "faça a indução pra
> você" (calculando os termos \sum \prod z_i que vão aparecer como
> coeficientes dos monômios z^k), mas é "quase" como se você estivesse
> empurrando a indução um andar abaixo ;-)
>
>
> Israel: qual a demonstração por indução que você conhece?  E porque
> você gostaria de outra??
>
> Abraços,
> --
> Bernardo Freitas Paulo da Costa
>
>
> =
> Instru�ões para entrar na lista, sair da lista e usar a lista em
> http://www.mat.puc-rio.br/~obmlistas/obm-l.html
>
> =
>


 --
 Israel Meireles Chrisostomo

>>>
>>>
>>> --
>>> Israel Meireles Chrisostomo
>>>
>>
>>
>> --
>> Israel Meireles Chrisostomo
>>
>
>
> --
> Israel Meireles Chrisostomo
>


[obm-l] Re: [obm-l] Re: [obm-l] Re: [obm-l] Re: [obm-l] Ajuda em teoria dos números

2020-10-22 Por tôpico Pedro José
Boa tarde!
Na verdade: 2^a=64; a= 6 e y=12.

Em qui., 22 de out. de 2020 às 11:17, Pedro José 
escreveu:

> Bom dia!
> Recebi esse problema hoje: 615 + x^2 = 2^y., para x,y inteiros Não saberia
> fazer, como não soube resolver esse, acima. Mas devido a solução do colega
> Esdras, pensei:"já vi algo parecido".
> Basta restringir y aos pares.
> Se y é ímpar x^2=2 mod3, absurdo então y é par. Logo y=2a, com a inteiro.
> (2^a + x) (2^a-x)= 615= 1*615=3*205=5*123=15*41 e como a soma dos fatores
> necessita ser uma potência de 2, só serve para 123 e 5.
> Logo 2^y=64 e y=6 e x= 59 ou x=-59.
> Uma resolução levou a outra, não pelo talento nato, mas por aprendizado, o
> que é válido.
> Teve uma feita que estava tentando provar que o triângulo órtico, era o
> triângulo de menor perímetro que poderia ser inscrito em um triângulo
> acutângulo. Tentei por geometria analítica e só levando tinta. Tinha
> desistido. Quando me deparei com um problema que não consegui resolver, mas
> que tinha um caminho para a solução. Quando vi o rebatimento feito, pensei
> está resolvido. O curioso, é que, quando desisti, pesquisei na internet e
> não achei nada. Depois que consegui resolver, achei duas soluções, e
> infelizmente e como esperado, a minha não era novidade, era clássica.
> Obrigado, Esdras! Sem a sua solução, certamente, não teria resolvido essa
> última questão.
>
> Cordialmente,
> PJMS
>
> Em sex., 24 de jul. de 2020 às 12:19, Prof. Douglas Oliveira <
> profdouglaso.del...@gmail.com> escreveu:
>
>> Obrigado Claudio e Esdras, fatoração show
>>
>>
>> Em sex., 24 de jul. de 2020 às 11:12, Esdras Muniz <
>> esdrasmunizm...@gmail.com> escreveu:
>>
>>> Se for solução inteira positiva, acho que só tem 3 e 4. Vc supõe spdg x
>>> maior ou igual a y, vê que y=1 não tem solução e x=y tb não. Daí, x>y>1.
>>> Fatorando a expressão, fica: (xy-8-(x-y))(xy-8+(x-y))=15. Como
>>> (xy-8-(x-y))>(xy-8+(x-y))>-2. Temos que ou (xy-8-(x-y))=1 e (xy-8+(x-y))=15,
>>> o que não tem soluções inteiras positivas, ou (xy-8-(x-y))=3 e 
>>> (xy-8+(x-y))=5,
>>> cujas únicas soluções inteiras são x=4 e y=3.
>>>
>>> Em sex, 24 de jul de 2020 10:36, Claudio Buffara <
>>> claudio.buff...@gmail.com> escreveu:
>>>
 Pelo que entendi, a solução é a porção dessa curva algébrica situada no
 1o quadrante.
 Dá pra fazer isso no Wolfram Alpha, com o comando plot (x*y-7)^2 - x^2
 - y^2 = 0.

 []s,
 Claudio.

 On Fri, Jul 24, 2020 at 9:58 AM Prof. Douglas Oliveira <
 profdouglaso.del...@gmail.com> wrote:

> Preciso de ajuda para encontrar todas as soluções não negativas da
> equação
> (xy-7)^2=x^2+y^2.
>
> Desde já agradeço a ajuda
> Douglas Oliveira
>
> --
> Esta mensagem foi verificada pelo sistema de antivírus e
> acredita-se estar livre de perigo.


 --
 Esta mensagem foi verificada pelo sistema de antivírus e
 acredita-se estar livre de perigo.
>>>
>>>
>>> --
>>> Esta mensagem foi verificada pelo sistema de antivírus e
>>> acredita-se estar livre de perigo.
>>
>>
>> --
>> Esta mensagem foi verificada pelo sistema de antivírus e
>> acredita-se estar livre de perigo.
>
>


[obm-l] Re: [obm-l] Re: [obm-l] Re: [obm-l] Re: [obm-l] Ajuda em teoria dos números

2020-10-22 Por tôpico Pedro José
Bom dia!
Recebi esse problema hoje: 615 + x^2 = 2^y., para x,y inteiros Não saberia
fazer, como não soube resolver esse, acima. Mas devido a solução do colega
Esdras, pensei:"já vi algo parecido".
Basta restringir y aos pares.
Se y é ímpar x^2=2 mod3, absurdo então y é par. Logo y=2a, com a inteiro.
(2^a + x) (2^a-x)= 615= 1*615=3*205=5*123=15*41 e como a soma dos fatores
necessita ser uma potência de 2, só serve para 123 e 5.
Logo 2^y=64 e y=6 e x= 59 ou x=-59.
Uma resolução levou a outra, não pelo talento nato, mas por aprendizado, o
que é válido.
Teve uma feita que estava tentando provar que o triângulo órtico, era o
triângulo de menor perímetro que poderia ser inscrito em um triângulo
acutângulo. Tentei por geometria analítica e só levando tinta. Tinha
desistido. Quando me deparei com um problema que não consegui resolver, mas
que tinha um caminho para a solução. Quando vi o rebatimento feito, pensei
está resolvido. O curioso, é que, quando desisti, pesquisei na internet e
não achei nada. Depois que consegui resolver, achei duas soluções, e
infelizmente e como esperado, a minha não era novidade, era clássica.
Obrigado, Esdras! Sem a sua solução, certamente, não teria resolvido essa
última questão.

Cordialmente,
PJMS

Em sex., 24 de jul. de 2020 às 12:19, Prof. Douglas Oliveira <
profdouglaso.del...@gmail.com> escreveu:

> Obrigado Claudio e Esdras, fatoração show
>
>
> Em sex., 24 de jul. de 2020 às 11:12, Esdras Muniz <
> esdrasmunizm...@gmail.com> escreveu:
>
>> Se for solução inteira positiva, acho que só tem 3 e 4. Vc supõe spdg x
>> maior ou igual a y, vê que y=1 não tem solução e x=y tb não. Daí, x>y>1.
>> Fatorando a expressão, fica: (xy-8-(x-y))(xy-8+(x-y))=15. Como
>> (xy-8-(x-y))>(xy-8+(x-y))>-2. Temos que ou (xy-8-(x-y))=1 e (xy-8+(x-y))=15,
>> o que não tem soluções inteiras positivas, ou (xy-8-(x-y))=3 e 
>> (xy-8+(x-y))=5,
>> cujas únicas soluções inteiras são x=4 e y=3.
>>
>> Em sex, 24 de jul de 2020 10:36, Claudio Buffara <
>> claudio.buff...@gmail.com> escreveu:
>>
>>> Pelo que entendi, a solução é a porção dessa curva algébrica situada no
>>> 1o quadrante.
>>> Dá pra fazer isso no Wolfram Alpha, com o comando plot (x*y-7)^2 - x^2 -
>>> y^2 = 0.
>>>
>>> []s,
>>> Claudio.
>>>
>>> On Fri, Jul 24, 2020 at 9:58 AM Prof. Douglas Oliveira <
>>> profdouglaso.del...@gmail.com> wrote:
>>>
 Preciso de ajuda para encontrar todas as soluções não negativas da
 equação
 (xy-7)^2=x^2+y^2.

 Desde já agradeço a ajuda
 Douglas Oliveira

 --
 Esta mensagem foi verificada pelo sistema de antivírus e
 acredita-se estar livre de perigo.
>>>
>>>
>>> --
>>> Esta mensagem foi verificada pelo sistema de antivírus e
>>> acredita-se estar livre de perigo.
>>
>>
>> --
>> Esta mensagem foi verificada pelo sistema de antivírus e
>> acredita-se estar livre de perigo.
>
>
> --
> Esta mensagem foi verificada pelo sistema de antivírus e
> acredita-se estar livre de perigo.


[obm-l] Re: [obm-l] Re: [obm-l] Re: [obm-l] Re: [obm-l] Re: [obm-l] Re: [obm-l] Geometria plana com desigualdade de médias?

2020-08-27 Por tôpico Anderson Torres
Em qua., 26 de ago. de 2020 às 18:29, Pedro José  escreveu:
>
> Boa noite!
> Anderson,
> achei legal a sua visão. Mas não consegui evoluir com nada.
> Todavia, fiquei com uma dúvida. Como x+y é um dos ângulos do triângulo temos 
> a restrição 0 E entendo que tanto para cotg(x) + cot(y) , como para tg(x) + tg(y) ocorrerá 
> um mínimo em x=y=K/2, onde x+y=k,k sendo um constante.
> Não acompanhei a sua dedução d quando um é mínimo o outro é máximo.

Eu não fui muito claro.

Você converteu o problema em "calcule o valor mínimo de cot(x)+cot(y)
com x+y fixo". Isso é essencialmente o mesmo que resolver o problema
"calcule o valor mínimo de tan(a)+tan(b) com a+b fixo" - pois sabendo
resolver um é só usar a mesma solução para x=90-a, y=90-b.

>
> Saudações,
> PJMS
>
> Em qui., 20 de ago. de 2020 às 22:40, Anderson Torres 
>  escreveu:
>>
>> Em qui., 20 de ago. de 2020 às 22:03, Anderson Torres
>>  escreveu:
>> >
>> > Em ter., 18 de ago. de 2020 às 19:51, Pedro José  
>> > escreveu:
>> > >
>> > > Boa noite!
>> > > Cláudio,
>> > > não consegui nada geométrico.
>> > > O máximo que atingi foi:
>> > > a/ha + b/hb + c/hc= [cotg(A1) +cotg (A2)]  + [cotg(B1) +cotg (B2)] + 
>> > > co[tg(C1) +cotg (C2)] com A1 + A2 = A; B1 + B2 + B e C1 + C2 = C.
>> > > Para ser mínimo cada termo entre colchetes deve ser mínimo, o que ocorre 
>> > > quando A1 = A2; B1 = B2 e C1 = C2. Logo P seria o encontro das 
>> > > bissetrizes e logo I.
>> > > Onde: A1= PAB e A2=PAC; B1=PBA e B2=PBC; C1=PCA e C2=PCB.
>> >
>> > Acho que daqui poderia sair uma interpretação mais escamoteada.
>> > Afinal, trigonometria é uma espécie de "ponto de contato" entre a
>> > geometria analítica e a sintética, entre a nuvem de desenhos e a de
>> > números.
>> >
>> > Acredito que a solução aqui seria arranjar uma interpretação
>> > geométrica desses colchetes de co-tangentes. Acredito que possamos
>> > apelar para Ptolomeu em algum momento ou para um macete de
>> > semelhanças, pois as projeções de um ponto sobre duas retas criam um
>> > quadrilátero cíclico.
>>
>> Acrescentando mais coisas: se queremos minimizar cot(x) +cot(y) com
>> x+y fixo, isto é equivalente a minimizar tan(90-x)+tan(90-y) com
>> 90-x+90-y fixo. Ou como maximizar tan(x) + tan(y) com x+y fixo.
>>
>> Geometricamente, tangente é cateto oposto dividido por cateto
>> adjacente. Logo uma soma de tangentes com catetos adjacentes iguais
>> equivale a uma soma de catetos opostos! Assim sendo, nosso problema
>> pode ser pensado da seguinte forma:
>>
>> Dados um ponto A e uma reta d fixos, temos que construir duas retas x
>> e y, com ângulo 'alfa' entre elas, ambas passando por A e tais que a
>> distância entre os pontos X e Y, que elas geram ao intersectar d, seja
>> mínima.
>>
>> Daí fica fácil argumentar que a altura por A também tem que ser a
>> bissetriz por A.
>>
>> No fundo do fundo é uma forma de geometrizar a solução trigonométrica.
>> A trigonometria se torna apenas um atalho.
>>
>> Vou formalizar isso mais tarde, com desenhos e tudo.
>>
>>
>>
>> >
>> > Isso até me lembra o famoso artigo do Shine sobre geometria cearense
>> > VS geometria paulista:
>> > https://cyshine.webs.com/geometria-2005.pdf
>> >
>> >
>> > >
>> > > Saudações,
>> > > PJMS
>> > >
>> > > Em ter., 18 de ago. de 2020 às 11:34, Claudio Buffara 
>> > >  escreveu:
>> > >>
>> > >> Será que tem uma demonstração mais geométrica e menos algébrica disso? 
>> > >> E que torne o resultado mais intuitivo?
>> > >> É razoável que o ponto P não esteja muito próximo de qualquer dos 
>> > >> lados, pois neste caso, se P se aproximasse do lado a, por exemplo, 
>> > >> a/h_a cresceria e a expressão se afastaria do valor mínimo.
>> > >> Mas, com lados não necessariamente congruentes, não é óbvio, a priori, 
>> > >> que P deva ser equidistante dos três.
>> > >> De fato, seria razoável esperar que P estivesse mais próximo do maior 
>> > >> lado e conjecturar, por exemplo, que o P que minimiza a expressão é tal 
>> > >> que a/h_a = b/h_b = c/h_c.
>> > >> O fato de P ser o incentro não me parece a conjectura mais evidente 
>> > >> neste caso.
>> > >>
>> > >>
>> > >> On Sun, Aug 16, 2020 at 10:11 AM Matheus Secco  
>> > >> wrote:
>> > >>>
>> > >>> Olá, Vanderlei.
>> > >>> Por Cauchy-Schwarz, temos
>> > >>>
>> > >>> (a/ha + b/hb + c/hc) * (a*ha + b*hb + c*hc) >= (a+b+c)^2.  (#)
>> > >>>
>> > >>> Como (a*ha + b*hb + c*hc) = 2S, onde S é a área de ABC, segue que a 
>> > >>> expressão a/ha + b/hb + c/hc é pelo menos 2p^2/S, onde p é o 
>> > >>> semi-perimetro.
>> > >>>
>> > >>> Por outro lado, a igualdade em (#) ocorre se, e somente se, ha = hb = 
>> > >>> hc, ou seja, quando P é o incentro do triângulo
>> > >>>
>> > >>> Abraços,
>> > >>> Matheus
>> > >>>
>> > >>> Em dom, 16 de ago de 2020 08:59, Professor Vanderlei Nemitz 
>> > >>>  escreveu:
>> > 
>> >  Bom dia!
>> > 
>> >  Tentei utilizar alguma desigualdade de médias aqui, mas não tive 
>> >  êxito. Alguém ajuda?
>> >  Muito agradecido!
>> > 
>> >  Seja P um ponto no 

[obm-l] Re: [obm-l] Re: [obm-l] Re: [obm-l] Re: [obm-l] Re: [obm-l] Geometria plana com desigualdade de médias?

2020-08-26 Por tôpico Pedro José
Boa noite!
Anderson,
achei legal a sua visão. Mas não consegui evoluir com nada.
Todavia, fiquei com uma dúvida. Como x+y é um dos ângulos do triângulo
temos a restrição 0 escreveu:

> Em qui., 20 de ago. de 2020 às 22:03, Anderson Torres
>  escreveu:
> >
> > Em ter., 18 de ago. de 2020 às 19:51, Pedro José 
> escreveu:
> > >
> > > Boa noite!
> > > Cláudio,
> > > não consegui nada geométrico.
> > > O máximo que atingi foi:
> > > a/ha + b/hb + c/hc= [cotg(A1) +cotg (A2)]  + [cotg(B1) +cotg (B2)] +
> co[tg(C1) +cotg (C2)] com A1 + A2 = A; B1 + B2 + B e C1 + C2 = C.
> > > Para ser mínimo cada termo entre colchetes deve ser mínimo, o que
> ocorre quando A1 = A2; B1 = B2 e C1 = C2. Logo P seria o encontro das
> bissetrizes e logo I.
> > > Onde: A1= PAB e A2=PAC; B1=PBA e B2=PBC; C1=PCA e C2=PCB.
> >
> > Acho que daqui poderia sair uma interpretação mais escamoteada.
> > Afinal, trigonometria é uma espécie de "ponto de contato" entre a
> > geometria analítica e a sintética, entre a nuvem de desenhos e a de
> > números.
> >
> > Acredito que a solução aqui seria arranjar uma interpretação
> > geométrica desses colchetes de co-tangentes. Acredito que possamos
> > apelar para Ptolomeu em algum momento ou para um macete de
> > semelhanças, pois as projeções de um ponto sobre duas retas criam um
> > quadrilátero cíclico.
>
> Acrescentando mais coisas: se queremos minimizar cot(x) +cot(y) com
> x+y fixo, isto é equivalente a minimizar tan(90-x)+tan(90-y) com
> 90-x+90-y fixo. Ou como maximizar tan(x) + tan(y) com x+y fixo.
>
> Geometricamente, tangente é cateto oposto dividido por cateto
> adjacente. Logo uma soma de tangentes com catetos adjacentes iguais
> equivale a uma soma de catetos opostos! Assim sendo, nosso problema
> pode ser pensado da seguinte forma:
>
> Dados um ponto A e uma reta d fixos, temos que construir duas retas x
> e y, com ângulo 'alfa' entre elas, ambas passando por A e tais que a
> distância entre os pontos X e Y, que elas geram ao intersectar d, seja
> mínima.
>
> Daí fica fácil argumentar que a altura por A também tem que ser a
> bissetriz por A.
>
> No fundo do fundo é uma forma de geometrizar a solução trigonométrica.
> A trigonometria se torna apenas um atalho.
>
> Vou formalizar isso mais tarde, com desenhos e tudo.
>
>
>
> >
> > Isso até me lembra o famoso artigo do Shine sobre geometria cearense
> > VS geometria paulista:
> > https://cyshine.webs.com/geometria-2005.pdf
> >
> >
> > >
> > > Saudações,
> > > PJMS
> > >
> > > Em ter., 18 de ago. de 2020 às 11:34, Claudio Buffara <
> claudio.buff...@gmail.com> escreveu:
> > >>
> > >> Será que tem uma demonstração mais geométrica e menos algébrica
> disso? E que torne o resultado mais intuitivo?
> > >> É razoável que o ponto P não esteja muito próximo de qualquer dos
> lados, pois neste caso, se P se aproximasse do lado a, por exemplo, a/h_a
> cresceria e a expressão se afastaria do valor mínimo.
> > >> Mas, com lados não necessariamente congruentes, não é óbvio, a
> priori, que P deva ser equidistante dos três.
> > >> De fato, seria razoável esperar que P estivesse mais próximo do maior
> lado e conjecturar, por exemplo, que o P que minimiza a expressão é tal que
> a/h_a = b/h_b = c/h_c.
> > >> O fato de P ser o incentro não me parece a conjectura mais evidente
> neste caso.
> > >>
> > >>
> > >> On Sun, Aug 16, 2020 at 10:11 AM Matheus Secco <
> matheusse...@gmail.com> wrote:
> > >>>
> > >>> Olá, Vanderlei.
> > >>> Por Cauchy-Schwarz, temos
> > >>>
> > >>> (a/ha + b/hb + c/hc) * (a*ha + b*hb + c*hc) >= (a+b+c)^2.  (#)
> > >>>
> > >>> Como (a*ha + b*hb + c*hc) = 2S, onde S é a área de ABC, segue que a
> expressão a/ha + b/hb + c/hc é pelo menos 2p^2/S, onde p é o semi-perimetro.
> > >>>
> > >>> Por outro lado, a igualdade em (#) ocorre se, e somente se, ha = hb
> = hc, ou seja, quando P é o incentro do triângulo
> > >>>
> > >>> Abraços,
> > >>> Matheus
> > >>>
> > >>> Em dom, 16 de ago de 2020 08:59, Professor Vanderlei Nemitz <
> vanderma...@gmail.com> escreveu:
> > 
> >  Bom dia!
> > 
> >  Tentei utilizar alguma desigualdade de médias aqui, mas não tive
> êxito. Alguém ajuda?
> >  Muito agradecido!
> > 
> >  Seja P um ponto no interior de um triângulo e sejam ha, hb e hc as
> distâncias de P aos lados a, b e c, respectivamente. Mostre que o valor
> mínimo de (a/ha) + (b/hb) + (c/hc) ocorre quando P é o incentivo do
> triângulo ABC.
> > 
> >  --
> >  Esta mensagem foi verificada pelo sistema de antivírus e
> >  acredita-se estar livre de perigo.
> > >>>
> > >>>
> > >>> --
> > >>> Esta mensagem foi verificada pelo sistema de antivírus e
> > >>> acredita-se estar livre de perigo.
> > >>
> > >>
> > >> --
> > >> Esta mensagem foi verificada pelo sistema de antivírus e
> > >> acredita-se estar livre de perigo.
> > >
> > >
> > > --
> > > Esta mensagem foi verificada pelo sistema de antivírus e
> > > acredita-se estar livre de perigo.
>
> --
> Esta mensagem foi verificada pelo sistema de antivírus e
>  

Re: [obm-l] Re: [obm-l] Re: [obm-l] Re: [obm-l] Elipse e lugar g eométrico

2020-08-25 Por tôpico qedtexte

Tenho um arquivo com uma figura mostrando as
elipses. Posso mandar no privado pra quem quiser.

Lus
--
Esta mensagem foi verificada pelo sistema de antiv�rus e
acredita-se estar livre de perigo.



[obm-l] Re: [obm-l] Re: [obm-l] Re: [obm-l] Re: [obm-l] Geometria plana com desigualdade de médias?

2020-08-20 Por tôpico Anderson Torres
Em qui., 20 de ago. de 2020 às 22:03, Anderson Torres
 escreveu:
>
> Em ter., 18 de ago. de 2020 às 19:51, Pedro José  
> escreveu:
> >
> > Boa noite!
> > Cláudio,
> > não consegui nada geométrico.
> > O máximo que atingi foi:
> > a/ha + b/hb + c/hc= [cotg(A1) +cotg (A2)]  + [cotg(B1) +cotg (B2)] + 
> > co[tg(C1) +cotg (C2)] com A1 + A2 = A; B1 + B2 + B e C1 + C2 = C.
> > Para ser mínimo cada termo entre colchetes deve ser mínimo, o que ocorre 
> > quando A1 = A2; B1 = B2 e C1 = C2. Logo P seria o encontro das bissetrizes 
> > e logo I.
> > Onde: A1= PAB e A2=PAC; B1=PBA e B2=PBC; C1=PCA e C2=PCB.
>
> Acho que daqui poderia sair uma interpretação mais escamoteada.
> Afinal, trigonometria é uma espécie de "ponto de contato" entre a
> geometria analítica e a sintética, entre a nuvem de desenhos e a de
> números.
>
> Acredito que a solução aqui seria arranjar uma interpretação
> geométrica desses colchetes de co-tangentes. Acredito que possamos
> apelar para Ptolomeu em algum momento ou para um macete de
> semelhanças, pois as projeções de um ponto sobre duas retas criam um
> quadrilátero cíclico.

Acrescentando mais coisas: se queremos minimizar cot(x) +cot(y) com
x+y fixo, isto é equivalente a minimizar tan(90-x)+tan(90-y) com
90-x+90-y fixo. Ou como maximizar tan(x) + tan(y) com x+y fixo.

Geometricamente, tangente é cateto oposto dividido por cateto
adjacente. Logo uma soma de tangentes com catetos adjacentes iguais
equivale a uma soma de catetos opostos! Assim sendo, nosso problema
pode ser pensado da seguinte forma:

Dados um ponto A e uma reta d fixos, temos que construir duas retas x
e y, com ângulo 'alfa' entre elas, ambas passando por A e tais que a
distância entre os pontos X e Y, que elas geram ao intersectar d, seja
mínima.

Daí fica fácil argumentar que a altura por A também tem que ser a
bissetriz por A.

No fundo do fundo é uma forma de geometrizar a solução trigonométrica.
A trigonometria se torna apenas um atalho.

Vou formalizar isso mais tarde, com desenhos e tudo.



>
> Isso até me lembra o famoso artigo do Shine sobre geometria cearense
> VS geometria paulista:
> https://cyshine.webs.com/geometria-2005.pdf
>
>
> >
> > Saudações,
> > PJMS
> >
> > Em ter., 18 de ago. de 2020 às 11:34, Claudio Buffara 
> >  escreveu:
> >>
> >> Será que tem uma demonstração mais geométrica e menos algébrica disso? E 
> >> que torne o resultado mais intuitivo?
> >> É razoável que o ponto P não esteja muito próximo de qualquer dos lados, 
> >> pois neste caso, se P se aproximasse do lado a, por exemplo, a/h_a 
> >> cresceria e a expressão se afastaria do valor mínimo.
> >> Mas, com lados não necessariamente congruentes, não é óbvio, a priori, que 
> >> P deva ser equidistante dos três.
> >> De fato, seria razoável esperar que P estivesse mais próximo do maior lado 
> >> e conjecturar, por exemplo, que o P que minimiza a expressão é tal que 
> >> a/h_a = b/h_b = c/h_c.
> >> O fato de P ser o incentro não me parece a conjectura mais evidente neste 
> >> caso.
> >>
> >>
> >> On Sun, Aug 16, 2020 at 10:11 AM Matheus Secco  
> >> wrote:
> >>>
> >>> Olá, Vanderlei.
> >>> Por Cauchy-Schwarz, temos
> >>>
> >>> (a/ha + b/hb + c/hc) * (a*ha + b*hb + c*hc) >= (a+b+c)^2.  (#)
> >>>
> >>> Como (a*ha + b*hb + c*hc) = 2S, onde S é a área de ABC, segue que a 
> >>> expressão a/ha + b/hb + c/hc é pelo menos 2p^2/S, onde p é o 
> >>> semi-perimetro.
> >>>
> >>> Por outro lado, a igualdade em (#) ocorre se, e somente se, ha = hb = hc, 
> >>> ou seja, quando P é o incentro do triângulo
> >>>
> >>> Abraços,
> >>> Matheus
> >>>
> >>> Em dom, 16 de ago de 2020 08:59, Professor Vanderlei Nemitz 
> >>>  escreveu:
> 
>  Bom dia!
> 
>  Tentei utilizar alguma desigualdade de médias aqui, mas não tive êxito. 
>  Alguém ajuda?
>  Muito agradecido!
> 
>  Seja P um ponto no interior de um triângulo e sejam ha, hb e hc as 
>  distâncias de P aos lados a, b e c, respectivamente. Mostre que o valor 
>  mínimo de (a/ha) + (b/hb) + (c/hc) ocorre quando P é o incentivo do 
>  triângulo ABC.
> 
>  --
>  Esta mensagem foi verificada pelo sistema de antivírus e
>  acredita-se estar livre de perigo.
> >>>
> >>>
> >>> --
> >>> Esta mensagem foi verificada pelo sistema de antivírus e
> >>> acredita-se estar livre de perigo.
> >>
> >>
> >> --
> >> Esta mensagem foi verificada pelo sistema de antivírus e
> >> acredita-se estar livre de perigo.
> >
> >
> > --
> > Esta mensagem foi verificada pelo sistema de antivírus e
> > acredita-se estar livre de perigo.

-- 
Esta mensagem foi verificada pelo sistema de antiv�rus e
 acredita-se estar livre de perigo.


=
Instru��es para entrar na lista, sair da lista e usar a lista em
http://www.mat.puc-rio.br/~obmlistas/obm-l.html
=


[obm-l] Re: [obm-l] Re: [obm-l] Re: [obm-l] Re: [obm-l] Geometria plana com desigualdade de médias?

2020-08-20 Por tôpico Anderson Torres
Em ter., 18 de ago. de 2020 às 19:51, Pedro José  escreveu:
>
> Boa noite!
> Cláudio,
> não consegui nada geométrico.
> O máximo que atingi foi:
> a/ha + b/hb + c/hc= [cotg(A1) +cotg (A2)]  + [cotg(B1) +cotg (B2)] + 
> co[tg(C1) +cotg (C2)] com A1 + A2 = A; B1 + B2 + B e C1 + C2 = C.
> Para ser mínimo cada termo entre colchetes deve ser mínimo, o que ocorre 
> quando A1 = A2; B1 = B2 e C1 = C2. Logo P seria o encontro das bissetrizes e 
> logo I.
> Onde: A1= PAB e A2=PAC; B1=PBA e B2=PBC; C1=PCA e C2=PCB.

Acho que daqui poderia sair uma interpretação mais escamoteada.
Afinal, trigonometria é uma espécie de "ponto de contato" entre a
geometria analítica e a sintética, entre a nuvem de desenhos e a de
números.

Acredito que a solução aqui seria arranjar uma interpretação
geométrica desses colchetes de co-tangentes. Acredito que possamos
apelar para Ptolomeu em algum momento ou para um macete de
semelhanças, pois as projeções de um ponto sobre duas retas criam um
quadrilátero cíclico.

Isso até me lembra o famoso artigo do Shine sobre geometria cearense
VS geometria paulista:
https://cyshine.webs.com/geometria-2005.pdf


>
> Saudações,
> PJMS
>
> Em ter., 18 de ago. de 2020 às 11:34, Claudio Buffara 
>  escreveu:
>>
>> Será que tem uma demonstração mais geométrica e menos algébrica disso? E que 
>> torne o resultado mais intuitivo?
>> É razoável que o ponto P não esteja muito próximo de qualquer dos lados, 
>> pois neste caso, se P se aproximasse do lado a, por exemplo, a/h_a cresceria 
>> e a expressão se afastaria do valor mínimo.
>> Mas, com lados não necessariamente congruentes, não é óbvio, a priori, que P 
>> deva ser equidistante dos três.
>> De fato, seria razoável esperar que P estivesse mais próximo do maior lado e 
>> conjecturar, por exemplo, que o P que minimiza a expressão é tal que a/h_a = 
>> b/h_b = c/h_c.
>> O fato de P ser o incentro não me parece a conjectura mais evidente neste 
>> caso.
>>
>>
>> On Sun, Aug 16, 2020 at 10:11 AM Matheus Secco  
>> wrote:
>>>
>>> Olá, Vanderlei.
>>> Por Cauchy-Schwarz, temos
>>>
>>> (a/ha + b/hb + c/hc) * (a*ha + b*hb + c*hc) >= (a+b+c)^2.  (#)
>>>
>>> Como (a*ha + b*hb + c*hc) = 2S, onde S é a área de ABC, segue que a 
>>> expressão a/ha + b/hb + c/hc é pelo menos 2p^2/S, onde p é o semi-perimetro.
>>>
>>> Por outro lado, a igualdade em (#) ocorre se, e somente se, ha = hb = hc, 
>>> ou seja, quando P é o incentro do triângulo
>>>
>>> Abraços,
>>> Matheus
>>>
>>> Em dom, 16 de ago de 2020 08:59, Professor Vanderlei Nemitz 
>>>  escreveu:

 Bom dia!

 Tentei utilizar alguma desigualdade de médias aqui, mas não tive êxito. 
 Alguém ajuda?
 Muito agradecido!

 Seja P um ponto no interior de um triângulo e sejam ha, hb e hc as 
 distâncias de P aos lados a, b e c, respectivamente. Mostre que o valor 
 mínimo de (a/ha) + (b/hb) + (c/hc) ocorre quando P é o incentivo do 
 triângulo ABC.

 --
 Esta mensagem foi verificada pelo sistema de antivírus e
 acredita-se estar livre de perigo.
>>>
>>>
>>> --
>>> Esta mensagem foi verificada pelo sistema de antivírus e
>>> acredita-se estar livre de perigo.
>>
>>
>> --
>> Esta mensagem foi verificada pelo sistema de antivírus e
>> acredita-se estar livre de perigo.
>
>
> --
> Esta mensagem foi verificada pelo sistema de antivírus e
> acredita-se estar livre de perigo.

-- 
Esta mensagem foi verificada pelo sistema de antiv�rus e
 acredita-se estar livre de perigo.


=
Instru��es para entrar na lista, sair da lista e usar a lista em
http://www.mat.puc-rio.br/~obmlistas/obm-l.html
=


[obm-l] Re: [obm-l] Re: [obm-l] Re: [obm-l] Re: [obm-l] Re: [obm-l] Re: [obm-l] Re: [obm-l] polinômio irredutível

2020-08-20 Por tôpico Anderson Torres
Em seg., 17 de ago. de 2020 às 12:14, Claudio Buffara
 escreveu:
>
> Eu acho que o Eisenstein inventou este critério pra polinômios da forma x^n + 
> a ou, mais geralmente, pra polinômios ciclotômicos.
> Daí funciona bem.
>
> On Mon, Aug 17, 2020 at 11:02 AM Esdras Muniz  
> wrote:
>>
>> E se p=3, e p divide N^2+9, então p^2 divide N^2+9.
>>
>> Então o critério de Eisenstein realmente não é tão abrangente. Será que tem 
>> algum outro critério que cubra casos em que o de Eisenstein não cubra?
>>
>> Em seg, 17 de ago de 2020 09:46, Claudio Buffara  
>> escreveu:
>>>
>>> Boa! Se p <> 3 mas p divide 3N e 3N^2, então p divide N ==> p não divide 
>>> N^3 + 9.
>>>
>>> On Sun, Aug 16, 2020 at 10:51 PM Esdras Muniz  
>>> wrote:

 Tenta com x^3+9.

 Em dom, 16 de ago de 2020 15:24, Claudio Buffara 
  escreveu:
>
> f(x) em Z[x], bem entendido...
>
>
> On Sun, Aug 16, 2020 at 3:08 PM Claudio Buffara 
>  wrote:
>>
>> Que tal essa aqui?
>> Prove ou disprove que, dado um polinômio f(x), irredutível sobre Q, 
>> existe um inteiro N tal que a irredutibilidade de f pode ser provada 
>> pelo critério de Eisenstein aplicado a f(x+N).

Isso me parece uma daquelas questões ultra capciosas sobre "prove ou
disprove que existe um algoritmo que..."

Inclusive imagino que esta seja uma questão indecidível neste caso particular...

>>
>> On Sun, Aug 16, 2020 at 2:31 PM Matheus Secco  
>> wrote:
>>>
>>> O melhor jeito é pensar na contrapositiva (supondo que você esteja 
>>> falando sobre irredutibilidade em Z[x] ou até em Q[x]): se f(x) fatora 
>>> como g(x)*h(x), então f(x+a) fatora como g(x+a) *h(x+a) e é claro que 
>>> uma vez que g(x) e h(x) têm coeficientes inteiros, então g(x+a) e 
>>> h(x+a) também têm. A recíproca é essencialmente idêntica.
>>>
>>> Abraços
>>>
>>> Em dom, 16 de ago de 2020 14:11, Luís Lopes  
>>> escreveu:

 Sauda,c~oes,

 Como provar que um polinômio f(x) tendo como coeficientes números 
 inteiros
 é irredutível se e somente se f(x+a) é irredutível para algum  
 inteiro ?

 Luís




 --
 Esta mensagem foi verificada pelo sistema de antivírus e
 acredita-se estar livre de perigo.
>>>
>>>
>>> --
>>> Esta mensagem foi verificada pelo sistema de antivírus e
>>> acredita-se estar livre de perigo.
>
>
> --
> Esta mensagem foi verificada pelo sistema de antivírus e
> acredita-se estar livre de perigo.


 --
 Esta mensagem foi verificada pelo sistema de antivírus e
 acredita-se estar livre de perigo.
>>>
>>>
>>> --
>>> Esta mensagem foi verificada pelo sistema de antivírus e
>>> acredita-se estar livre de perigo.
>>
>>
>> --
>> Esta mensagem foi verificada pelo sistema de antivírus e
>> acredita-se estar livre de perigo.
>
>
> --
> Esta mensagem foi verificada pelo sistema de antivírus e
> acredita-se estar livre de perigo.

-- 
Esta mensagem foi verificada pelo sistema de antiv�rus e
 acredita-se estar livre de perigo.


=
Instru��es para entrar na lista, sair da lista e usar a lista em
http://www.mat.puc-rio.br/~obmlistas/obm-l.html
=


[obm-l] Re: [obm-l] Re: [obm-l] Re: [obm-l] Re: [obm-l] Geometria plana com desigualdade de médias?

2020-08-18 Por tôpico Claudio Buffara
Realmente, não era isso que eu estava procurando...  mas valeu! É outra
solução.


On Tue, Aug 18, 2020 at 7:51 PM Pedro José  wrote:

> Boa noite!
> Cláudio,
> não consegui nada geométrico.
> O máximo que atingi foi:
> a/ha + b/hb + c/hc= [cotg(A1) +cotg (A2)]  + [cotg(B1) +cotg (B2)] +
> co[tg(C1) +cotg (C2)] com A1 + A2 = A; B1 + B2 + B e C1 + C2 = C.
> Para ser mínimo cada termo entre colchetes deve ser mínimo, o que ocorre
> quando A1 = A2; B1 = B2 e C1 = C2. Logo P seria o encontro das bissetrizes
> e logo I.
> Onde: A1= PAB e A2=PAC; B1=PBA e B2=PBC; C1=PCA e C2=PCB.
>
> Saudações,
> PJMS
>
> Em ter., 18 de ago. de 2020 às 11:34, Claudio Buffara <
> claudio.buff...@gmail.com> escreveu:
>
>> Será que tem uma demonstração mais geométrica e menos algébrica disso? E
>> que torne o resultado mais intuitivo?
>> É razoável que o ponto P não esteja muito próximo de qualquer dos lados,
>> pois neste caso, se P se aproximasse do lado a, por exemplo,
>> a/h_a cresceria e a expressão se afastaria do valor mínimo.
>> Mas, com lados não necessariamente congruentes, não é óbvio, a priori,
>> que P deva ser equidistante dos três.
>> De fato, seria razoável esperar que P estivesse mais próximo do maior
>> lado e conjecturar, por exemplo, que o P que minimiza a expressão é tal que
>> a/h_a = b/h_b = c/h_c.
>> O fato de P ser o incentro não me parece a conjectura mais evidente neste
>> caso.
>>
>>
>> On Sun, Aug 16, 2020 at 10:11 AM Matheus Secco 
>> wrote:
>>
>>> Olá, Vanderlei.
>>> Por Cauchy-Schwarz, temos
>>>
>>> (a/ha + b/hb + c/hc) * (a*ha + b*hb + c*hc) >= (a+b+c)^2.  (#)
>>>
>>> Como (a*ha + b*hb + c*hc) = 2S, onde S é a área de ABC, segue que a
>>> expressão a/ha + b/hb + c/hc é pelo menos 2p^2/S, onde p é o
>>> semi-perimetro.
>>>
>>> Por outro lado, a igualdade em (#) ocorre se, e somente se, ha = hb =
>>> hc, ou seja, quando P é o incentro do triângulo
>>>
>>> Abraços,
>>> Matheus
>>>
>>> Em dom, 16 de ago de 2020 08:59, Professor Vanderlei Nemitz <
>>> vanderma...@gmail.com> escreveu:
>>>
 Bom dia!

 Tentei utilizar alguma desigualdade de médias aqui, mas não tive êxito.
 Alguém ajuda?
 Muito agradecido!

 Seja P um ponto no interior de um triângulo e sejam ha, hb e hc as
 distâncias de P aos lados a, b e c, respectivamente. Mostre que o valor
 mínimo de (a/ha) + (b/hb) + (c/hc) ocorre quando P é o incentivo do
 triângulo ABC.

 --
 Esta mensagem foi verificada pelo sistema de antivírus e
 acredita-se estar livre de perigo.
>>>
>>>
>>> --
>>> Esta mensagem foi verificada pelo sistema de antivírus e
>>> acredita-se estar livre de perigo.
>>
>>
>> --
>> Esta mensagem foi verificada pelo sistema de antivírus e
>> acredita-se estar livre de perigo.
>
>
> --
> Esta mensagem foi verificada pelo sistema de antivírus e
> acredita-se estar livre de perigo.

-- 
Esta mensagem foi verificada pelo sistema de antiv�rus e
 acredita-se estar livre de perigo.



Re: [obm-l] Re: [obm-l] Re: [obm-l] Re: [obm-l] Re: [obm-l] Re: [obm-l] Re: [obm-l] polinômio irredutível

2020-08-17 Por tôpico qedtexte

Sauda,c~oes,

Legal o estudo dox^3+9.

Sobre oEisenstein generalizado (teorema 3 em

http://yufeizhao.com/olympiad/intpoly.pdf;), tenho duas 
dvidas:





Theorem 3(Extended Eisenstein).Letf(x) =anxn+an1xn1++a1x+a0be a polynomial with integer coefficients 
such thatp|aifor 0i  k,pﰀ|/akandp2ﰀ|/a0. Thenf(x) has an irreducible factor of degree greater thank.


Quando k=n obtm-se o critrio tradicional.

i) quais as condies para os outros coeficientes a_(k+1), a_(k+2), 
. , a_n ?
p pode dividi-los ou no ?

ii) o grau do fator irredutvel   k ou = k ?

Lus














--
Esta mensagem foi verificada pelo sistema de antiv�rus e
acredita-se estar livre de perigo.



[obm-l] Re: [obm-l] Re: [obm-l] Re: [obm-l] Re: [obm-l] Re: [obm-l] Re: [obm-l] polinômio irredutível

2020-08-17 Por tôpico Claudio Buffara
Eu acho que o Eisenstein inventou este critério pra polinômios da forma
x^n + a ou, mais geralmente, pra polinômios ciclotômicos.
Daí funciona bem.

On Mon, Aug 17, 2020 at 11:02 AM Esdras Muniz 
wrote:

> E se p=3, e p divide N^2+9, então p^2 divide N^2+9.
>
> Então o critério de Eisenstein realmente não é tão abrangente. Será que
> tem algum outro critério que cubra casos em que o de Eisenstein não cubra?
>
> Em seg, 17 de ago de 2020 09:46, Claudio Buffara <
> claudio.buff...@gmail.com> escreveu:
>
>> Boa! Se p <> 3 mas p divide 3N e 3N^2, então p divide N ==> p não divide
>> N^3 + 9.
>>
>> On Sun, Aug 16, 2020 at 10:51 PM Esdras Muniz 
>> wrote:
>>
>>> Tenta com x^3+9.
>>>
>>> Em dom, 16 de ago de 2020 15:24, Claudio Buffara <
>>> claudio.buff...@gmail.com> escreveu:
>>>
 f(x) em Z[x], bem entendido...


 On Sun, Aug 16, 2020 at 3:08 PM Claudio Buffara <
 claudio.buff...@gmail.com> wrote:

> Que tal essa aqui?
> Prove ou disprove que, dado um polinômio f(x), irredutível sobre Q,
> existe um inteiro N tal que a irredutibilidade de f pode ser provada pelo
> critério de Eisenstein aplicado a f(x+N).
>
> On Sun, Aug 16, 2020 at 2:31 PM Matheus Secco 
> wrote:
>
>> O melhor jeito é pensar na contrapositiva (supondo que você esteja
>> falando sobre irredutibilidade em Z[x] ou até em Q[x]): se f(x) fatora 
>> como
>> g(x)*h(x), então f(x+a) fatora como g(x+a) *h(x+a) e é claro que uma vez
>> que g(x) e h(x) têm coeficientes inteiros, então g(x+a) e h(x+a) também
>> têm. A recíproca é essencialmente idêntica.
>>
>> Abraços
>>
>> Em dom, 16 de ago de 2020 14:11, Luís Lopes 
>> escreveu:
>>
>>> Sauda,c~oes,
>>>
>>> Como provar que um polinômio f(x) tendo como coeficientes números
>>> inteiros
>>> é irredutível se e somente se f(x+a) é irredutível para algum 
>>> inteiro ?
>>>
>>> Luís
>>>
>>>
>>>
>>>
>>> --
>>> Esta mensagem foi verificada pelo sistema de antivírus e
>>> acredita-se estar livre de perigo.
>>>
>>
>> --
>> Esta mensagem foi verificada pelo sistema de antivírus e
>> acredita-se estar livre de perigo.
>
>
 --
 Esta mensagem foi verificada pelo sistema de antivírus e
 acredita-se estar livre de perigo.
>>>
>>>
>>> --
>>> Esta mensagem foi verificada pelo sistema de antivírus e
>>> acredita-se estar livre de perigo.
>>
>>
>> --
>> Esta mensagem foi verificada pelo sistema de antivírus e
>> acredita-se estar livre de perigo.
>
>
> --
> Esta mensagem foi verificada pelo sistema de antivírus e
> acredita-se estar livre de perigo.

-- 
Esta mensagem foi verificada pelo sistema de antiv�rus e
 acredita-se estar livre de perigo.



[obm-l] Re: [obm-l] Re: [obm-l] Re: [obm-l] Re: [obm-l] Re: [obm-l] polinômio irredutível

2020-08-17 Por tôpico Esdras Muniz
E se p=3, e p divide N^2+9, então p^2 divide N^2+9.

Então o critério de Eisenstein realmente não é tão abrangente. Será que tem
algum outro critério que cubra casos em que o de Eisenstein não cubra?

Em seg, 17 de ago de 2020 09:46, Claudio Buffara 
escreveu:

> Boa! Se p <> 3 mas p divide 3N e 3N^2, então p divide N ==> p não divide
> N^3 + 9.
>
> On Sun, Aug 16, 2020 at 10:51 PM Esdras Muniz 
> wrote:
>
>> Tenta com x^3+9.
>>
>> Em dom, 16 de ago de 2020 15:24, Claudio Buffara <
>> claudio.buff...@gmail.com> escreveu:
>>
>>> f(x) em Z[x], bem entendido...
>>>
>>>
>>> On Sun, Aug 16, 2020 at 3:08 PM Claudio Buffara <
>>> claudio.buff...@gmail.com> wrote:
>>>
 Que tal essa aqui?
 Prove ou disprove que, dado um polinômio f(x), irredutível sobre Q,
 existe um inteiro N tal que a irredutibilidade de f pode ser provada pelo
 critério de Eisenstein aplicado a f(x+N).

 On Sun, Aug 16, 2020 at 2:31 PM Matheus Secco 
 wrote:

> O melhor jeito é pensar na contrapositiva (supondo que você esteja
> falando sobre irredutibilidade em Z[x] ou até em Q[x]): se f(x) fatora 
> como
> g(x)*h(x), então f(x+a) fatora como g(x+a) *h(x+a) e é claro que uma vez
> que g(x) e h(x) têm coeficientes inteiros, então g(x+a) e h(x+a) também
> têm. A recíproca é essencialmente idêntica.
>
> Abraços
>
> Em dom, 16 de ago de 2020 14:11, Luís Lopes 
> escreveu:
>
>> Sauda,c~oes,
>>
>> Como provar que um polinômio f(x) tendo como coeficientes números
>> inteiros
>> é irredutível se e somente se f(x+a) é irredutível para algum 
>> inteiro ?
>>
>> Luís
>>
>>
>>
>>
>> --
>> Esta mensagem foi verificada pelo sistema de antivírus e
>> acredita-se estar livre de perigo.
>>
>
> --
> Esta mensagem foi verificada pelo sistema de antivírus e
> acredita-se estar livre de perigo.


>>> --
>>> Esta mensagem foi verificada pelo sistema de antivírus e
>>> acredita-se estar livre de perigo.
>>
>>
>> --
>> Esta mensagem foi verificada pelo sistema de antivírus e
>> acredita-se estar livre de perigo.
>
>
> --
> Esta mensagem foi verificada pelo sistema de antivírus e
> acredita-se estar livre de perigo.

-- 
Esta mensagem foi verificada pelo sistema de antiv�rus e
 acredita-se estar livre de perigo.



[obm-l] Re: [obm-l] Re: [obm-l] Re: [obm-l] Re: [obm-l] polinômio irredutível

2020-08-17 Por tôpico Claudio Buffara
Boa! Se p <> 3 mas p divide 3N e 3N^2, então p divide N ==> p não divide
N^3 + 9.

On Sun, Aug 16, 2020 at 10:51 PM Esdras Muniz 
wrote:

> Tenta com x^3+9.
>
> Em dom, 16 de ago de 2020 15:24, Claudio Buffara <
> claudio.buff...@gmail.com> escreveu:
>
>> f(x) em Z[x], bem entendido...
>>
>>
>> On Sun, Aug 16, 2020 at 3:08 PM Claudio Buffara <
>> claudio.buff...@gmail.com> wrote:
>>
>>> Que tal essa aqui?
>>> Prove ou disprove que, dado um polinômio f(x), irredutível sobre Q,
>>> existe um inteiro N tal que a irredutibilidade de f pode ser provada pelo
>>> critério de Eisenstein aplicado a f(x+N).
>>>
>>> On Sun, Aug 16, 2020 at 2:31 PM Matheus Secco 
>>> wrote:
>>>
 O melhor jeito é pensar na contrapositiva (supondo que você esteja
 falando sobre irredutibilidade em Z[x] ou até em Q[x]): se f(x) fatora como
 g(x)*h(x), então f(x+a) fatora como g(x+a) *h(x+a) e é claro que uma vez
 que g(x) e h(x) têm coeficientes inteiros, então g(x+a) e h(x+a) também
 têm. A recíproca é essencialmente idêntica.

 Abraços

 Em dom, 16 de ago de 2020 14:11, Luís Lopes 
 escreveu:

> Sauda,c~oes,
>
> Como provar que um polinômio f(x) tendo como coeficientes números
> inteiros
> é irredutível se e somente se f(x+a) é irredutível para algum 
> inteiro ?
>
> Luís
>
>
>
>
> --
> Esta mensagem foi verificada pelo sistema de antivírus e
> acredita-se estar livre de perigo.
>

 --
 Esta mensagem foi verificada pelo sistema de antivírus e
 acredita-se estar livre de perigo.
>>>
>>>
>> --
>> Esta mensagem foi verificada pelo sistema de antivírus e
>> acredita-se estar livre de perigo.
>
>
> --
> Esta mensagem foi verificada pelo sistema de antivírus e
> acredita-se estar livre de perigo.

-- 
Esta mensagem foi verificada pelo sistema de antiv�rus e
 acredita-se estar livre de perigo.



[obm-l] Re: [obm-l] Re: [obm-l] Re: [obm-l] Re: [obm-l] Decrescimento de Funções Exponenciais

2020-05-13 Por tôpico Luiz Antonio Rodrigues
Olá, Ralph!
Tudo bem?
Muito obrigado!
Vou acessar os links!
Abraço!
Luiz


Em ter, 12 de mai de 2020 8:35 PM, Ralph Costa Teixeira 
escreveu:

> Bom, o assunto me parece ser "crescimento/decrescimento assintótico"...
> Não consigo pensar num texto para recomendar, mas olhe aqui:
> https://en.wikipedia.org/wiki/Big_O_notation
> E, em especial:
> https://en.wikipedia.org/wiki/Big_O_notation#Little-o_notation
>
> Abraço, Ralph.
>
> On Tue, May 12, 2020 at 7:09 PM Luiz Antonio Rodrigues <
> rodrigue...@gmail.com> wrote:
>
>> Olá, Ralph!
>> Tudo bem?
>> Sim, melhorou muito!
>> Muito obrigado!
>> Então, na função (5), nós temos uma incerteza...
>> Eu não havia percebido isso...
>> Muito interessante...
>> Vou ler mais sobre o assunto...
>> Você conhece algum bom livro que trate disso com mais profundidade?
>> Abraço!
>> Luiz
>>
>>
>> Em ter, 12 de mai de 2020 3:04 PM, Ralph Costa Teixeira <
>> ralp...@gmail.com> escreveu:
>>
>>> O assunto é delicado. Primeiro, precisamos de uma boa definição de
>>> "decresce mais rápido" (a gente diz que as exponenciais decrescem rápido,
>>> mas se você ler **ao pé da letra** isso é falso! A velocidade delas vai
>>> para 0 quando t vai para infinito... ou seja, elas decrescem mito
>>> devagar!?!?). Para esclarecer, suponho que queremos usar esta aqui:
>>>
>>> DEF. f(x) decresce (para 0) mais rápido (quando x vai para +Inf) do que
>>> g(x) quando lim f(x)/g(x) =0 (quando x vai para +Inf).
>>>
>>> Agora sim, você resolve tudo:
>>>
>>> 1) lim h(x)^2/h(x) = 0, portanto h^2 decresce mais rapido que h;
>>> 2) lim g(x)^2/h(x) = lim g(x)/h(x) . g(x) = 0.0=0, portanto g^2 decresce
>>> mais rapido que h;
>>> 3) lim f(x)*g(x)/h(x) = lim f(x) * (g(x)/h(x)) =0 (com f limitada),
>>> portanto fg decresce mais rapido que h;
>>> 4) lim sqrt(h)/h = lim 1/sqrt(h) =+Inf; assim, lim h/sqrt(h) = 0, ou
>>> seja, h decresce mais rapido que sqrt(h);
>>> 5) lim sqrt(g)/h = ??? Nao da para saber. Poderia ser g(x)=1/x^n e
>>> h(x)=1/x. Tomando n<2 ou n>2 podemos obter ambos comportamentos.
>>>
>>> Melhorou?
>>>
>>> Abraço, Ralph.
>>>
>>> On Tue, May 12, 2020 at 9:52 AM Luiz Antonio Rodrigues <
>>> rodrigue...@gmail.com> wrote:
>>>
 Olá, pessoal!

 Bom dia!

 Tudo bem?

 Estou tentando resolver um problema há uns 10 dias.

 Já tentei de tudo e estou com dúvidas.

 O problema é o seguinte:

 São dadas duas funções: h(x) e g(x).

 A função g(x) tende a zero mais rápido do que h(x), quando x tende a
 infinito.

 O problema pede que as seguintes funções sejam comparadas com h(x):


1.

(h(x))^2
2.

(g(x))^2
3.

f(x)*g(x)
4.

sqrt(h(x))
5.

sqrt(g(x))


 A pergunta é: quais dessas funções decrescem mais rápido do que h(x),
 quando x tende a infinito?

 Eu usei, entre outras, as seguintes funções:


 1/ln(x)

 1/x

 1/x^5

 1/e^x


 Utilizei a regra de L’Hospital e descobri que a única função que não
 decresce mais rápido do que h(x) é a (4).

 Também utilizei softwares gráficos e confirmei o meu resultado.

 Só sei que a resposta não está correta, mas ainda não sei qual seria a
 solução.

 Não consigo entender o motivo...

 Será que preciso achar um contra-exemplo?

 Alguém pode me ajudar?

 Muito obrigado!

 Abraços!

 Luiz

 --
 Esta mensagem foi verificada pelo sistema de antivírus e
 acredita-se estar livre de perigo.
>>>
>>>
>>> --
>>> Esta mensagem foi verificada pelo sistema de antivírus e
>>> acredita-se estar livre de perigo.
>>
>>
>> --
>> Esta mensagem foi verificada pelo sistema de antivírus e
>> acredita-se estar livre de perigo.
>
>
> --
> Esta mensagem foi verificada pelo sistema de antivírus e
> acredita-se estar livre de perigo.

-- 
Esta mensagem foi verificada pelo sistema de antiv�rus e
 acredita-se estar livre de perigo.



[obm-l] Re: [obm-l] Re: [obm-l] Re: [obm-l] Re: [obm-l] Teoria dos números

2020-03-22 Por tôpico Israel Meireles Chrisostomo
O meu sonho tmbm é esse kk

Em dom., 22 de mar. de 2020 às 13:22, Israel Meireles Chrisostomo <
israelmchrisost...@gmail.com> escreveu:

> vc é engenheiro?
>
> Em dom., 22 de mar. de 2020 às 13:19, Israel Meireles Chrisostomo <
> israelmchrisost...@gmail.com> escreveu:
>
>> mas vc possui algum graduação ?
>>
>> Em dom., 22 de mar. de 2020 às 13:00, Pedro José 
>> escreveu:
>>
>>> Boa tarde!
>>> Perfeita a sua correção.
>>> Quanto ao questionamento, nem tenho formação em matemática, meu sonho é
>>> cursar no IMPA ao me aposentar. Sou pitaqueiro. Ouço um assunto que não
>>> conheço, tento aprendê-lo. Na verdade, gosto de matemática. Talvez seja ela
>>> o "Mundo das ideias", o mundo ideal, a qual Platão se referiu.
>>> Saudações,
>>> PJMS
>>>
>>> Em dom, 22 de mar de 2020 12:25, Israel Meireles Chrisostomo <
>>> israelmchrisost...@gmail.com> escreveu:
>>>
 Acho q tem uma ´pequena correção no seguinte passo "4k+1. Pegando os
 fatores (4n-1)^2 e (4n+1)^2, teremos que 2^6 |p(n) para qualquer n=4k+1."O
 correto seria "Para n=4k+1.Pegando os fatores (n-1)^2 e (n+1)^2"

 Em dom., 22 de mar. de 2020 às 10:14, Israel Meireles Chrisostomo <
 israelmchrisost...@gmail.com> escreveu:

> Primeiramente obrigado pela solução.Mas Pedro, tenho uma pergunta :   o
> sr. é professor de Matemática?
>
> Em dom., 22 de mar. de 2020 às 01:34, Pedro José 
> escreveu:
>
>> Bom dia!
>> Dei uma mancada.
>> O expoente de 3 é 3 e não 2.
>> Retornando às classes mod 3.
>> Ao último fator é côngruo à (n-1)*n
>> Para n=3k aparece outro fator e 3^3|p(n), n=3k.
>> n=3k+1, tenho (n-1)^2 e (n-1), 3^3|p(n), n=3k+1
>> n=3k+2, tenho(n-2)^2 é (n+1)^2, 3^3|p(n), n=3k+2,
>> Logo 3^3|p(n) para todo n inteiro.
>> D>=2^6*3^3*5. Mas D<=2^6*3^3*5, então D=8640
>> Desculpem-me pelo erro.
>> Saudações,
>> PJMS.
>>
>>
>>
>> Em sáb, 21 de mar de 2020 13:20, Pedro José 
>> escreveu:
>>
>>> Boa tarde!
>>> Nem carece método numérico.
>>> Para n=1 ou n=0 ou n=2 temos que qualquer inteiro divide o polinômio
>>> p(n)=(n-2)^2*(n-1)^2*n^2*(n+1)^2*(4n^2-4n-9)
>>>
>>> p(3)=8640
>>> p(4)=561600 então (p(3),p(4))=8640=2^6*3^3*5.
>>> Seja D o maior inteiro que divide p(n) para todo n inteiro, D<=8640
>>> Vamos pegar as classes de equivalência mod 4. Seja k um inteiro.
>>> Para 4k temos que n^2= 16k^2 e (n-2) é par logo (n-2)^2= 4s^2 com s
>>> inteiro. Logo 2^6 divide p(n) para qualquer n =4k.
>>> 4k+1. Pegando os fatores (4n-1)^2 e (4n+1)^2, teremos que 2^6 |p(n)
>>> para qualquer n=4k+1.
>>> 4k+2. Pegando (n-2)^2 e n^2, teremos que 2^6|p(n) para qualquer
>>> n=4k+2
>>> 4k+3, pegando os mesmos fatores de 4k+1, 2^6|p(n) para n=4k+3.
>>> Portanto 2^6|p(n) para qualquer inteiro
>>> Agora classes de equivalência mod 3
>>> 3k, pegando n^2, 3^2|p(n) para n=3k
>>> 3k+1, pegando (n-1)^2; 3^2| p(n), n=3k+1
>>> 3k+2, pegando (n-2)^2, 3^2| p(n), n=3k+2
>>> Daí 3^2|p(n) para qualquer n inteiro.
>>> Classes de equivalência mod 5.
>>> 5k, n^2 , 5 |p(n), n =5
>>> 5k +1, (n-1)^2, 5|p(n), n=5k+1
>>> 5k+2, (n-2)^2, 5|p(n), n=5k+2
>>> 5k+3, (4n^2-4n-9)=(100k^2-100k+15)
>>> 5|p(n), n=5k+3
>>> 5k+4, (n+1)^2, 5|p(n) , n=5k+4.
>>> Então 5|p(n) para todo inteiro
>>> D>=2^6*3^2×*5
>>> Mas D<=2^6*3^2*5, logo D=8640
>>>
>>> Saudações,
>>> PJMS
>>>
>>> Em sáb, 21 de mar de 2020 04:39, Pedro José 
>>> escreveu:
>>>
 Bom dia!
 Falta de novo, em seu questionamento, informar que n é inteiro ou
 natural e colocar a condição para qualquer valor de n. Chamando o 
 polinômio
 de p(n)
 Para n=0, 1 ou 2, qualquer inteiro divide.
 Faria mdc(p(3),p(4))= A1
 Se der "pequeno", com poucos fatores primos e expoentes pequenos.
 Paro em A1, se não.
 (p(5),A1)=A2 uso o mesmo critério de parar
 (p(6),A2)=A3 até parar em:
 Ai=(p(i+3),A(i-1)).
 Aí faço o polinômio módfi^xi, onde fi é um fator primo de Aí e xi
 seu expoente. verifico se para cada resíduon= 1, 2...fi^n-1 se P(n)=0 
 mod
 fi^si
 Se falhar diminuto xi em 1 e repito o teste para todos resíduos de
 fi^(xi-1)-1 até um dado xki em que todos os p(resíduos) foram 
 equivalente a
 zero módulo fi^xki ou quando fizer para o expoente 1  e não zerar para
 todos resíduos de fi, quando o fator será descartado.
 Depois repito para cada fator primo f e seu respectivo expoente.
 Ao final D = Produtório de cada fator fi elevado ao expoente xki
 que zerou p(n) mod fi^xki para todos os resíduos, descartando os fí em 
 que
 xji chegou a 1 e não atendeu ou considerando nesse caso xki=0.

 Mas resolveria por método numérico.
 Depois poste sua solução.

 

[obm-l] Re: [obm-l] Re: [obm-l] Re: [obm-l] Re: [obm-l] Teoria dos números

2020-03-22 Por tôpico Israel Meireles Chrisostomo
vc é engenheiro?

Em dom., 22 de mar. de 2020 às 13:19, Israel Meireles Chrisostomo <
israelmchrisost...@gmail.com> escreveu:

> mas vc possui algum graduação ?
>
> Em dom., 22 de mar. de 2020 às 13:00, Pedro José 
> escreveu:
>
>> Boa tarde!
>> Perfeita a sua correção.
>> Quanto ao questionamento, nem tenho formação em matemática, meu sonho é
>> cursar no IMPA ao me aposentar. Sou pitaqueiro. Ouço um assunto que não
>> conheço, tento aprendê-lo. Na verdade, gosto de matemática. Talvez seja ela
>> o "Mundo das ideias", o mundo ideal, a qual Platão se referiu.
>> Saudações,
>> PJMS
>>
>> Em dom, 22 de mar de 2020 12:25, Israel Meireles Chrisostomo <
>> israelmchrisost...@gmail.com> escreveu:
>>
>>> Acho q tem uma ´pequena correção no seguinte passo "4k+1. Pegando os
>>> fatores (4n-1)^2 e (4n+1)^2, teremos que 2^6 |p(n) para qualquer n=4k+1."O
>>> correto seria "Para n=4k+1.Pegando os fatores (n-1)^2 e (n+1)^2"
>>>
>>> Em dom., 22 de mar. de 2020 às 10:14, Israel Meireles Chrisostomo <
>>> israelmchrisost...@gmail.com> escreveu:
>>>
 Primeiramente obrigado pela solução.Mas Pedro, tenho uma pergunta :   o
 sr. é professor de Matemática?

 Em dom., 22 de mar. de 2020 às 01:34, Pedro José 
 escreveu:

> Bom dia!
> Dei uma mancada.
> O expoente de 3 é 3 e não 2.
> Retornando às classes mod 3.
> Ao último fator é côngruo à (n-1)*n
> Para n=3k aparece outro fator e 3^3|p(n), n=3k.
> n=3k+1, tenho (n-1)^2 e (n-1), 3^3|p(n), n=3k+1
> n=3k+2, tenho(n-2)^2 é (n+1)^2, 3^3|p(n), n=3k+2,
> Logo 3^3|p(n) para todo n inteiro.
> D>=2^6*3^3*5. Mas D<=2^6*3^3*5, então D=8640
> Desculpem-me pelo erro.
> Saudações,
> PJMS.
>
>
>
> Em sáb, 21 de mar de 2020 13:20, Pedro José 
> escreveu:
>
>> Boa tarde!
>> Nem carece método numérico.
>> Para n=1 ou n=0 ou n=2 temos que qualquer inteiro divide o polinômio
>> p(n)=(n-2)^2*(n-1)^2*n^2*(n+1)^2*(4n^2-4n-9)
>>
>> p(3)=8640
>> p(4)=561600 então (p(3),p(4))=8640=2^6*3^3*5.
>> Seja D o maior inteiro que divide p(n) para todo n inteiro, D<=8640
>> Vamos pegar as classes de equivalência mod 4. Seja k um inteiro.
>> Para 4k temos que n^2= 16k^2 e (n-2) é par logo (n-2)^2= 4s^2 com s
>> inteiro. Logo 2^6 divide p(n) para qualquer n =4k.
>> 4k+1. Pegando os fatores (4n-1)^2 e (4n+1)^2, teremos que 2^6 |p(n)
>> para qualquer n=4k+1.
>> 4k+2. Pegando (n-2)^2 e n^2, teremos que 2^6|p(n) para qualquer n=4k+2
>> 4k+3, pegando os mesmos fatores de 4k+1, 2^6|p(n) para n=4k+3.
>> Portanto 2^6|p(n) para qualquer inteiro
>> Agora classes de equivalência mod 3
>> 3k, pegando n^2, 3^2|p(n) para n=3k
>> 3k+1, pegando (n-1)^2; 3^2| p(n), n=3k+1
>> 3k+2, pegando (n-2)^2, 3^2| p(n), n=3k+2
>> Daí 3^2|p(n) para qualquer n inteiro.
>> Classes de equivalência mod 5.
>> 5k, n^2 , 5 |p(n), n =5
>> 5k +1, (n-1)^2, 5|p(n), n=5k+1
>> 5k+2, (n-2)^2, 5|p(n), n=5k+2
>> 5k+3, (4n^2-4n-9)=(100k^2-100k+15)
>> 5|p(n), n=5k+3
>> 5k+4, (n+1)^2, 5|p(n) , n=5k+4.
>> Então 5|p(n) para todo inteiro
>> D>=2^6*3^2×*5
>> Mas D<=2^6*3^2*5, logo D=8640
>>
>> Saudações,
>> PJMS
>>
>> Em sáb, 21 de mar de 2020 04:39, Pedro José 
>> escreveu:
>>
>>> Bom dia!
>>> Falta de novo, em seu questionamento, informar que n é inteiro ou
>>> natural e colocar a condição para qualquer valor de n. Chamando o 
>>> polinômio
>>> de p(n)
>>> Para n=0, 1 ou 2, qualquer inteiro divide.
>>> Faria mdc(p(3),p(4))= A1
>>> Se der "pequeno", com poucos fatores primos e expoentes pequenos.
>>> Paro em A1, se não.
>>> (p(5),A1)=A2 uso o mesmo critério de parar
>>> (p(6),A2)=A3 até parar em:
>>> Ai=(p(i+3),A(i-1)).
>>> Aí faço o polinômio módfi^xi, onde fi é um fator primo de Aí e xi
>>> seu expoente. verifico se para cada resíduon= 1, 2...fi^n-1 se P(n)=0 
>>> mod
>>> fi^si
>>> Se falhar diminuto xi em 1 e repito o teste para todos resíduos de
>>> fi^(xi-1)-1 até um dado xki em que todos os p(resíduos) foram 
>>> equivalente a
>>> zero módulo fi^xki ou quando fizer para o expoente 1  e não zerar para
>>> todos resíduos de fi, quando o fator será descartado.
>>> Depois repito para cada fator primo f e seu respectivo expoente.
>>> Ao final D = Produtório de cada fator fi elevado ao expoente xki que
>>> zerou p(n) mod fi^xki para todos os resíduos, descartando os fí em que 
>>> xji
>>> chegou a 1 e não atendeu ou considerando nesse caso xki=0.
>>>
>>> Mas resolveria por método numérico.
>>> Depois poste sua solução.
>>>
>>> Saudações,
>>> PJMS.
>>>
>>>
>>>
>>>
>>> Em sex, 20 de mar de 2020 12:42, Israel Meireles Chrisostomo <
>>> israelmchrisost...@gmail.com> escreveu:
>>>
 Qual o maior inteiro que divide (n - 2)^2 (n - 1)^2 n^2 (n + 1)^2
 (4 n^2 

[obm-l] Re: [obm-l] Re: [obm-l] Re: [obm-l] Re: [obm-l] Teoria dos números

2020-03-22 Por tôpico Israel Meireles Chrisostomo
mas vc possui algum graduação ?

Em dom., 22 de mar. de 2020 às 13:00, Pedro José 
escreveu:

> Boa tarde!
> Perfeita a sua correção.
> Quanto ao questionamento, nem tenho formação em matemática, meu sonho é
> cursar no IMPA ao me aposentar. Sou pitaqueiro. Ouço um assunto que não
> conheço, tento aprendê-lo. Na verdade, gosto de matemática. Talvez seja ela
> o "Mundo das ideias", o mundo ideal, a qual Platão se referiu.
> Saudações,
> PJMS
>
> Em dom, 22 de mar de 2020 12:25, Israel Meireles Chrisostomo <
> israelmchrisost...@gmail.com> escreveu:
>
>> Acho q tem uma ´pequena correção no seguinte passo "4k+1. Pegando os
>> fatores (4n-1)^2 e (4n+1)^2, teremos que 2^6 |p(n) para qualquer n=4k+1."O
>> correto seria "Para n=4k+1.Pegando os fatores (n-1)^2 e (n+1)^2"
>>
>> Em dom., 22 de mar. de 2020 às 10:14, Israel Meireles Chrisostomo <
>> israelmchrisost...@gmail.com> escreveu:
>>
>>> Primeiramente obrigado pela solução.Mas Pedro, tenho uma pergunta :   o
>>> sr. é professor de Matemática?
>>>
>>> Em dom., 22 de mar. de 2020 às 01:34, Pedro José 
>>> escreveu:
>>>
 Bom dia!
 Dei uma mancada.
 O expoente de 3 é 3 e não 2.
 Retornando às classes mod 3.
 Ao último fator é côngruo à (n-1)*n
 Para n=3k aparece outro fator e 3^3|p(n), n=3k.
 n=3k+1, tenho (n-1)^2 e (n-1), 3^3|p(n), n=3k+1
 n=3k+2, tenho(n-2)^2 é (n+1)^2, 3^3|p(n), n=3k+2,
 Logo 3^3|p(n) para todo n inteiro.
 D>=2^6*3^3*5. Mas D<=2^6*3^3*5, então D=8640
 Desculpem-me pelo erro.
 Saudações,
 PJMS.



 Em sáb, 21 de mar de 2020 13:20, Pedro José 
 escreveu:

> Boa tarde!
> Nem carece método numérico.
> Para n=1 ou n=0 ou n=2 temos que qualquer inteiro divide o polinômio
> p(n)=(n-2)^2*(n-1)^2*n^2*(n+1)^2*(4n^2-4n-9)
>
> p(3)=8640
> p(4)=561600 então (p(3),p(4))=8640=2^6*3^3*5.
> Seja D o maior inteiro que divide p(n) para todo n inteiro, D<=8640
> Vamos pegar as classes de equivalência mod 4. Seja k um inteiro.
> Para 4k temos que n^2= 16k^2 e (n-2) é par logo (n-2)^2= 4s^2 com s
> inteiro. Logo 2^6 divide p(n) para qualquer n =4k.
> 4k+1. Pegando os fatores (4n-1)^2 e (4n+1)^2, teremos que 2^6 |p(n)
> para qualquer n=4k+1.
> 4k+2. Pegando (n-2)^2 e n^2, teremos que 2^6|p(n) para qualquer n=4k+2
> 4k+3, pegando os mesmos fatores de 4k+1, 2^6|p(n) para n=4k+3.
> Portanto 2^6|p(n) para qualquer inteiro
> Agora classes de equivalência mod 3
> 3k, pegando n^2, 3^2|p(n) para n=3k
> 3k+1, pegando (n-1)^2; 3^2| p(n), n=3k+1
> 3k+2, pegando (n-2)^2, 3^2| p(n), n=3k+2
> Daí 3^2|p(n) para qualquer n inteiro.
> Classes de equivalência mod 5.
> 5k, n^2 , 5 |p(n), n =5
> 5k +1, (n-1)^2, 5|p(n), n=5k+1
> 5k+2, (n-2)^2, 5|p(n), n=5k+2
> 5k+3, (4n^2-4n-9)=(100k^2-100k+15)
> 5|p(n), n=5k+3
> 5k+4, (n+1)^2, 5|p(n) , n=5k+4.
> Então 5|p(n) para todo inteiro
> D>=2^6*3^2×*5
> Mas D<=2^6*3^2*5, logo D=8640
>
> Saudações,
> PJMS
>
> Em sáb, 21 de mar de 2020 04:39, Pedro José 
> escreveu:
>
>> Bom dia!
>> Falta de novo, em seu questionamento, informar que n é inteiro ou
>> natural e colocar a condição para qualquer valor de n. Chamando o 
>> polinômio
>> de p(n)
>> Para n=0, 1 ou 2, qualquer inteiro divide.
>> Faria mdc(p(3),p(4))= A1
>> Se der "pequeno", com poucos fatores primos e expoentes pequenos.
>> Paro em A1, se não.
>> (p(5),A1)=A2 uso o mesmo critério de parar
>> (p(6),A2)=A3 até parar em:
>> Ai=(p(i+3),A(i-1)).
>> Aí faço o polinômio módfi^xi, onde fi é um fator primo de Aí e xi seu
>> expoente. verifico se para cada resíduon= 1, 2...fi^n-1 se P(n)=0 mod 
>> fi^si
>> Se falhar diminuto xi em 1 e repito o teste para todos resíduos de
>> fi^(xi-1)-1 até um dado xki em que todos os p(resíduos) foram 
>> equivalente a
>> zero módulo fi^xki ou quando fizer para o expoente 1  e não zerar para
>> todos resíduos de fi, quando o fator será descartado.
>> Depois repito para cada fator primo f e seu respectivo expoente.
>> Ao final D = Produtório de cada fator fi elevado ao expoente xki que
>> zerou p(n) mod fi^xki para todos os resíduos, descartando os fí em que 
>> xji
>> chegou a 1 e não atendeu ou considerando nesse caso xki=0.
>>
>> Mas resolveria por método numérico.
>> Depois poste sua solução.
>>
>> Saudações,
>> PJMS.
>>
>>
>>
>>
>> Em sex, 20 de mar de 2020 12:42, Israel Meireles Chrisostomo <
>> israelmchrisost...@gmail.com> escreveu:
>>
>>> Qual o maior inteiro que divide (n - 2)^2 (n - 1)^2 n^2 (n + 1)^2 (4
>>> n^2 - 4 n - 9))?
>>> Eu sei resolver esse problema com meu algoritmo, porém gostaria de
>>> saber como os colegas o resolveriam.
>>> --
>>> Israel Meireles Chrisostomo
>>>
>>> --
>>> Esta mensagem foi verificada pelo sistema de antivírus e

[obm-l] Re: [obm-l] Re: [obm-l] Re: [obm-l] Re: [obm-l] Re: [obm-l] Re: [obm-l] Teoria dos números

2020-03-17 Por tôpico Pedro José
Boa noite!
Aí, como dizia minha falecida vó, são outros quinhentos.
Como nas propostas anteriores n era natural. Vamos seguir nessa linha, se
não for reformule o problema.
Seja f(n)=   n (427 - 90n - 70n^2 + 45n^3 + 18n^4)
f(0)=0 qualquer natural divide, portanto, é indiferente.
f(1)= 330
f(2)= 1230
É fácil verificar que mdc(330,1230)=30 então D<=30, onde D é o máximo
inteiro que divide f(n) para todo n natural.
f(n) = 7n +5n^4 + 8 n^5 mod 10.
f(0)=0 mod10
f(1)= 20 = 0 mod10
f(2)= 350= 0 mod10
f(3)= 2370 = 0 mod10
f(4)= 9500 = 0 mod10
f(5)= 28160 = 0 mod10
f(6)=68730 = 0 mod10
f(7)=146510 = 0 mod10
f(8)=282680 = 0 mod10
f(9)=505260 = 0 mod10
logo 10 | f(n) para qualquer n natural.

f(n) = n -n^3 mod 3
f(0) = 0 mod 3
f(1) = 0 mod 3
f(2)= -6 = 0 mod 3
logo 3| f(n) para todo n natural
então D = 30.

Saudações,
PJMS



Em ter., 17 de mar. de 2020 às 11:57, Israel Meireles Chrisostomo <
israelmchrisost...@gmail.com> escreveu:

> Sim é isso q eu quis dizer
>
> Em ter, 17 de mar de 2020 11:12, Carlos Gustavo Tamm de Araujo Moreira <
> g...@impa.br> escreveu:
>
>> Acho que a pergunta deve ser qual é o maior inteiro positivo que divide
>> essa expressão para todo valor de n ao mesmo tempo.
>>
>> On Tue, Mar 17, 2020 at 6:58 AM Pedro José  wrote:
>>
>>> Bom dia!
>>> Se você considerar a expressão n(427-90n-70n^2+45n^3+18n^4)
>>> D=|n(427-90n-70n^2+45n^3+18n^4)|
>>> Por exemplo, n=1
>>> D=330.
>>> Agora se liberar n para variar D tende a oo.
>>>
>>> Se n for raiz da expressão, também tende a oi, pois qualquer inteiro
>>> divide 0.
>>>
>>>
>>> Em seg, 16 de mar de 2020 22:16, Israel Meireles Chrisostomo <
>>> israelmchrisost...@gmail.com> escreveu:
>>>
 não entendi

 Em seg., 16 de mar. de 2020 às 22:01, Pedro José 
 escreveu:

> Para um dado n é o módulo do valor da expressão.
>
> Em seg, 16 de mar de 2020 21:49, Pedro José 
> escreveu:
>
>> Boa noite!
>> O módulo dessa expressão tende a oo. Não existe máximo.
>> Saudações,
>> PJMS
>>
>> Em seg, 16 de mar de 2020 20:36, Israel Meireles Chrisostomo <
>> israelmchrisost...@gmail.com> escreveu:
>>
>>> Qual é o maior inteiro que divide  n (427 - 90n - 70n^2 + 45n^3 +
>>> 18n^4)?
>>>
>>> --
>>> Israel Meireles Chrisostomo
>>>
>>> --
>>> Esta mensagem foi verificada pelo sistema de antivírus e
>>> acredita-se estar livre de perigo.
>>
>>
> --
> Esta mensagem foi verificada pelo sistema de antivírus e
> acredita-se estar livre de perigo.



 --
 Israel Meireles Chrisostomo

 --
 Esta mensagem foi verificada pelo sistema de antivírus e
 acredita-se estar livre de perigo.
>>>
>>>
>>> --
>>> Esta mensagem foi verificada pelo sistema de antivírus e
>>> acredita-se estar livre de perigo.
>>
>>
>> --
>> Esta mensagem foi verificada pelo sistema de antivírus e
>> acredita-se estar livre de perigo.
>
>
> --
> Esta mensagem foi verificada pelo sistema de antivírus e
> acredita-se estar livre de perigo.

-- 
Esta mensagem foi verificada pelo sistema de antiv�rus e
 acredita-se estar livre de perigo.



[obm-l] Re: [obm-l] Re: [obm-l] Re: [obm-l] Re: [obm-l] Re: [obm-l] Teoria dos números

2020-03-17 Por tôpico Israel Meireles Chrisostomo
Sim é isso q eu quis dizer

Em ter, 17 de mar de 2020 11:12, Carlos Gustavo Tamm de Araujo Moreira <
g...@impa.br> escreveu:

> Acho que a pergunta deve ser qual é o maior inteiro positivo que divide
> essa expressão para todo valor de n ao mesmo tempo.
>
> On Tue, Mar 17, 2020 at 6:58 AM Pedro José  wrote:
>
>> Bom dia!
>> Se você considerar a expressão n(427-90n-70n^2+45n^3+18n^4)
>> D=|n(427-90n-70n^2+45n^3+18n^4)|
>> Por exemplo, n=1
>> D=330.
>> Agora se liberar n para variar D tende a oo.
>>
>> Se n for raiz da expressão, também tende a oi, pois qualquer inteiro
>> divide 0.
>>
>>
>> Em seg, 16 de mar de 2020 22:16, Israel Meireles Chrisostomo <
>> israelmchrisost...@gmail.com> escreveu:
>>
>>> não entendi
>>>
>>> Em seg., 16 de mar. de 2020 às 22:01, Pedro José 
>>> escreveu:
>>>
 Para um dado n é o módulo do valor da expressão.

 Em seg, 16 de mar de 2020 21:49, Pedro José 
 escreveu:

> Boa noite!
> O módulo dessa expressão tende a oo. Não existe máximo.
> Saudações,
> PJMS
>
> Em seg, 16 de mar de 2020 20:36, Israel Meireles Chrisostomo <
> israelmchrisost...@gmail.com> escreveu:
>
>> Qual é o maior inteiro que divide  n (427 - 90n - 70n^2 + 45n^3 +
>> 18n^4)?
>>
>> --
>> Israel Meireles Chrisostomo
>>
>> --
>> Esta mensagem foi verificada pelo sistema de antivírus e
>> acredita-se estar livre de perigo.
>
>
 --
 Esta mensagem foi verificada pelo sistema de antivírus e
 acredita-se estar livre de perigo.
>>>
>>>
>>>
>>> --
>>> Israel Meireles Chrisostomo
>>>
>>> --
>>> Esta mensagem foi verificada pelo sistema de antivírus e
>>> acredita-se estar livre de perigo.
>>
>>
>> --
>> Esta mensagem foi verificada pelo sistema de antivírus e
>> acredita-se estar livre de perigo.
>
>
> --
> Esta mensagem foi verificada pelo sistema de antivírus e
> acredita-se estar livre de perigo.

-- 
Esta mensagem foi verificada pelo sistema de antiv�rus e
 acredita-se estar livre de perigo.



[obm-l] Re: [obm-l] Re: [obm-l] Re: [obm-l] Re: [obm-l] Teoria dos números

2020-03-17 Por tôpico Carlos Gustavo Tamm de Araujo Moreira
Acho que a pergunta deve ser qual é o maior inteiro positivo que divide
essa expressão para todo valor de n ao mesmo tempo.

On Tue, Mar 17, 2020 at 6:58 AM Pedro José  wrote:

> Bom dia!
> Se você considerar a expressão n(427-90n-70n^2+45n^3+18n^4)
> D=|n(427-90n-70n^2+45n^3+18n^4)|
> Por exemplo, n=1
> D=330.
> Agora se liberar n para variar D tende a oo.
>
> Se n for raiz da expressão, também tende a oi, pois qualquer inteiro
> divide 0.
>
>
> Em seg, 16 de mar de 2020 22:16, Israel Meireles Chrisostomo <
> israelmchrisost...@gmail.com> escreveu:
>
>> não entendi
>>
>> Em seg., 16 de mar. de 2020 às 22:01, Pedro José 
>> escreveu:
>>
>>> Para um dado n é o módulo do valor da expressão.
>>>
>>> Em seg, 16 de mar de 2020 21:49, Pedro José 
>>> escreveu:
>>>
 Boa noite!
 O módulo dessa expressão tende a oo. Não existe máximo.
 Saudações,
 PJMS

 Em seg, 16 de mar de 2020 20:36, Israel Meireles Chrisostomo <
 israelmchrisost...@gmail.com> escreveu:

> Qual é o maior inteiro que divide  n (427 - 90n - 70n^2 + 45n^3 +
> 18n^4)?
>
> --
> Israel Meireles Chrisostomo
>
> --
> Esta mensagem foi verificada pelo sistema de antivírus e
> acredita-se estar livre de perigo.


>>> --
>>> Esta mensagem foi verificada pelo sistema de antivírus e
>>> acredita-se estar livre de perigo.
>>
>>
>>
>> --
>> Israel Meireles Chrisostomo
>>
>> --
>> Esta mensagem foi verificada pelo sistema de antivírus e
>> acredita-se estar livre de perigo.
>
>
> --
> Esta mensagem foi verificada pelo sistema de antivírus e
> acredita-se estar livre de perigo.

-- 
Esta mensagem foi verificada pelo sistema de antiv�rus e
 acredita-se estar livre de perigo.



[obm-l] Re: [obm-l] Re: [obm-l] Re: [obm-l] Re: [obm-l] Cálculo do Volume de um Sólido

2020-02-13 Por tôpico Luiz Antonio Rodrigues
Olá, Ralph!
Tudo bem?
Eu gostei muito da maneira que você indicou na segunda opção de resolução.
Olhamos o plano xy "por cima" e calculamos a integral "empilhando" os
trapézios em relação ao eixo z.
Muito obrigado pela resposta!
Abraços!
Luiz

Em qua, 12 de fev de 2020 2:27 PM, Ralph Teixeira 
escreveu:

> Vamos fixar um z (entre 0 e 2) para desenhar a seção horizontal. Como
> x+y=z^2 e x+y=2z são duas retas paralelas, a seção horizontal é um trapézio
> mais ou menos assim:
>
> |\
> | \
> |  \
> |   \
> |\
>  \\
>   \\
>
> As retas inclinadas são x+y=z^2, e x+y=2z. A reta vertical é o eixo y
> entre z^2 e z, e a horizontal é o eixo x entre z^2 e z.
>
> Então o problema na sua integral é que nem sempre o x vai de z^2-y até
> 2z-y, depende do valor de y! Trace uma vareta horizontal atravessando o
> trapézio:
>
> -- Na parte "de baixo" do trapézio, a vareta fura o trapézio nas duas
> retas inclinadas, ou seja, ali temos o x variando de z^2-y até 2z-y que nem
> você falou. Mas isso é só na parte de baixo, ou seja, apenas quando 0 -- Na parte de cima, a vareta fura na reta vertrical e na inclinada, isto
> é, 0
> Ou seja, para resolver isso com uma integral tripla dxdydz, vai ter que
> dividi-la em duas:
>
> Int (0 a 2) Int (0 a z^2) Int (z^2-y a 2z-y) dx dy dz +
> + Int (0 a 2) Int (z^2 a z) Int (0 a 2z-y) dx dy dz
>
> ---///---
>
> Outra opção (equivalente ao que o Buffara fez, mas subtraindo no plano
> antes de integrar para achar o volume): o trapézio pode ser pensado como a
> diferença de dois triângulos retângulos isósceles com vértice na origem --
> um grande tem cateto 2z, o pequeno z^2. Então a área de cada trapézio é:
>
> [(2z)^2 - (z^2)^2]/2 = 2z^2-z^4/2
>
> que, naturalmente, varia com z. Note que, como era de se esperar, a área
> dá 0 em z=0 e z=2.
>
> Agora é só integrar essa área para 0<=z<=2. Ou seja:
>
> Volume = 2.2^3/3 - 2^5/10 = 16/3 - 16/5 = 32/15.
>
> Abraço, Ralph.
>
>
> On Wed, Feb 12, 2020 at 9:29 AM Pedro José  wrote:
>
>> Bom dia!
>> Alguém poderia me ajudar e mostrar onde errei os limites? Resolvendo por
>> integral tripla, usando f(x,y,z)=1.
>>
>> Grato,
>> PJMS
>>
>> Em ter, 11 de fev de 2020 13:11, Pedro José 
>> escreveu:
>>
>>> Boa tarde!
>>>
>>> Resolvi por método numérico usando, pelo menos penso eu, os mesmos
>>> limites e encontrei 2,1329, muito próximo da resposta. Gostaria que alguém
>>> me ajudasse onde errei na integral tripla.
>>> Usei z^2-y e 2z-y como os limites para integral em dx. Em seguida, z^2 e
>>> 2z para dy e finalmente 0 e 2 para dz.
>>> Onde está o erro?
>>> Grato,
>>> PJMS
>>>
>>> Em ter, 11 de fev de 2020 12:49, Claudio Buffara <
>>> claudio.buff...@gmail.com> escreveu:
>>>
 O sólido é a região do 1o octante (todas as coordenadas positivas)
 compreendida entre os planos x-z e y-z, acima do plano z = (x+y)/2 e abaixo
 da z = raiz(x+y).
 A superfície e o plano se intersectam numa reta:
 raiz(x+y) = (x+y)/2 ==> x+y = (x+y)^2/4 ==> x+y = 4, contida no plano z
 = 2.

 Assim, o volume pode ser dado pela diferença entre duas integrais
 duplas, calculadas sobre o domínio D, no plano x-y, dado por x > 0, y > 0 e
 x+y = 4.
 Volume = Integral(D) raiz(x+y)*dA - Integral(D) (x+y)/2*dA.

 Usando coordenadas cartesianas, a primeira integral fica:
 Integral(x=0...4)Integral(y=0...4-x)*raiz(x+y)*dy*dx
 = Integral(0...4) (2/3)*(4^(3/2) - x^(3/2))*dx
 = Integral(0...4) (16/3 - (2/3)*x^(3/2))
 = 64/3 - (4/15)*4^(5/2)
 = 64/3 - 128/15
 = 64/5

 A segunda integral é:
 Integral(x=0...4)Integral(y=0...4-x) (x+y)/2*dy*dx
 = Integral(x=0...4) (1/2)*(x*(4-x) + (4-x)^2/2)*dx
 = Integral(0...4) (4 - x^2/4)*dx
 = 32/3

 Logo, o volume é 64/5 - 32/3 = 32/15  (se não errei nenhuma conta...)

 []s,
 Claudio.


 On Mon, Feb 3, 2020 at 8:55 PM Luiz Antonio Rodrigues <
 rodrigue...@gmail.com> wrote:

> Olá, pessoal!
> Tudo bem?
> Estou tentando resolver o seguinte problema:
>
> Ache o volume da região tridimensional definida por:
>
> z^2
> Sendo que:
> x>0 e y>0 e z>0
>
> Com o auxílio de um software eu consegui visualizar o sólido em
> questão.
> Eu calculei o volume do sólido girando em torno do eixo z e dividindo
> o resultado por 4.
> A resposta que eu obtive foi (16*pi)/15, que não está correta.
> Já refiz os cálculos muitas vezes e chego sempre na mesma resposta.
> Alguém pode me ajudar?
> Muito obrigado e um abraço!
>
> --
> Esta mensagem foi verificada pelo sistema de antivírus e
> acredita-se estar livre de perigo.


 --
 Esta mensagem foi verificada pelo sistema de antivírus e
 acredita-se estar livre de perigo.
>>>
>>>
>> --
>> Esta mensagem foi verificada pelo sistema de antivírus e
>> acredita-se estar livre de perigo.
>
>
> --
> Esta mensagem foi verificada pelo sistema de antivírus e
> acredita-se estar livre de 

[obm-l] Re: [obm-l] Re: [obm-l] Re: [obm-l] Re: [obm-l] Cálculo do Volume de um Sólido

2020-02-12 Por tôpico Rodrigo Ângelo
Luiz Antonio,

Creio que os livros de Cálculo cubram integrais iteradas. Eu estudei pelo
livro do James Stewart, mas dê uma olhada no livro que você já está
acostumado que deve ter esse conteúdo.

Mas, basicamente, quando você tem algo do tipo

[image: image.png]

Você primeiro integra f(x,y,z) de x0 até x1. O resultado vai ser uma função
g(y,z).
Então integra g(y,z) de y0 até y1, e o resultado vai ser uma h(z).
Então integra h(z) de z0 até z1 e obtém um número.

(ou seja, vai integrando de dentro pra fora)

Outra coisa: esse processo é o mesmo independente da quantidade de
integrais que você tiver: integral dupla, tripla, quadrupla, etc é sempre
de dentro pra fora.


Uma aplicação de integrais iteradas é justamente o cálculo de volume. Se
f(x,y,z) = 1, então a integral iterada em dxdydz vai ser o volume do sólido
definido pelos limites de integração (volume da região de integração).

Um detalhe é que os limites de integração podem ser em função das variáveis
mais externas (no caso da imagem, os limites de x podem depender de y e z,
e os limites de y podem depender de z, mas os limites de z devem ser fixos)


Atenciosamente,
Rodrigo de Castro Ângelo


Em qua., 12 de fev. de 2020 às 12:14, Luiz Antonio Rodrigues <
rodrigue...@gmail.com> escreveu:

> Olá, Claudio!
> Olá, Pedro!
> Tudo bem?
> Muito obrigado pela resposta!
> Eu estava tentando resolver o problema "empilhando" secções do plano xy,
> mas demorei para perceber que eram trapézios.
> Isso não deixa de ser uma forma de integração.
> Vocês podem me indicar um bom material para eu aprender a trabalhar com as
> integrais duplas e triplas?
> Percebi que pelas integrais duplas é bem mais fácil.
> Não tenho muito conhecimento para utilizar as integrais triplas.
> Abraços!
> Luiz
>
>
>
> Em qua, 12 de fev de 2020 9:29 AM, Pedro José 
> escreveu:
>
>> Bom dia!
>> Alguém poderia me ajudar e mostrar onde errei os limites? Resolvendo por
>> integral tripla, usando f(x,y,z)=1.
>>
>> Grato,
>> PJMS
>>
>> Em ter, 11 de fev de 2020 13:11, Pedro José 
>> escreveu:
>>
>>> Boa tarde!
>>>
>>> Resolvi por método numérico usando, pelo menos penso eu, os mesmos
>>> limites e encontrei 2,1329, muito próximo da resposta. Gostaria que alguém
>>> me ajudasse onde errei na integral tripla.
>>> Usei z^2-y e 2z-y como os limites para integral em dx. Em seguida, z^2 e
>>> 2z para dy e finalmente 0 e 2 para dz.
>>> Onde está o erro?
>>> Grato,
>>> PJMS
>>>
>>> Em ter, 11 de fev de 2020 12:49, Claudio Buffara <
>>> claudio.buff...@gmail.com> escreveu:
>>>
 O sólido é a região do 1o octante (todas as coordenadas positivas)
 compreendida entre os planos x-z e y-z, acima do plano z = (x+y)/2 e abaixo
 da z = raiz(x+y).
 A superfície e o plano se intersectam numa reta:
 raiz(x+y) = (x+y)/2 ==> x+y = (x+y)^2/4 ==> x+y = 4, contida no plano z
 = 2.

 Assim, o volume pode ser dado pela diferença entre duas integrais
 duplas, calculadas sobre o domínio D, no plano x-y, dado por x > 0, y > 0 e
 x+y = 4.
 Volume = Integral(D) raiz(x+y)*dA - Integral(D) (x+y)/2*dA.

 Usando coordenadas cartesianas, a primeira integral fica:
 Integral(x=0...4)Integral(y=0...4-x)*raiz(x+y)*dy*dx
 = Integral(0...4) (2/3)*(4^(3/2) - x^(3/2))*dx
 = Integral(0...4) (16/3 - (2/3)*x^(3/2))
 = 64/3 - (4/15)*4^(5/2)
 = 64/3 - 128/15
 = 64/5

 A segunda integral é:
 Integral(x=0...4)Integral(y=0...4-x) (x+y)/2*dy*dx
 = Integral(x=0...4) (1/2)*(x*(4-x) + (4-x)^2/2)*dx
 = Integral(0...4) (4 - x^2/4)*dx
 = 32/3

 Logo, o volume é 64/5 - 32/3 = 32/15  (se não errei nenhuma conta...)

 []s,
 Claudio.


 On Mon, Feb 3, 2020 at 8:55 PM Luiz Antonio Rodrigues <
 rodrigue...@gmail.com> wrote:

> Olá, pessoal!
> Tudo bem?
> Estou tentando resolver o seguinte problema:
>
> Ache o volume da região tridimensional definida por:
>
> z^2
> Sendo que:
> x>0 e y>0 e z>0
>
> Com o auxílio de um software eu consegui visualizar o sólido em
> questão.
> Eu calculei o volume do sólido girando em torno do eixo z e dividindo
> o resultado por 4.
> A resposta que eu obtive foi (16*pi)/15, que não está correta.
> Já refiz os cálculos muitas vezes e chego sempre na mesma resposta.
> Alguém pode me ajudar?
> Muito obrigado e um abraço!
>
> --
> Esta mensagem foi verificada pelo sistema de antivírus e
> acredita-se estar livre de perigo.


 --
 Esta mensagem foi verificada pelo sistema de antivírus e
 acredita-se estar livre de perigo.
>>>
>>>
>> --
>> Esta mensagem foi verificada pelo sistema de antivírus e
>> acredita-se estar livre de perigo.
>
>
> --
> Esta mensagem foi verificada pelo sistema de antivírus e
> acredita-se estar livre de perigo.

-- 
Esta mensagem foi verificada pelo sistema de antiv�rus e
 acredita-se estar livre de perigo.



[obm-l] Re: [obm-l] Re: [obm-l] Re: [obm-l] Re: [obm-l] Re: [obm-l] Re: [obm-l] Re: [obm-l] Análise complexa - mostrar que f é sobrejetora

2020-02-11 Por tôpico Artur Costa Steiner
Em

>
> > Talvez seja uma tradução um tanto infeliz de entire function, do Inglês.
> No Inglês, entire em nada lembra integer.
>
> Em geral, eu chuto que um termo matemático usado antes do século XX
> não vem do inglês; a França e a Alemanha eram os grandes centros
> praticamente até a segunda guerra.
>
> > Mas será que é possível provar o teorema sem invocar Picard?
>
> Boa pergunta.  Será que o resultado é equivalente a Picard?  Acho
> pouco provável, mas talvez valha a pena tentar...
> --
> Bernardo Freitas Paulo da Costa
>
> A minha prova é:


Como a função identicamente nula é a única simultaneamente constante e
> ímpar, f não é constante. Logo, se for polinomial, pelo T. Fundamental da
> Álgebra, f é sobrejetora.
>

Suponhamos  agora que f não seja polinomial. Sendo inteira, se não for
sobrejetora, por Picard em seu conjunto imagem falta precisamente um
complexo w. Como f é ímpar e definida em todo o C, f(0) = 0, de modo que f
assume 0 e, portanto, w <> 0 e -w <> w. Logo, existe z com f(z) = -w. Como
f é ímpar e definida em -z, segue-se que f(-z) = -f(z) = w, contradizendo o
fato de que f não assume w. Logo, f é sobrejetora.

Artur

>
>
>
>

-- 
Esta mensagem foi verificada pelo sistema de antiv�rus e
 acredita-se estar livre de perigo.



[obm-l] Re: [obm-l] Re: [obm-l] Re: [obm-l] Re: [obm-l] Re: [obm-l] Re: [obm-l] Análise complexa - mostrar que f é sobrejetora

2020-02-11 Por tôpico Bernardo Freitas Paulo da Costa
On Mon, Feb 10, 2020 at 10:12 PM Artur Costa Steiner
 wrote:
>
> Em seg, 10 de fev de 2020 21:13, Pedro Angelo  
> escreveu:
>>
>> Aparentemente, errei hehe. Achei engraçada essa explicação: funções
>> holomorfas não-inteiras também têm "série de potências inteiras" em
>> torno de cada ponto. Por que só as inteiras levam o nome?
>
> Acho que inteira é no sentido de global, completa.

De fato.  A primeira evidência vem do próprio Picard: "Nous donnerons,
avec M. Weierstrass, le nom de fonctions entières d'une variable
complexe z aux fonctions uniformes et continues dans toute l'étendue
du plan; ce seront, par suite, des fonctions représentées par une
série, toujours convergente, ordonnée suivant les puissances
croissantes de la variable." (Mémoire sur les fonctions entières,
1880, justamente onde ele demonstra os "teoremas de Picard",
http://www.numdam.org/article/ASENS_1880_2_9__145_0.pdf).  Depois, tem
que ler em alemão alguém falando da história do Weierstrass (não achei
o livro / artigo onde ele usa esta notação pela primeira vez).  Eu
achei o Felix Klein, em
https://books.google.co.uk/books?id=XtunBgAAQBAJ=PA286=PA286=weierstrass+ganze+funktion=bl=E5OhNVM3WW=ACfU3U2ABjdB68sIwWisSpPCLJZf_8KdTQ=en=X=2ahUKEwjYteu1lsnnAhXioVwKHWlDAGYQ6AEwAHoECAcQAQ#v=onepage=weierstrass%20ganze%20funktion=false,
e de fato ele usa a mesma terminologia do Picard: ganzen Ebene (o
plano inteiro) e Potenzreihe (série de potências).

> Talvez seja uma tradução um tanto infeliz de entire function, do Inglês. No 
> Inglês, entire em nada lembra integer.

Em geral, eu chuto que um termo matemático usado antes do século XX
não vem do inglês; a França e a Alemanha eram os grandes centros
praticamente até a segunda guerra.

> Mas será que é possível provar o teorema sem invocar Picard?

Boa pergunta.  Será que o resultado é equivalente a Picard?  Acho
pouco provável, mas talvez valha a pena tentar...
--
Bernardo Freitas Paulo da Costa

-- 
Esta mensagem foi verificada pelo sistema de antiv�rus e
 acredita-se estar livre de perigo.


=
Instru��es para entrar na lista, sair da lista e usar a lista em
http://www.mat.puc-rio.br/~obmlistas/obm-l.html
=


[obm-l] Re: [obm-l] Re: [obm-l] Re: [obm-l] Re: [obm-l] Re: [obm-l] Análise complexa - mostrar que f é sobrejetora

2020-02-10 Por tôpico Artur Costa Steiner
Em seg, 10 de fev de 2020 21:13, Pedro Angelo 
escreveu:

> Aparentemente, errei hehe. Achei engraçada essa explicação: funções
> holomorfas não-inteiras também têm "série de potências inteiras" em
> torno de cada ponto. Por que só as inteiras levam o nome?
>

Acho que inteira é no sentido de global, completa. Talvez seja uma tradução
um tanto infeliz de entire function, do Inglês. No Inglês, entire em nada
lembra integer.

Mas será que é possível provar o teorema sem invocar Picard?

Artur

>
> Le lun. 10 févr. 2020 à 20:52, Bernardo Freitas Paulo da Costa
>  a écrit :
> >
> > On Mon, Feb 10, 2020 at 8:16 PM Artur Costa Steiner
> >  wrote:
> > > O adjetivo inteira, em análise complexa,  não tem nada a ver com o que
> ele sugere. Acho uma terminologia infeliz, mas é consagrada.
> >
> > Um chute: em francês, o termo "série inteira" (por oposição a série
> > fracionária) se refere às séries de potências (inteiras) da variável z
> > (por oposição às "séries de Puiseux" onde há expoentes fracionários).
> > E as funções inteiras têm expansão, convergente, como série de
> > potências (inteiras) da variável z, f(z) = \sum_{n=0}^\infty a_n z^n.
> >
> > Abraços,
> > --
> > Bernardo Freitas Paulo da Costa
> >
> > --
> > Esta mensagem foi verificada pelo sistema de antivírus e
> >  acredita-se estar livre de perigo.
> >
> >
> > =
> > Instru�ões para entrar na lista, sair da lista e usar a lista em
> > http://www.mat.puc-rio.br/~obmlistas/obm-l.html
> > =
>
> --
> Esta mensagem foi verificada pelo sistema de antivírus e
>  acredita-se estar livre de perigo.
>
>
> =
> Instru�ões para entrar na lista, sair da lista e usar a lista em
> http://www.mat.puc-rio.br/~obmlistas/obm-l.html
> =
>

-- 
Esta mensagem foi verificada pelo sistema de antiv�rus e
 acredita-se estar livre de perigo.



[obm-l] Re: [obm-l] Re: [obm-l] Re: [obm-l] Re: [obm-l] Análise complexa - mostrar que f é sobrejetora

2020-02-10 Por tôpico Pedro Angelo
Aparentemente, errei hehe. Achei engraçada essa explicação: funções
holomorfas não-inteiras também têm "série de potências inteiras" em
torno de cada ponto. Por que só as inteiras levam o nome?

Le lun. 10 févr. 2020 à 20:52, Bernardo Freitas Paulo da Costa
 a écrit :
>
> On Mon, Feb 10, 2020 at 8:16 PM Artur Costa Steiner
>  wrote:
> > O adjetivo inteira, em análise complexa,  não tem nada a ver com o que ele 
> > sugere. Acho uma terminologia infeliz, mas é consagrada.
>
> Um chute: em francês, o termo "série inteira" (por oposição a série
> fracionária) se refere às séries de potências (inteiras) da variável z
> (por oposição às "séries de Puiseux" onde há expoentes fracionários).
> E as funções inteiras têm expansão, convergente, como série de
> potências (inteiras) da variável z, f(z) = \sum_{n=0}^\infty a_n z^n.
>
> Abraços,
> --
> Bernardo Freitas Paulo da Costa
>
> --
> Esta mensagem foi verificada pelo sistema de antivírus e
>  acredita-se estar livre de perigo.
>
>
> =
> Instru�ões para entrar na lista, sair da lista e usar a lista em
> http://www.mat.puc-rio.br/~obmlistas/obm-l.html
> =

-- 
Esta mensagem foi verificada pelo sistema de antiv�rus e
 acredita-se estar livre de perigo.


=
Instru��es para entrar na lista, sair da lista e usar a lista em
http://www.mat.puc-rio.br/~obmlistas/obm-l.html
=


[obm-l] Re: [obm-l] Re: [obm-l] Re: [obm-l] Re: [obm-l] Re: [obm-l] Cálculo do Volume de um Sólido

2020-02-10 Por tôpico Luiz Antonio Rodrigues
Olá, Pedro!
Vou pensar na questão novamente e ver se consigo chegar na resposta.
Eu escreverei para dizer se consegui.
Muito obrigado!
Abraços!
Luiz


Em seg, 10 de fev de 2020 7:19 PM, Pedro José 
escreveu:

> Boa noite!
> Não sei onde errei está dando exatamente a metade 16/15.
> Saudações,
> PJMS
>
> Em seg, 10 de fev de 2020 15:46, Luiz Antonio Rodrigues <
> rodrigue...@gmail.com> escreveu:
>
>> Olá, Pedro!
>> Tudo bem?
>> Obrigado pela resposta!
>> A resposta realmente não tem pi: é 32/15.
>> Eu percebi ontem que o meu erro foi fazer uma rotação em torno do eixo z.
>> Se seccionarmos a figura no plano xy teremos um trapézio.
>> Vou pensar na sua sugestão e tentar fazer tudo de novo.
>> Muito obrigado!
>> Abraços!
>> Luiz
>>
>>
>>
>> Em seg, 10 de fev de 2020 1:38 PM, Pedro José 
>> escreveu:
>>
>>> Boa tarde!
>>> Como no caso você tem a resposta, facilitaria se a expusesse.
>>> Para evitar que postemos soluções erradas.
>>>
>>> Saudações,
>>> PJMS
>>>
>>> Em qui., 6 de fev. de 2020 às 07:41, Anderson Torres <
>>> torres.anderson...@gmail.com> escreveu:
>>>
 Em seg., 3 de fev. de 2020 às 20:55, Luiz Antonio Rodrigues
  escreveu:
 >
 > Olá, pessoal!
 > Tudo bem?
 > Estou tentando resolver o seguinte problema:
 >
 > Ache o volume da região tridimensional definida por:
 >
 > z^2>>> >
 > Sendo que:
 > x>0 e y>0 e z>0
 >
 > Com o auxílio de um software eu consegui visualizar o sólido em
 questão.
 > Eu calculei o volume do sólido girando em torno do eixo z e dividindo
 o resultado por 4.
 > A resposta que eu obtive foi (16*pi)/15, que não está correta.
 > Já refiz os cálculos muitas vezes e chego sempre na mesma resposta.
 > Alguém pode me ajudar?

 Tem como cê enviar as contas e o desenho que cê fez?

 > Muito obrigado e um abraço!
 >
 > --
 > Esta mensagem foi verificada pelo sistema de antivírus e
 > acredita-se estar livre de perigo.

 --
 Esta mensagem foi verificada pelo sistema de antivírus e
  acredita-se estar livre de perigo.



 =
 Instru�ões para entrar na lista, sair da lista e usar a lista em
 http://www.mat.puc-rio.br/~obmlistas/obm-l.html

 =

>>>
>>> --
>>> Esta mensagem foi verificada pelo sistema de antivírus e
>>> acredita-se estar livre de perigo.
>>
>>
>> --
>> Esta mensagem foi verificada pelo sistema de antivírus e
>> acredita-se estar livre de perigo.
>
>
> --
> Esta mensagem foi verificada pelo sistema de antivírus e
> acredita-se estar livre de perigo.

-- 
Esta mensagem foi verificada pelo sistema de antiv�rus e
 acredita-se estar livre de perigo.



[obm-l] Re: [obm-l] Re: [obm-l] Re: [obm-l] Re: [obm-l] Cálculo do Volume de um Sólido

2020-02-10 Por tôpico Pedro José
Boa noite!
Não sei onde errei está dando exatamente a metade 16/15.
Saudações,
PJMS

Em seg, 10 de fev de 2020 15:46, Luiz Antonio Rodrigues <
rodrigue...@gmail.com> escreveu:

> Olá, Pedro!
> Tudo bem?
> Obrigado pela resposta!
> A resposta realmente não tem pi: é 32/15.
> Eu percebi ontem que o meu erro foi fazer uma rotação em torno do eixo z.
> Se seccionarmos a figura no plano xy teremos um trapézio.
> Vou pensar na sua sugestão e tentar fazer tudo de novo.
> Muito obrigado!
> Abraços!
> Luiz
>
>
>
> Em seg, 10 de fev de 2020 1:38 PM, Pedro José 
> escreveu:
>
>> Boa tarde!
>> Como no caso você tem a resposta, facilitaria se a expusesse.
>> Para evitar que postemos soluções erradas.
>>
>> Saudações,
>> PJMS
>>
>> Em qui., 6 de fev. de 2020 às 07:41, Anderson Torres <
>> torres.anderson...@gmail.com> escreveu:
>>
>>> Em seg., 3 de fev. de 2020 às 20:55, Luiz Antonio Rodrigues
>>>  escreveu:
>>> >
>>> > Olá, pessoal!
>>> > Tudo bem?
>>> > Estou tentando resolver o seguinte problema:
>>> >
>>> > Ache o volume da região tridimensional definida por:
>>> >
>>> > z^2>> >
>>> > Sendo que:
>>> > x>0 e y>0 e z>0
>>> >
>>> > Com o auxílio de um software eu consegui visualizar o sólido em
>>> questão.
>>> > Eu calculei o volume do sólido girando em torno do eixo z e dividindo
>>> o resultado por 4.
>>> > A resposta que eu obtive foi (16*pi)/15, que não está correta.
>>> > Já refiz os cálculos muitas vezes e chego sempre na mesma resposta.
>>> > Alguém pode me ajudar?
>>>
>>> Tem como cê enviar as contas e o desenho que cê fez?
>>>
>>> > Muito obrigado e um abraço!
>>> >
>>> > --
>>> > Esta mensagem foi verificada pelo sistema de antivírus e
>>> > acredita-se estar livre de perigo.
>>>
>>> --
>>> Esta mensagem foi verificada pelo sistema de antivírus e
>>>  acredita-se estar livre de perigo.
>>>
>>>
>>> =
>>> Instru�ões para entrar na lista, sair da lista e usar a lista em
>>> http://www.mat.puc-rio.br/~obmlistas/obm-l.html
>>> =
>>>
>>
>> --
>> Esta mensagem foi verificada pelo sistema de antivírus e
>> acredita-se estar livre de perigo.
>
>
> --
> Esta mensagem foi verificada pelo sistema de antivírus e
> acredita-se estar livre de perigo.

-- 
Esta mensagem foi verificada pelo sistema de antiv�rus e
 acredita-se estar livre de perigo.



[obm-l] Re: [obm-l] Re: [obm-l] Re: [obm-l] Re: [obm-l] Re: [obm-l] Re: [obm-l] Questão OBM - U

2020-01-23 Por tôpico Esdras Muniz
É fácil ver que esse ínfimo tem que ser no mínimo 4, basta fazer
desigualdade triângulos com os triângulos que têm dois vértices comuns com
o quadrilátero e o terceiro sendo a interseção das diagonais. E por esse
argumento do Caio, vemos que é 4 mesmo.

Em qui, 23 de jan de 2020 08:59, Caio Costa  escreveu:

> Minimiza-se a soma das diagonais ao tomar-se um losango degenerado, com
> uma diagonal valendo 4 e outra valendo 0.
>
> Em qui, 23 de jan de 2020 08:34, gilberto azevedo 
> escreveu:
>
>> Pensei em minimizar √(a² + (4-a)²)
>> 4 - a, devido ao fato do perímetro ser 8.
>> No caso obtenho o mínimo sendo 2√2, quando o retângulo é um quadrado de
>> lado 2.
>> A soma das diagonais seria no caso 4√2, e não bate com o gabarito.
>>
>> Em qui, 23 de jan de 2020 08:20, Bernardo Freitas Paulo da Costa <
>> bernardo...@gmail.com> escreveu:
>>
>>> On Thu, Jan 23, 2020 at 7:24 AM gilberto azevedo 
>>> wrote:
>>> >> On Sat, Jan 11, 2020 at 11:24 AM gilberto azevedo <
>>> gil159...@gmail.com> wrote:
>>> >> >
>>> >> > Qual o ínfimo sobre todos os quadriláteros convexos com
>>> perímetro 8 da soma dos comprimentos de suas diagonais ?
>>> >
>>> > Tentei com o retângulo e o quadrado, porém não obtive a resposta...  O
>>> gabarito é 4.
>>>
>>> Qual (ou quais?) retângulo(s) você testou??  Que resposta você obteve?
>>> --
>>> Bernardo Freitas Paulo da Costa
>>>
>>> --
>>> Esta mensagem foi verificada pelo sistema de antivírus e
>>>  acredita-se estar livre de perigo.
>>>
>>>
>>> =
>>> Instru�ões para entrar na lista, sair da lista e usar a lista em
>>> http://www.mat.puc-rio.br/~obmlistas/obm-l.html
>>> =
>>>
>>
>> --
>> Esta mensagem foi verificada pelo sistema de antivírus e
>> acredita-se estar livre de perigo.
>
>
> --
> Esta mensagem foi verificada pelo sistema de antivírus e
> acredita-se estar livre de perigo.

-- 
Esta mensagem foi verificada pelo sistema de antiv�rus e
 acredita-se estar livre de perigo.



[obm-l] Re: [obm-l] Re: [obm-l] Re: [obm-l] Re: [obm-l] Re: [obm-l] Questão OBM - U

2020-01-23 Por tôpico Caio Costa
Minimiza-se a soma das diagonais ao tomar-se um losango degenerado, com uma
diagonal valendo 4 e outra valendo 0.

Em qui, 23 de jan de 2020 08:34, gilberto azevedo 
escreveu:

> Pensei em minimizar √(a² + (4-a)²)
> 4 - a, devido ao fato do perímetro ser 8.
> No caso obtenho o mínimo sendo 2√2, quando o retângulo é um quadrado de
> lado 2.
> A soma das diagonais seria no caso 4√2, e não bate com o gabarito.
>
> Em qui, 23 de jan de 2020 08:20, Bernardo Freitas Paulo da Costa <
> bernardo...@gmail.com> escreveu:
>
>> On Thu, Jan 23, 2020 at 7:24 AM gilberto azevedo 
>> wrote:
>> >> On Sat, Jan 11, 2020 at 11:24 AM gilberto azevedo 
>> wrote:
>> >> >
>> >> > Qual o ínfimo sobre todos os quadriláteros convexos com perímetro
>> 8 da soma dos comprimentos de suas diagonais ?
>> >
>> > Tentei com o retângulo e o quadrado, porém não obtive a resposta...  O
>> gabarito é 4.
>>
>> Qual (ou quais?) retângulo(s) você testou??  Que resposta você obteve?
>> --
>> Bernardo Freitas Paulo da Costa
>>
>> --
>> Esta mensagem foi verificada pelo sistema de antivírus e
>>  acredita-se estar livre de perigo.
>>
>>
>> =
>> Instru�ões para entrar na lista, sair da lista e usar a lista em
>> http://www.mat.puc-rio.br/~obmlistas/obm-l.html
>> =
>>
>
> --
> Esta mensagem foi verificada pelo sistema de antivírus e
> acredita-se estar livre de perigo.

-- 
Esta mensagem foi verificada pelo sistema de antiv�rus e
 acredita-se estar livre de perigo.



[obm-l] Re: [obm-l] Re: [obm-l] Re: [obm-l] Re: [obm-l] Questão OBM - U

2020-01-23 Por tôpico gilberto azevedo
Pensei em minimizar √(a² + (4-a)²)
4 - a, devido ao fato do perímetro ser 8.
No caso obtenho o mínimo sendo 2√2, quando o retângulo é um quadrado de
lado 2.
A soma das diagonais seria no caso 4√2, e não bate com o gabarito.

Em qui, 23 de jan de 2020 08:20, Bernardo Freitas Paulo da Costa <
bernardo...@gmail.com> escreveu:

> On Thu, Jan 23, 2020 at 7:24 AM gilberto azevedo 
> wrote:
> >> On Sat, Jan 11, 2020 at 11:24 AM gilberto azevedo 
> wrote:
> >> >
> >> > Qual o ínfimo sobre todos os quadriláteros convexos com perímetro
> 8 da soma dos comprimentos de suas diagonais ?
> >
> > Tentei com o retângulo e o quadrado, porém não obtive a resposta...  O
> gabarito é 4.
>
> Qual (ou quais?) retângulo(s) você testou??  Que resposta você obteve?
> --
> Bernardo Freitas Paulo da Costa
>
> --
> Esta mensagem foi verificada pelo sistema de antivírus e
>  acredita-se estar livre de perigo.
>
>
> =
> Instru�ões para entrar na lista, sair da lista e usar a lista em
> http://www.mat.puc-rio.br/~obmlistas/obm-l.html
> =
>

-- 
Esta mensagem foi verificada pelo sistema de antiv�rus e
 acredita-se estar livre de perigo.



[obm-l] Re: [obm-l] Re: [obm-l] Re: [obm-l] Re: [obm-l] Função Desconhecida

2020-01-15 Por tôpico Luiz Antonio Rodrigues
Olá, Anderson!
Bom dia!
Visitei o site que você indicou.
É muito bom!
Muito obrigado!
Abs

Em qua, 15 de jan de 2020 8:11 AM, Anderson Torres <
torres.anderson...@gmail.com> escreveu:

> Em sex., 20 de dez. de 2019 às 18:24, Luiz Antonio Rodrigues
>  escreveu:
> >
> > Olá, Esdras!
> > Eu de novo!
> > Você, ou alguém do grupo, pode me indicar um bom material relacionado às
> funções transcendentes?
> > É um assunto que me interessa bastante!
> > Abraços!
> > Luiz
> >
> > Em sex, 20 de dez de 2019 4:38 PM, Esdras Muniz <
> esdrasmunizm...@gmail.com> escreveu:
> >>
> >> Acho que essa função é trancendente.
>
> Pelo que eu sei, existe um algoritmo (sim, um programa de computador)
> que verifica se uma funçao é ou não passível de "integração
> bonitinha".
>
> Sempre que a dúvida bater, use esse site:
>
> https://www.integral-calculator.com/
>
> >>
> >> Em sex, 20 de dez de 2019 14:42, Luiz Antonio Rodrigues <
> rodrigue...@gmail.com> escreveu:
> >>>
> >>> Olá, pessoal!
> >>> Tudo bem?
> >>> Estou tentando, há alguns dias, resolver o seguinte problema:
> >>>
> >>> Preciso descobrir uma função f(x) cuja derivada é sen(x^3). Sabe-se
> que f(0)=2.
> >>>
> >>> Utilizei um software e mesmo assim não cheguei numa resposta para esta
> integral...
> >>> Alguém sabe se esta função é de algum tipo "especial"?
> >>> Muito obrigado!
> >>> Luiz
> >>>
> >>> --
> >>> Esta mensagem foi verificada pelo sistema de antivírus e
> >>> acredita-se estar livre de perigo.
> >>
> >>
> >> --
> >> Esta mensagem foi verificada pelo sistema de antivírus e
> >> acredita-se estar livre de perigo.
> >
> >
> > --
> > Esta mensagem foi verificada pelo sistema de antivírus e
> > acredita-se estar livre de perigo.
>
> --
> Esta mensagem foi verificada pelo sistema de antivírus e
>  acredita-se estar livre de perigo.
>
>
> =
> Instru�ões para entrar na lista, sair da lista e usar a lista em
> http://www.mat.puc-rio.br/~obmlistas/obm-l.html
> =
>

-- 
Esta mensagem foi verificada pelo sistema de antiv�rus e
 acredita-se estar livre de perigo.



[obm-l] Re: [obm-l] Re: [obm-l] Re: [obm-l] Re: [obm-l] Re: [obm-l] Re: [obm-l] ordem lexicográfica dos numerais

2019-11-12 Por tôpico lumpa lumpa
Sim, mas naquele problema eu ERRONEAMENTE falei em ordem lexicográfica, mas
quando descrevi a sequencia postei outra ordem em que
as sequencias de menor quantidade de letras sempre precedem qualquer outra
cuja quantidade de letras é maior, por isso ao invés de fazer assim:
a, aa, ac, ae, ai, am, at, c, ca, cc, ce, ci, cm, ct, e, ea, ec, ee, ei,
em, et, i, ia, ic, ie, ii, im, it, m, ma, mc, me, mi, mm, mt, t, ta, tc,
te, ti, tm ,tt

fiz, equivocadamente, assim:


a, c, e, i, m, t, aa, ac, ae, ai, am, at, ca, cc, ce, ci, cm, ct, ea, ec,
ee, ei, em, et, ia, ic, ie, ii, im, it, ma, mc, me, mi, mt, ta, tc, te. ti
,tm, tt, aaa, aac, aae, aai, aam, aat, etc































Em ter., 12 de nov. de 2019 às 21:45, Pedro José 
escreveu:

> É curioso, pois, no problema que você postou com letras às vinha depois de
> t.
>
> Saudações,
> PJMS
>
> Em ter, 12 de nov de 2019 21:22, lumpa lumpa <1vp4l...@gmail.com>
> escreveu:
>
>>
>>
>>
>>
>>
>>
>>
>>
>> Boa noite !
>>
>> Não. 01 vem depois de 00 que é o sucessor de 0, assim:
>>
>> 0, 00, 01, 02, 03, 04, 05, 06, 07, 08, 09, 1, 10, 11, 12, 13, 14, 15, 16,
>> 17, 18, 19, 2, 20, 21, ... etc.
>>
>> É óbvio que a sequencia acima mostra apenas as combinações de no máximo
>> dois algarismos, mas sabemos que há outros infinitos termos entre eles.
>> Mostrando as combinações de três algarismos, seria assim:
>>
>> 0, 00, 000, 001, 002, 003, 004, 00, 006, 007, 008, 009, 01, 010, 011,
>> 012, 013, 014, 01, 016, 017, 018, 019, 02, ., 03, 04,
>> ,,..., 09, 090, 091, 092, 093, 094, 095, 096, 097, 098, 099, 1
>>
>>
>>
>>
>>
>>
>>
>>
>>
>>
>>
>>
>>
>>
>>
>>
>>
>>
>>
>> Em ter., 12 de nov. de 2019 às 19:49, Pedro José 
>> escreveu:
>>
>>> Boa noite!
>>> Mas 1 ocorre antes de 01, não. Tenho que esgotar primeiro as de uma
>>> posição, para depois usar as de duas se não eu não andaria nunca. 0, 00,
>>> 000,  
>>> Só confirme que penso uma solução, caso consiga.
>>>
>>> Saudações,
>>> PJMS
>>>
>>> Em ter., 12 de nov. de 2019 às 18:15, lumpa lumpa <1vp4l...@gmail.com>
>>> escreveu:
>>>
 Boa tarde, Pedro.

 Por menor posição, estou considerando aquela que conta a partir de zero
 todas as sequencias de no máximo quatro algarismos até  2019.

 0, 00, 000,  são todos sequencias diferentes. Pense nos algarismos
 como símbolos quaisquer, como se fossem letras e as combinações palavras na
 ordem alfabética

 [0], 00, 01, 02, 03, 04, 05, 06, 07, 08, 09, [1], 10, 11, 12, 13, 14,
 15, 16, 17, 18, 19, [2], 20, 21, 22, 23 etc

 A linha acima é só um pequeno exemplo dos primeiros termos da sequencia
 com termos de no máximo dois algarismos..

 Em ter., 12 de nov. de 2019 às 17:45, Pedro José 
 escreveu:

> Boa tarde!
> Vai depender do conceito!
> 0 e 00 são contados como um só ou duas vezes?
> Não entendi a menor posição. No meu entender há uma bijeção entre a
> posição e o número.
> A menos, que se contem 02019 e 2019 como o mesmo número, porém como
> "palavras diferentes.
>
> Saudações,
> PJMS
>
>
> Em ter., 12 de nov. de 2019 às 15:31, lumpa lumpa <1vp4l...@gmail.com>
> escreveu:
>
>> Qual a menor posição do número 2019 na ordem lexicográfica de todas
>> as sequências possíveis dos algarismos indo-arábicos: {0, 1, 2, 3, 4, 5, 
>> 6,
>> 7, 8, 9} ?
>>
>> --
>> Esta mensagem foi verificada pelo sistema de antivírus e
>> acredita-se estar livre de perigo.
>
>
> --
> Esta mensagem foi verificada pelo sistema de antivírus e
> acredita-se estar livre de perigo.


 --
 Esta mensagem foi verificada pelo sistema de antivírus e
 acredita-se estar livre de perigo.
>>>
>>>
>>> --
>>> Esta mensagem foi verificada pelo sistema de antivírus e
>>> acredita-se estar livre de perigo.
>>
>>
>> --
>> Esta mensagem foi verificada pelo sistema de antivírus e
>> acredita-se estar livre de perigo.
>
>
> --
> Esta mensagem foi verificada pelo sistema de antivírus e
> acredita-se estar livre de perigo.

-- 
Esta mensagem foi verificada pelo sistema de antiv�rus e
 acredita-se estar livre de perigo.



[obm-l] Re: [obm-l] Re: [obm-l] Re: [obm-l] Re: [obm-l] Re: [obm-l] ordem lexicográfica dos numerais

2019-11-12 Por tôpico Pedro José
É curioso, pois, no problema que você postou com letras às vinha depois de
t.

Saudações,
PJMS

Em ter, 12 de nov de 2019 21:22, lumpa lumpa <1vp4l...@gmail.com> escreveu:

>
>
>
>
>
>
>
>
> Boa noite !
>
> Não. 01 vem depois de 00 que é o sucessor de 0, assim:
>
> 0, 00, 01, 02, 03, 04, 05, 06, 07, 08, 09, 1, 10, 11, 12, 13, 14, 15, 16,
> 17, 18, 19, 2, 20, 21, ... etc.
>
> É óbvio que a sequencia acima mostra apenas as combinações de no máximo
> dois algarismos, mas sabemos que há outros infinitos termos entre eles.
> Mostrando as combinações de três algarismos, seria assim:
>
> 0, 00, 000, 001, 002, 003, 004, 00, 006, 007, 008, 009, 01, 010, 011, 012,
> 013, 014, 01, 016, 017, 018, 019, 02, ., 03, 04, ,,..., 09,
> 090, 091, 092, 093, 094, 095, 096, 097, 098, 099, 1
>
>
>
>
>
>
>
>
>
>
>
>
>
>
>
>
>
>
>
> Em ter., 12 de nov. de 2019 às 19:49, Pedro José 
> escreveu:
>
>> Boa noite!
>> Mas 1 ocorre antes de 01, não. Tenho que esgotar primeiro as de uma
>> posição, para depois usar as de duas se não eu não andaria nunca. 0, 00,
>> 000,  
>> Só confirme que penso uma solução, caso consiga.
>>
>> Saudações,
>> PJMS
>>
>> Em ter., 12 de nov. de 2019 às 18:15, lumpa lumpa <1vp4l...@gmail.com>
>> escreveu:
>>
>>> Boa tarde, Pedro.
>>>
>>> Por menor posição, estou considerando aquela que conta a partir de zero
>>> todas as sequencias de no máximo quatro algarismos até  2019.
>>>
>>> 0, 00, 000,  são todos sequencias diferentes. Pense nos algarismos
>>> como símbolos quaisquer, como se fossem letras e as combinações palavras na
>>> ordem alfabética
>>>
>>> [0], 00, 01, 02, 03, 04, 05, 06, 07, 08, 09, [1], 10, 11, 12, 13, 14,
>>> 15, 16, 17, 18, 19, [2], 20, 21, 22, 23 etc
>>>
>>> A linha acima é só um pequeno exemplo dos primeiros termos da sequencia
>>> com termos de no máximo dois algarismos..
>>>
>>> Em ter., 12 de nov. de 2019 às 17:45, Pedro José 
>>> escreveu:
>>>
 Boa tarde!
 Vai depender do conceito!
 0 e 00 são contados como um só ou duas vezes?
 Não entendi a menor posição. No meu entender há uma bijeção entre a
 posição e o número.
 A menos, que se contem 02019 e 2019 como o mesmo número, porém como
 "palavras diferentes.

 Saudações,
 PJMS


 Em ter., 12 de nov. de 2019 às 15:31, lumpa lumpa <1vp4l...@gmail.com>
 escreveu:

> Qual a menor posição do número 2019 na ordem lexicográfica de todas as
> sequências possíveis dos algarismos indo-arábicos: {0, 1, 2, 3, 4, 5, 6, 
> 7,
> 8, 9} ?
>
> --
> Esta mensagem foi verificada pelo sistema de antivírus e
> acredita-se estar livre de perigo.


 --
 Esta mensagem foi verificada pelo sistema de antivírus e
 acredita-se estar livre de perigo.
>>>
>>>
>>> --
>>> Esta mensagem foi verificada pelo sistema de antivírus e
>>> acredita-se estar livre de perigo.
>>
>>
>> --
>> Esta mensagem foi verificada pelo sistema de antivírus e
>> acredita-se estar livre de perigo.
>
>
> --
> Esta mensagem foi verificada pelo sistema de antivírus e
> acredita-se estar livre de perigo.

-- 
Esta mensagem foi verificada pelo sistema de antiv�rus e
 acredita-se estar livre de perigo.



[obm-l] Re: [obm-l] Re: [obm-l] Re: [obm-l] Re: [obm-l] ordem lexicográfica dos numerais

2019-11-12 Por tôpico lumpa lumpa
Boa noite !

Não. 01 vem depois de 00 que é o sucessor de 0, assim:

0, 00, 01, 02, 03, 04, 05, 06, 07, 08, 09, 1, 10, 11, 12, 13, 14, 15, 16,
17, 18, 19, 2, 20, 21, ... etc.

É óbvio que a sequencia acima mostra apenas as combinações de no máximo
dois algarismos, mas sabemos que há outros infinitos termos entre eles.
Mostrando as combinações de três algarismos, seria assim:

0, 00, 000, 001, 002, 003, 004, 00, 006, 007, 008, 009, 01, 010, 011, 012,
013, 014, 01, 016, 017, 018, 019, 02, ., 03, 04, ,,..., 09,
090, 091, 092, 093, 094, 095, 096, 097, 098, 099, 1



















Em ter., 12 de nov. de 2019 às 19:49, Pedro José 
escreveu:

> Boa noite!
> Mas 1 ocorre antes de 01, não. Tenho que esgotar primeiro as de uma
> posição, para depois usar as de duas se não eu não andaria nunca. 0, 00,
> 000,  
> Só confirme que penso uma solução, caso consiga.
>
> Saudações,
> PJMS
>
> Em ter., 12 de nov. de 2019 às 18:15, lumpa lumpa <1vp4l...@gmail.com>
> escreveu:
>
>> Boa tarde, Pedro.
>>
>> Por menor posição, estou considerando aquela que conta a partir de zero
>> todas as sequencias de no máximo quatro algarismos até  2019.
>>
>> 0, 00, 000,  são todos sequencias diferentes. Pense nos algarismos
>> como símbolos quaisquer, como se fossem letras e as combinações palavras na
>> ordem alfabética
>>
>> [0], 00, 01, 02, 03, 04, 05, 06, 07, 08, 09, [1], 10, 11, 12, 13, 14, 15,
>> 16, 17, 18, 19, [2], 20, 21, 22, 23 etc
>>
>> A linha acima é só um pequeno exemplo dos primeiros termos da sequencia
>> com termos de no máximo dois algarismos..
>>
>> Em ter., 12 de nov. de 2019 às 17:45, Pedro José 
>> escreveu:
>>
>>> Boa tarde!
>>> Vai depender do conceito!
>>> 0 e 00 são contados como um só ou duas vezes?
>>> Não entendi a menor posição. No meu entender há uma bijeção entre a
>>> posição e o número.
>>> A menos, que se contem 02019 e 2019 como o mesmo número, porém como
>>> "palavras diferentes.
>>>
>>> Saudações,
>>> PJMS
>>>
>>>
>>> Em ter., 12 de nov. de 2019 às 15:31, lumpa lumpa <1vp4l...@gmail.com>
>>> escreveu:
>>>
 Qual a menor posição do número 2019 na ordem lexicográfica de todas as
 sequências possíveis dos algarismos indo-arábicos: {0, 1, 2, 3, 4, 5, 6, 7,
 8, 9} ?

 --
 Esta mensagem foi verificada pelo sistema de antivírus e
 acredita-se estar livre de perigo.
>>>
>>>
>>> --
>>> Esta mensagem foi verificada pelo sistema de antivírus e
>>> acredita-se estar livre de perigo.
>>
>>
>> --
>> Esta mensagem foi verificada pelo sistema de antivírus e
>> acredita-se estar livre de perigo.
>
>
> --
> Esta mensagem foi verificada pelo sistema de antivírus e
> acredita-se estar livre de perigo.

-- 
Esta mensagem foi verificada pelo sistema de antiv�rus e
 acredita-se estar livre de perigo.



[obm-l] Re: [obm-l] Re: [obm-l] Re: [obm-l] Re: [obm-l] Ordem Lexicográfica

2019-11-11 Por tôpico Pedro José
Boa tarde!

Errata
3o passo  (4.052.405.310)base6+ (1.111.111.111)base6= (5.203.520.421)base6
e não 3o passo  (4.052.405.31)base6+ (1.111.111.111)base6=
(5.203.520.421)base6

Então você agora, pode tanto compor uma palavra de ordem n, quanto
descobrir qual ordem de uma dada palavra e não. Então você agora, pode
tanto compor uma palavra de ordem n, quanto descobrir qual a palavra de
ordem n.

Saudações,
PJMS

Em seg., 11 de nov. de 2019 às 11:51, Pedro José 
escreveu:

> Bom dia!
>
> É muita coincidência. Teve um problema agora a respeito de numeração na
> terra do IMPA que é muito, mas muito semelhante a esse.
> Só que nesse caso caso é o contrário, ou seja a função inversa. O da terra
> dos Impa, dá uma posição e quer saber qual o número IMPA.
> Aqui se dá uma palavra e se quer a posição.
> Podemos fazer uma bijeção: {a,e,c,i,m,t} em {1,2,3,4,5,6}  se x
> f(x) Usando-se o conceito que já mencionara no problema referido.
> Para saber, no caso a palavra de3 ordem n.
> Passo 1. Escrever o número n na base 6.
> Passo 2. Subtrair na base 6 (n)base 6 - (...111) base6 o maior número
> somente com algarismos 1 e menor que n. Nota considerar da esquerda para
> direita, tantos algarismos quanto o do número .,
> Passo 3 Substituir cada algarismo do resultado da conta anterior por esse
> algarismo acrescido de 1.
> Passo 4. Usar a função inversa da bijeção e tornar os algarismos em letras.
>
> Por exemplo vamos pegar a palavra de ordem imediatamente inferior a
> MATEMATICA, como a resposta foi dada 53.929.309.
> A palavra seria de ordem: 53.929.308
> 1o passo. 53.929.30= (5203520420)base6
> 2o passo (5.203.520.420)base6 - (1.111.111.111)base6= (4.052.405.305)
> 3o passo (5.163.516416)
> 4o passo MATEMATIAT, que realmente é a palavra imediatamente anterior a
> MATEMATICA.
> Então já temos como ir da ordem para a palavra. Agora é só determinar a
> inversa.
> 1o Passo Usar a bijeção e transformar a palavra em Número
> 2o Passo Substituir cada algarismo por o algarismo que o antecede.
> 3o passo Somar (.111)Base6 com tantos algarismos obtidos no passo
> anterior ao obtido no passo anterior, considerando-o como base 6.
> 4o passar transformar o valor obtido da base 6 para pase 10.
>
> Aplicando para MATEMATICA
>
> 1o passo: 5.163.516.421
> 2opasso 4.052.405.310
> 3o passo  (4.052.405.31)base6+ (1.111.111.111)base6= (5.203.520.421)base6
> 4o passo 5*6^9+2*6^8+3*6^6+5*6^5+2*6^4+4*6^2+2*6+1 =53.929.309
>
> Se você quiser entender o porquê, veja nota Minha solução para o item c)
> do problema 3 da prova da OBM-2017 lá eu faço uma abordagem, pegando
> carona na ideia do Cauã, e aqui uma adaptação para a base 6.
>
> Então você agora, pode tanto compor uma palavra de ordem n, quanto
> descobrir qual a palavra de ordem n.
>
> Deixo claro, que me apoiei na ideia do Cauã. A ideia de levar para a base
> 5 no caso dos Impa foi dele. Infelizmente ele não deu um prosseguimento
> satisfatório.
>
> Mas a ideia foi muito boa e serviu até para esse novo problema. Uma grata
> coincidência. Pois a abordagem anterior foi, digamos, um pouco vilipendiada.
> Talvez agora, com mais uma aplicação receba mais atenção e méritos ao Cauã.
>
> Saudações,
> PJMS
>
>
>
>
>
> Em dom., 10 de nov. de 2019 às 21:22, lumpa lumpa <1vp4l...@gmail.com>
> escreveu:
>
>> Boa solução, Rodrigo, mas pq sua contagem resultou um a menos ?
>> A resposta é 53.929.309
>>
>> On Sun, Nov 10, 2019 at 9:31 PM Rodrigo Ângelo 
>> wrote:
>>
>>> A primeira palavra com 10 letras tem posição δ = 6 + 6^2 + ... + 6^9 =
>>> 12.093.234.
>>>
>>> Das palavras que tem 10 letras, fixando a primeira letra em "a", temos
>>> 6^9 palavras. Depois, fixando a primeira letra em "c", "e", e "i", temos a
>>> mesma quantidade.
>>>
>>> Então, das palavras de 10 letras, a primeira que começa com "m" tem
>>> posição
>>> δ + 4*6^9
>>>
>>> Repetindo esse processo para todas as letras que formam a palavra
>>> desejada, temos:
>>>
>>> δ + 4*6^9 + 5*6^7 + 2*6^6 + 4*6^5 + 5*6^3 + 3*6^2 + 6 = 53.929.308.
>>>
>>> Atenciosamente,
>>> Rodrigo Angelo
>>>
>>> On Sun, Nov 10, 2019, 18:42 jamil silva  wrote:
>>>
 Será que sai por base seis ?

 Em dom., 10 de nov. de 2019 às 18:31, lumpa lumpa <1vp4l...@gmail.com>
 escreveu:

> Qual a posição da palavra "matematica" na série de todas as sequências
> possíveis em ordem alfabética das letras do conjunto da palavra matemática
> {a, c, e, i, m, t} ?
> essa sequencia inicia assim:
> a, c, e, i, m, t, aa, ac, ae, ai, am, at, ca, cc, ce, ci, cm, ct, ea,
> ec, ee, ei, em, et, ia, ic, ie, ii, im ,it, ma, mc, me, mi, mm, mt, ta, 
> tc,
> te, ti, tm, tt, aaa, aac, aae, ...
>
> --
> Esta mensagem foi verificada pelo sistema de antivírus e
> acredita-se estar livre de perigo.


 --
 Esta mensagem foi verificada pelo sistema de antivírus e
 acredita-se estar livre de perigo.
>>>
>>>
>>> --
>>> Esta mensagem foi verificada pelo sistema de antivírus e
>>> acredita-se estar livre 

[obm-l] Re: [obm-l] Re: [obm-l] Re: [obm-l] Re: [obm-l] Ordem Lexicográfica

2019-11-11 Por tôpico Pedro José
Bom dia!

É muita coincidência. Teve um problema agora a respeito de numeração na
terra do IMPA que é muito, mas muito semelhante a esse.
Só que nesse caso caso é o contrário, ou seja a função inversa. O da terra
dos Impa, dá uma posição e quer saber qual o número IMPA.
Aqui se dá uma palavra e se quer a posição.
Podemos fazer uma bijeção: {a,e,c,i,m,t} em {1,2,3,4,5,6}  se x
f(x)
escreveu:

> Boa solução, Rodrigo, mas pq sua contagem resultou um a menos ?
> A resposta é 53.929.309
>
> On Sun, Nov 10, 2019 at 9:31 PM Rodrigo Ângelo 
> wrote:
>
>> A primeira palavra com 10 letras tem posição δ = 6 + 6^2 + ... + 6^9 =
>> 12.093.234.
>>
>> Das palavras que tem 10 letras, fixando a primeira letra em "a", temos
>> 6^9 palavras. Depois, fixando a primeira letra em "c", "e", e "i", temos a
>> mesma quantidade.
>>
>> Então, das palavras de 10 letras, a primeira que começa com "m" tem
>> posição
>> δ + 4*6^9
>>
>> Repetindo esse processo para todas as letras que formam a palavra
>> desejada, temos:
>>
>> δ + 4*6^9 + 5*6^7 + 2*6^6 + 4*6^5 + 5*6^3 + 3*6^2 + 6 = 53.929.308.
>>
>> Atenciosamente,
>> Rodrigo Angelo
>>
>> On Sun, Nov 10, 2019, 18:42 jamil silva  wrote:
>>
>>> Será que sai por base seis ?
>>>
>>> Em dom., 10 de nov. de 2019 às 18:31, lumpa lumpa <1vp4l...@gmail.com>
>>> escreveu:
>>>
 Qual a posição da palavra "matematica" na série de todas as sequências
 possíveis em ordem alfabética das letras do conjunto da palavra matemática
 {a, c, e, i, m, t} ?
 essa sequencia inicia assim:
 a, c, e, i, m, t, aa, ac, ae, ai, am, at, ca, cc, ce, ci, cm, ct, ea,
 ec, ee, ei, em, et, ia, ic, ie, ii, im ,it, ma, mc, me, mi, mm, mt, ta, tc,
 te, ti, tm, tt, aaa, aac, aae, ...

 --
 Esta mensagem foi verificada pelo sistema de antivírus e
 acredita-se estar livre de perigo.
>>>
>>>
>>> --
>>> Esta mensagem foi verificada pelo sistema de antivírus e
>>> acredita-se estar livre de perigo.
>>
>>
>> --
>> Esta mensagem foi verificada pelo sistema de antivírus e
>> acredita-se estar livre de perigo.
>
>
> --
> Esta mensagem foi verificada pelo sistema de antivírus e
> acredita-se estar livre de perigo.

-- 
Esta mensagem foi verificada pelo sistema de antiv�rus e
 acredita-se estar livre de perigo.



[obm-l] Re: [obm-l] Re: [obm-l] Re: [obm-l] Re: [obm-l] Minha solução para o item c) do problema 3 da prova da OBM-2017

2019-11-08 Por tôpico Pedro José
Bom dia!
Creio que não.
Por exemplo, 11 na base 6 é 15.
Daria 39, do jeito que você propôs. Mas dá 31.
Fiz a transformação de 2017 de várias formas e deu sempre 39953.
Alguém tem a resposta?

Saudações, PJMS


Em sex, 8 de nov de 2019 06:58, Esdras Muniz 
escreveu:

> Acho que é só passar 2017 para a base 6 e depois substituir os algarismos
> 0, 1, 2, 3, 4, 5 por 1, 3, 5, 7, 9 respectivamente.
>
> Assim, 2017 na base 6 é 13201, trocando os algarismos, fica: 37513.
>
> Em qui, 7 de nov de 2019 22:16, Cauã DSR 
> escreveu:
>
>> Muito obrigado! É realmente uma honra ler isso.
>> Sobre a questão eu ficarei de analisá-la (principalmente algumas funções
>> que não entendi ainda) no sábado, se possível
>>
>> Em qui, 7 de nov de 2019 9:27 PM, Pedro José 
>> escreveu:
>>
>>> Boa noite!
>>>
>>> Pode-se usar a soma da PG de razão 5 e o primeir termo 1
>>>
>>> então, no sistema impa, teremos 5 números com 1 algarismo, 30 números
>>> com 1ou 2 algarismos, 155 números com até 3 algarismos, 780 números com até
>>> 4 algarismos e Sn=(5^n-1)/4 números com até n algarismos.
>>>
>>> Os algarismos de ordem mais baixa tem um padrão 1 3 5 7 9 1 3 5 7 9...
>>> depois 1 3 5 7 9, depois ...1(5^2vezes)  3..33
>>> .55
>>> 77.777  e assim sucessivamente.
>>>
>>> Do algarismo menos significativo para o mais.
>>> Até 5 só há um algarismo.
>>> De 6= S2 em diante teremos pelo menos dois algarismos
>>> De  31= S3 em diante teremos pelo menos dois algarismos.
>>> De 156 = S4 em diante teremos pelo menos três algarismos
>>>
>>> De Sn+1 teremos pelo menos n algarismos.
>>> Podemos achar os algarismos xn xn-1 ...x2 x1  para um número na base
>>> decimal assim
>>> se ai= 0 xi=1, se ai =1 xi=3; se ai=3 x1= 7 e se ai=4 xi=9 para i <=n
>>> k1=Int (y-S1) e a1= mod (k1;5)
>>> k2=int((y-S2)/5) e a2= mod(k2;5)
>>>
>>> kn=int((y-Sn)/ 5^(n-1)) e an = mod(kn;5)
>>>
>>> Para o número em questão: 2017
>>> k1 = 2016 e a1=1  então x1=3
>>> k2=int((2017-6)/5)=402; a2=2 então x2=5
>>> k3=int((2017-31)/25)=79. a3=4 então x3=9
>>> k4=int((2017-156)/125)=14; a4=4 e x4=9
>>> k5=int((2017-781)/625)=1; a5=1 e x5=3
>>>
>>> Não há mais algarismos pois 2017 <3906=S6. Portanto a representação é:
>>> 39953.
>>>
>>> Porém você, Cauã DSR ,deu uma ideia muito legal.
>>>
>>> Estou querendo provar duas coisas, que não consegui, mas estou certo que
>>> acontece.
>>>
>>>
>>> Se o número em decimal passado para base 5 não tiver algarismos zero,
>>> você pode simplesmente.
>>> 1 permanece 1 na impa
>>> 2 vira 3 na impa
>>> 3 vira 5 na impa
>>> 4 vira 7 na impa.
>>>
>>> Caso você tenha um número com algarismo zero quando transformado para a
>>> base 5,e.g., y= (x6x5x40x2x1)base5
>>> Você pode,sendo o o indicado o menos significativo, Impa (y)=
>>> Concat(impa(x6x5x40);impa(x2x1)) oNde concat é a concatenação.
>>>
>>> Assim para o nosso número original 2017= (31032) base5
>>> impa (310)base 5
>>> (310)base5=80
>>> k1=79; a1=4 e x1=9
>>> k2=int((80-6)/5=14 ;a2=4 e x2=9
>>> k3=int(80-31)/25=1 a3= 1 e x3 = 3
>>>
>>> então impa (310)base5= 399
>>> impa(32)= 53, faz direto pois não tem nenhum algarismo zero.
>>> Então impa (31032)=Concat (impa(310);(impa(32))= 39953; como achado
>>> acima.
>>>
>>> É uma sacada legal. Pois; se não tem algarismo zero na base 5 sai direto.
>>>
>>> Caso haja você quebra o número o da direita sai direto. E na esquerda
>>> você trabalha com um número menor.
>>> Depois é só concatenar.
>>> Só não consegui provar ainda. Sua ideia foi muito boa.
>>>
>>> Parabéns,
>>> PJMS.
>>>
>>>
>>>
>>>
>>>
>>>
>>>
>>>
>>>
>>>
>>>
>>>
>>> Em qui., 7 de nov. de 2019 às 17:27, Pedro José 
>>> escreveu:
>>>
 Boa tarde!
 Você seguiu uma linha de argumentação interessante.
 Mas não está correto.
 Pois existem 5 números com 1 algarismo 5^2 números com 2 algarismos,
 5^3 com 3 e assim sucessivamente.
 Usando a soma da PG
 6-11
 31 -111
 156 -
 781- 1
 Assim o maior número de 4 algarismos  representaria 780.
 O número teria que ter 5 algarismos.

 Saudações,
 PJMS



 Em qui., 7 de nov. de 2019 às 12:36, Cauã DSR 
 escreveu:

>
> Tenho um pequeno problema, eu fiz o item C) do problema 3 da prova da
> OBM de 2017, mas não tenho certeza sobre seu resultado, então achei uma 
> boa
> fazer minha primeira aparição no grupo perguntando se o que fiz está 
> certo.
>
> 3. Na Terra dos Impas, somente os algarismos ímpares são utilizados
> para contar e escrever números. Assim, em vez dos numeros 1, 2, 3, 4, 5, 
> 6,
> 7, 8, 9, 10, 11, 12,. . . os Impas tem os números correspondentes 1, 3, 5,
> 7, 9, 11, 13, 15, 17, 19, 31, 33, . . . (note que os números dos Impas tem
> somente algarismos ímpares). Por exemplo, se
> uma criança tem 11 anos, os Impas diriam que ela tem 31 anos.
>
> c) Escreva, na linguagem dos Impas, o numero que na nossa
> representação decimal é escrito como 2017.
>

[obm-l] Re: [obm-l] Re: [obm-l] Re: [obm-l] Re: [obm-l] Re: [obm-l] Re: [obm-l] Máximo e Mínimo de uma Função

2019-11-03 Por tôpico Luiz Antonio Rodrigues
Olá, Pedro!
Tudo bem?
Muito obrigado pela ajuda!
Gostei muito dessa forma de pensar no problema.
Vou fazer o que você indicou.
Um abraço!
Luiz

On Sun, Nov 3, 2019, 8:00 AM Pedro José  wrote:

> Bom dia!
> Eu coloquei só o resultado do cálculo.
> Note que, para cada jogo de pontos, há três pontos. Os dois da extremidade
> possuem sinais diversos na primeira derivada. Significa que entre eles a
> derivada se anula porque é contínua.
>
> Como o cos(x) apresenta picos de Pi/2 em Pi/2. Você pode fazer uma tabela
> com pontos defasados em 0,5 e verificar quando ocorre uma variação de sinal
> na derivada. Aí entre esses pontos tem um que anula a derivada.
> Pode-se fazer um segmento de reta e ver onde se anula. Nesse ponto calcula
> de novo o valor da derivada e dececide entre que pontos estão. Até que o
> intervalo fique nem pequeno ou o  valor da derivada bem próximo de zero.
> Mas pode usar pesquisa binária que é mais simples. Calcula a média dos "x"
> e o valor da derivada na média.  Depois define entre que pontos se anula.
>
> Saudações,
> PJMS
>
> Em sáb, 2 de nov de 2019 21:20, Luiz Antonio Rodrigues <
> rodrigue...@gmail.com> escreveu:
>
>> Olá, Pedro!
>> Boa noite!
>> Tudo bem?
>> Muito obrigado pelas informações!
>> Vou aguardar seus cálculos!
>> Um abraço!
>> Luiz
>>
>> On Sat, Nov 2, 2019, 6:02 PM Pedro José  wrote:
>>
>>> Boa tarde!
>>>
>>> Quando se fala em o máximo e o mínimo. Entendo como sendo globais, ou
>>> vão acontecer nas extremidades ou em algum máximo e mínimo local, que
>>> também será global.
>>>
>>> f(-12) = 0,453
>>> f(-3) = -0,475
>>>
>>> Não se está pedindo qual o máximo ou mínimo. Se fosse isso dever-se-ia
>>> usar algum método numérico.
>>> Mas a função é contínua nesse intervalo, se ela for monótona esses
>>> seriam o máximo e mínimo.
>>>
>>> Mas se não for ou o máximo e mínimo ocorrerá nas extremidades ou em
>>> algum ponto de derivada zero. Mas está garantido que exista tanto máximo
>>> quanto mínimo.
>>>
>>> Os intervalos, aos quais o 0 pertence também não tem pois a Lim f(x)
>>> quando x --> 0+ não existe, tende a 00 e quando tende a )- também não
>>> existe, tende a -oo.
>>>
>>> E o último intervalo certamente terá um máximo. Mas não um mínimo pois a
>>> primeira parcela de f(x), é monótona decrescente e tende a zero quando
>>> x-->oo e a segundo oscila periodicamente.
>>>
>>> Vamos supor que o mínimo ocorra em xo ==> 1/xo + sen(xo). Agora pegando
>>> o menor valor de x1 com x1> xo e sen (x1) =1 temos que f(x1) = 1/x1 +
>>> sen(x1)
>>>
>>> onde 1/x1 <1/xo e sen(x1)<=sen(xo) ==> f(x1) < f(xo), absurdo. Não
>>> existe mínimo.
>>>
>>> A resposta certa é a a)
>>>
>>> Vou tentar achá-los por método numérico e dou uma "garibada" na resposta.
>>>
>>> Saudações,
>>> PJMS.
>>>
>>>
>>>
>>>
>>>
>>>
>>>
>>>
>>>
>>>
>>> Em sáb., 2 de nov. de 2019 às 16:28, Luiz Antonio Rodrigues <
>>> rodrigue...@gmail.com> escreveu:
>>>
 Olá, Esdras!
 Olá, Rodrigo!
 Tudo bem?
 Muito obrigado pela ajuda!
 Sim, eu também pensei que a questão não tem solução...
 Vou começar a pensar que o problema pede intervalo, ou intervalos, nos
 quais existam mínimos ou máximos locais.
 Se for assim, acho que a saída é pensar nos intervalos onde o zero não
 está presente...
 Acho que meu erro foi pensar que em intervalos fechados existem, com
 certeza, máximos e mínimos locais...
 Vou continuar pensando e escrevo se tiver novidades.
 Abraços!
 Luiz

 On Sat, Nov 2, 2019, 2:55 PM Rodrigo Ângelo 
 wrote:

> Luiz,
>
> Quando x tende a zero pela direita e pela esquerda, f tende a mais e
> menos infinito, respetivamente.
>
> À rigor, a função não tem máximo nem mínimo, porque para todo x no
> domínio da f, é possível encontrar um xmax e um xmin tais que f(xmax) >
> f(x) e f(xmin) < f(x).
>
> Dito isso, eu responderia a alternativa c) por ser a única que contém
> o zero.
>
> On Sat, Nov 2, 2019, 13:53 Luiz Antonio Rodrigues <
> rodrigue...@gmail.com> wrote:
>
>> Olá, pessoal!
>> Bom dia!
>> Estou tentando resolver o seguinte problema:
>>
>> É dada a função:
>>
>> f(x)=(1/x)+sen(x)
>>
>> Pergunta-se:
>>
>> Em quais intervalos abaixo é garantido que encontremos o máximo e o
>> mínimo desta função?
>>
>> a) [-12;-3]
>> b) (-2;-1)
>> c) [-pi;pi]
>> d) [pi;2pi]
>> e) [5;+ infinito)
>>
>> Eu só consegui encontrar um ponto crítico em x=0.
>> Ele não é o único, pois vi isso num gráfico da função.
>> Não sei como resolver a equação f'(x)=0.
>> Acho que estamos lidando com números complexos.
>> Intervalos fechados fazem parte da solução?
>> Pergunto isso porque foi minha resposta, que está errada.
>> Estou confuso.
>> Alguém pode me ajudar?
>> Muito obrigado e um abraço!
>> Luiz
>>
>>
>>
>> --
>> Esta mensagem foi verificada pelo sistema de antivírus e
>> acredita-se 

[obm-l] Re: [obm-l] Re: [obm-l] Re: [obm-l] Re: [obm-l] Re: [obm-l] Máximo e Mínimo de uma Função

2019-11-03 Por tôpico Pedro José
Bom dia!
Eu coloquei só o resultado do cálculo.
Note que, para cada jogo de pontos, há três pontos. Os dois da extremidade
possuem sinais diversos na primeira derivada. Significa que entre eles a
derivada se anula porque é contínua.

Como o cos(x) apresenta picos de Pi/2 em Pi/2. Você pode fazer uma tabela
com pontos defasados em 0,5 e verificar quando ocorre uma variação de sinal
na derivada. Aí entre esses pontos tem um que anula a derivada.
Pode-se fazer um segmento de reta e ver onde se anula. Nesse ponto calcula
de novo o valor da derivada e dececide entre que pontos estão. Até que o
intervalo fique nem pequeno ou o  valor da derivada bem próximo de zero.
Mas pode usar pesquisa binária que é mais simples. Calcula a média dos "x"
e o valor da derivada na média.  Depois define entre que pontos se anula.

Saudações,
PJMS

Em sáb, 2 de nov de 2019 21:20, Luiz Antonio Rodrigues <
rodrigue...@gmail.com> escreveu:

> Olá, Pedro!
> Boa noite!
> Tudo bem?
> Muito obrigado pelas informações!
> Vou aguardar seus cálculos!
> Um abraço!
> Luiz
>
> On Sat, Nov 2, 2019, 6:02 PM Pedro José  wrote:
>
>> Boa tarde!
>>
>> Quando se fala em o máximo e o mínimo. Entendo como sendo globais, ou vão
>> acontecer nas extremidades ou em algum máximo e mínimo local, que também
>> será global.
>>
>> f(-12) = 0,453
>> f(-3) = -0,475
>>
>> Não se está pedindo qual o máximo ou mínimo. Se fosse isso dever-se-ia
>> usar algum método numérico.
>> Mas a função é contínua nesse intervalo, se ela for monótona esses seriam
>> o máximo e mínimo.
>>
>> Mas se não for ou o máximo e mínimo ocorrerá nas extremidades ou em algum
>> ponto de derivada zero. Mas está garantido que exista tanto máximo quanto
>> mínimo.
>>
>> Os intervalos, aos quais o 0 pertence também não tem pois a Lim f(x)
>> quando x --> 0+ não existe, tende a 00 e quando tende a )- também não
>> existe, tende a -oo.
>>
>> E o último intervalo certamente terá um máximo. Mas não um mínimo pois a
>> primeira parcela de f(x), é monótona decrescente e tende a zero quando
>> x-->oo e a segundo oscila periodicamente.
>>
>> Vamos supor que o mínimo ocorra em xo ==> 1/xo + sen(xo). Agora pegando o
>> menor valor de x1 com x1> xo e sen (x1) =1 temos que f(x1) = 1/x1 + sen(x1)
>>
>> onde 1/x1 <1/xo e sen(x1)<=sen(xo) ==> f(x1) < f(xo), absurdo. Não existe
>> mínimo.
>>
>> A resposta certa é a a)
>>
>> Vou tentar achá-los por método numérico e dou uma "garibada" na resposta.
>>
>> Saudações,
>> PJMS.
>>
>>
>>
>>
>>
>>
>>
>>
>>
>>
>> Em sáb., 2 de nov. de 2019 às 16:28, Luiz Antonio Rodrigues <
>> rodrigue...@gmail.com> escreveu:
>>
>>> Olá, Esdras!
>>> Olá, Rodrigo!
>>> Tudo bem?
>>> Muito obrigado pela ajuda!
>>> Sim, eu também pensei que a questão não tem solução...
>>> Vou começar a pensar que o problema pede intervalo, ou intervalos, nos
>>> quais existam mínimos ou máximos locais.
>>> Se for assim, acho que a saída é pensar nos intervalos onde o zero não
>>> está presente...
>>> Acho que meu erro foi pensar que em intervalos fechados existem, com
>>> certeza, máximos e mínimos locais...
>>> Vou continuar pensando e escrevo se tiver novidades.
>>> Abraços!
>>> Luiz
>>>
>>> On Sat, Nov 2, 2019, 2:55 PM Rodrigo Ângelo 
>>> wrote:
>>>
 Luiz,

 Quando x tende a zero pela direita e pela esquerda, f tende a mais e
 menos infinito, respetivamente.

 À rigor, a função não tem máximo nem mínimo, porque para todo x no
 domínio da f, é possível encontrar um xmax e um xmin tais que f(xmax) >
 f(x) e f(xmin) < f(x).

 Dito isso, eu responderia a alternativa c) por ser a única que contém o
 zero.

 On Sat, Nov 2, 2019, 13:53 Luiz Antonio Rodrigues <
 rodrigue...@gmail.com> wrote:

> Olá, pessoal!
> Bom dia!
> Estou tentando resolver o seguinte problema:
>
> É dada a função:
>
> f(x)=(1/x)+sen(x)
>
> Pergunta-se:
>
> Em quais intervalos abaixo é garantido que encontremos o máximo e o
> mínimo desta função?
>
> a) [-12;-3]
> b) (-2;-1)
> c) [-pi;pi]
> d) [pi;2pi]
> e) [5;+ infinito)
>
> Eu só consegui encontrar um ponto crítico em x=0.
> Ele não é o único, pois vi isso num gráfico da função.
> Não sei como resolver a equação f'(x)=0.
> Acho que estamos lidando com números complexos.
> Intervalos fechados fazem parte da solução?
> Pergunto isso porque foi minha resposta, que está errada.
> Estou confuso.
> Alguém pode me ajudar?
> Muito obrigado e um abraço!
> Luiz
>
>
>
> --
> Esta mensagem foi verificada pelo sistema de antivírus e
> acredita-se estar livre de perigo.


 --
 Esta mensagem foi verificada pelo sistema de antivírus e
 acredita-se estar livre de perigo.
>>>
>>>
>>> --
>>> Esta mensagem foi verificada pelo sistema de antivírus e
>>> acredita-se estar livre de perigo.
>>
>>
>> --
>> Esta mensagem foi verificada pelo sistema de antivírus e
>> acredita-se estar livre de 

[obm-l] Re: [obm-l] Re: [obm-l] Re: [obm-l] Re: [obm-l] Máximo e Mínimo de uma Função

2019-11-02 Por tôpico Luiz Antonio Rodrigues
Olá, Pedro!
Boa noite!
Tudo bem?
Muito obrigado pelas informações!
Vou aguardar seus cálculos!
Um abraço!
Luiz

On Sat, Nov 2, 2019, 6:02 PM Pedro José  wrote:

> Boa tarde!
>
> Quando se fala em o máximo e o mínimo. Entendo como sendo globais, ou vão
> acontecer nas extremidades ou em algum máximo e mínimo local, que também
> será global.
>
> f(-12) = 0,453
> f(-3) = -0,475
>
> Não se está pedindo qual o máximo ou mínimo. Se fosse isso dever-se-ia
> usar algum método numérico.
> Mas a função é contínua nesse intervalo, se ela for monótona esses seriam
> o máximo e mínimo.
>
> Mas se não for ou o máximo e mínimo ocorrerá nas extremidades ou em algum
> ponto de derivada zero. Mas está garantido que exista tanto máximo quanto
> mínimo.
>
> Os intervalos, aos quais o 0 pertence também não tem pois a Lim f(x)
> quando x --> 0+ não existe, tende a 00 e quando tende a )- também não
> existe, tende a -oo.
>
> E o último intervalo certamente terá um máximo. Mas não um mínimo pois a
> primeira parcela de f(x), é monótona decrescente e tende a zero quando
> x-->oo e a segundo oscila periodicamente.
>
> Vamos supor que o mínimo ocorra em xo ==> 1/xo + sen(xo). Agora pegando o
> menor valor de x1 com x1> xo e sen (x1) =1 temos que f(x1) = 1/x1 + sen(x1)
>
> onde 1/x1 <1/xo e sen(x1)<=sen(xo) ==> f(x1) < f(xo), absurdo. Não existe
> mínimo.
>
> A resposta certa é a a)
>
> Vou tentar achá-los por método numérico e dou uma "garibada" na resposta.
>
> Saudações,
> PJMS.
>
>
>
>
>
>
>
>
>
>
> Em sáb., 2 de nov. de 2019 às 16:28, Luiz Antonio Rodrigues <
> rodrigue...@gmail.com> escreveu:
>
>> Olá, Esdras!
>> Olá, Rodrigo!
>> Tudo bem?
>> Muito obrigado pela ajuda!
>> Sim, eu também pensei que a questão não tem solução...
>> Vou começar a pensar que o problema pede intervalo, ou intervalos, nos
>> quais existam mínimos ou máximos locais.
>> Se for assim, acho que a saída é pensar nos intervalos onde o zero não
>> está presente...
>> Acho que meu erro foi pensar que em intervalos fechados existem, com
>> certeza, máximos e mínimos locais...
>> Vou continuar pensando e escrevo se tiver novidades.
>> Abraços!
>> Luiz
>>
>> On Sat, Nov 2, 2019, 2:55 PM Rodrigo Ângelo 
>> wrote:
>>
>>> Luiz,
>>>
>>> Quando x tende a zero pela direita e pela esquerda, f tende a mais e
>>> menos infinito, respetivamente.
>>>
>>> À rigor, a função não tem máximo nem mínimo, porque para todo x no
>>> domínio da f, é possível encontrar um xmax e um xmin tais que f(xmax) >
>>> f(x) e f(xmin) < f(x).
>>>
>>> Dito isso, eu responderia a alternativa c) por ser a única que contém o
>>> zero.
>>>
>>> On Sat, Nov 2, 2019, 13:53 Luiz Antonio Rodrigues 
>>> wrote:
>>>
 Olá, pessoal!
 Bom dia!
 Estou tentando resolver o seguinte problema:

 É dada a função:

 f(x)=(1/x)+sen(x)

 Pergunta-se:

 Em quais intervalos abaixo é garantido que encontremos o máximo e o
 mínimo desta função?

 a) [-12;-3]
 b) (-2;-1)
 c) [-pi;pi]
 d) [pi;2pi]
 e) [5;+ infinito)

 Eu só consegui encontrar um ponto crítico em x=0.
 Ele não é o único, pois vi isso num gráfico da função.
 Não sei como resolver a equação f'(x)=0.
 Acho que estamos lidando com números complexos.
 Intervalos fechados fazem parte da solução?
 Pergunto isso porque foi minha resposta, que está errada.
 Estou confuso.
 Alguém pode me ajudar?
 Muito obrigado e um abraço!
 Luiz



 --
 Esta mensagem foi verificada pelo sistema de antivírus e
 acredita-se estar livre de perigo.
>>>
>>>
>>> --
>>> Esta mensagem foi verificada pelo sistema de antivírus e
>>> acredita-se estar livre de perigo.
>>
>>
>> --
>> Esta mensagem foi verificada pelo sistema de antivírus e
>> acredita-se estar livre de perigo.
>
>
> --
> Esta mensagem foi verificada pelo sistema de antivírus e
> acredita-se estar livre de perigo.

-- 
Esta mensagem foi verificada pelo sistema de antiv�rus e
 acredita-se estar livre de perigo.



[obm-l] Re: [obm-l] Re: [obm-l] Re: [obm-l] Re: [obm-l] Re: [obm-l] Re: [obm-l] Domínio de uma Função

2019-10-30 Por tôpico Luiz Antonio Rodrigues
Olá, Pedro!
Tudo bem?
Vou ficar atento em relação ao que você mencionou.
Muito obrigado pela ajuda!
Um abraço!
Luiz

On Wed, Oct 30, 2019, 1:14 PM Pedro José  wrote:

> Boa tarde!
> Faltara mencionar que o máximo também era local.
> Quando eu falo vizinhança de 0, é 0+ e 0-
> Se você observar para 0+ temos a primeira derivada positiva, logo a função
> é crescente. x^(-1/3)+1
> Para 0- o primeiro termo se sobressai ao segundo e a derivada é negativa,
> logo o valor também cresce se andarmos para esquerda, logo é mínimo local.
> Mas se a função tiver um comportamento monótono, tanto a esquerda quanto a
> direita do ponto, num intervalo mais amplo, você pode usar esse intervalo
> para analisar.
> Só lembre que se na prova, caso você seja estudante, não pode usar excel.
>
> Saudações,
> PJMS
>
> Em ter, 29 de out de 2019 às 21:54, Luiz Antonio Rodrigues <
> rodrigue...@gmail.com> escreveu:
>
>> Olá, Pedro!
>> Olá, Claudio!
>> Tudo bem?
>> Sim, cheguei agora há pouco nestes valores para máximo e mínimo locais.
>> Muito obrigado!
>> E você citou a minha próxima dúvida: existe um tamanho "ideal" para o
>> intervalo na vizinhança do ponto crítico?
>> Eu sei que ele não pode ser muito grande...
>> Mas ele pode ser bem pequeno?
>> Por exemplo, se o ponto crítico tiver x=5, o intervalo pode ser (4,6)?
>> E (4.5,5.5)?
>> Fiz várias pesquisas na internet e continuo confuso...
>> Claudio, vou seguir sua sugestão para analisar outras funções utiluzando
>> o Excel.
>> Muito obrigado pela ajuda!
>> Luiz
>>
>>
>>
>> On Tue, Oct 29, 2019, 7:38 PM Pedro José  wrote:
>>
>>> Boa noite.
>>> Não tinha conhecimento do fato citado por Ralph;
>>> Mas essa função tem um mínimo local em x=0 e um máximo em x=-1
>>> No ponto x=1 a segunda derivada é negativa, Em x=0 não existe a primeira
>>> derivad, tem que fazer análise da vizinhança do ponto.
>>>
>>> Saudações,
>>> PJMS
>>>
>>> Em ter, 29 de out de 2019 às 12:29, Claudio Buffara <
>>> claudio.buff...@gmail.com> escreveu:
>>>
 Use uma planilha. Eu acho melhor pra analisar funções.

 Enviado do meu iPhone

 Em 29 de out de 2019, à(s) 11:23, Luiz Antonio Rodrigues <
 rodrigue...@gmail.com> escreveu:

 
 Olá, Claudio!
 Bom dia!
 Foi assim que eu pensei também...
 Não entendi por que a calculadora gráfica indicou domínio [0, +
 infinito).
 Vou verificar tudo novamente...
 Muito obrigado pela ajuda!Â
 Abraço!
 Luiz

 On Tue, Oct 29, 2019, 10:49 AM Claudio Buffara <
 claudio.buff...@gmail.com> wrote:

> Estritamente falando, o domínio da função não foi definido.
> Nestes casos, o usual é tomar por domínio o maior subconjunto de R
> no qual a fórmula faz sentido.
> E, neste caso específico, a fórmula faz sentido para todo x real.
>
> O gráfico de h(x) = x^(2/3) tem uma "ponta" em x = 0, de modo que  a
> derivada h'(x) não é definida na origem.
>
> Mas não deveria haver problema algum em x = -1.
>
>
> On Tue, Oct 29, 2019 at 4:57 AM Luiz Antonio Rodrigues <
> rodrigue...@gmail.com> wrote:
>
>> Olá, pessoal!
>> Tudo bem?
>> Estou tentando descobrir os pontos  de máximo e mínimo da função:
>>
>> f(x)=1.5*(x)^(2/3)+x
>>
>> A primeira derivada se anula em x=-1.
>> Mas porque -1 não pertence ao domínio da função?
>> Vi isso numa calculadora gráfica.
>> Eu não consigo entender isso...
>> Não estou tirando a raiz cúbica de um número ao quadrado?
>> Alguém pode me ajudar?
>> Muito obrigado!
>> Luiz
>>
>> --
>> Esta mensagem foi verificada pelo sistema de antivírus e
>> acredita-se estar livre de perigo.
>
>
> --
> Esta mensagem foi verificada pelo sistema de antivírus e
> acredita-se estar livre de perigo.


 --
 Esta mensagem foi verificada pelo sistema de antivírus e
 acredita-se estar livre de perigo.


 --
 Esta mensagem foi verificada pelo sistema de antivírus e
 acredita-se estar livre de perigo.

>>>
>>> --
>>> Esta mensagem foi verificada pelo sistema de antivírus e
>>> acredita-se estar livre de perigo.
>>
>>
>> --
>> Esta mensagem foi verificada pelo sistema de antivírus e
>> acredita-se estar livre de perigo.
>
>
> --
> Esta mensagem foi verificada pelo sistema de antivírus e
> acredita-se estar livre de perigo.

-- 
Esta mensagem foi verificada pelo sistema de antiv�rus e
 acredita-se estar livre de perigo.



[obm-l] Re: [obm-l] Re: [obm-l] Re: [obm-l] Re: [obm-l] Re: [obm-l] Domínio de uma Função

2019-10-30 Por tôpico Pedro José
Boa tarde!
Faltara mencionar que o máximo também era local.
Quando eu falo vizinhança de 0, é 0+ e 0-
Se você observar para 0+ temos a primeira derivada positiva, logo a função
é crescente. x^(-1/3)+1
Para 0- o primeiro termo se sobressai ao segundo e a derivada é negativa,
logo o valor também cresce se andarmos para esquerda, logo é mínimo local.
Mas se a função tiver um comportamento monótono, tanto a esquerda quanto a
direita do ponto, num intervalo mais amplo, você pode usar esse intervalo
para analisar.
Só lembre que se na prova, caso você seja estudante, não pode usar excel.

Saudações,
PJMS

Em ter, 29 de out de 2019 às 21:54, Luiz Antonio Rodrigues <
rodrigue...@gmail.com> escreveu:

> Olá, Pedro!
> Olá, Claudio!
> Tudo bem?
> Sim, cheguei agora há pouco nestes valores para máximo e mínimo locais.
> Muito obrigado!
> E você citou a minha próxima dúvida: existe um tamanho "ideal" para o
> intervalo na vizinhança do ponto crítico?
> Eu sei que ele não pode ser muito grande...
> Mas ele pode ser bem pequeno?
> Por exemplo, se o ponto crítico tiver x=5, o intervalo pode ser (4,6)?
> E (4.5,5.5)?
> Fiz várias pesquisas na internet e continuo confuso...
> Claudio, vou seguir sua sugestão para analisar outras funções utiluzando o
> Excel.
> Muito obrigado pela ajuda!
> Luiz
>
>
>
> On Tue, Oct 29, 2019, 7:38 PM Pedro José  wrote:
>
>> Boa noite.
>> Não tinha conhecimento do fato citado por Ralph;
>> Mas essa função tem um mínimo local em x=0 e um máximo em x=-1
>> No ponto x=1 a segunda derivada é negativa, Em x=0 não existe a primeira
>> derivad, tem que fazer análise da vizinhança do ponto.
>>
>> Saudações,
>> PJMS
>>
>> Em ter, 29 de out de 2019 às 12:29, Claudio Buffara <
>> claudio.buff...@gmail.com> escreveu:
>>
>>> Use uma planilha. Eu acho melhor pra analisar funções.
>>>
>>> Enviado do meu iPhone
>>>
>>> Em 29 de out de 2019, à(s) 11:23, Luiz Antonio Rodrigues <
>>> rodrigue...@gmail.com> escreveu:
>>>
>>> 
>>> Olá, Claudio!
>>> Bom dia!
>>> Foi assim que eu pensei também...
>>> Não entendi por que a calculadora gráfica indicou domínio [0, +
>>> infinito).
>>> Vou verificar tudo novamente...
>>> Muito obrigado pela ajuda!Â
>>> Abraço!
>>> Luiz
>>>
>>> On Tue, Oct 29, 2019, 10:49 AM Claudio Buffara <
>>> claudio.buff...@gmail.com> wrote:
>>>
 Estritamente falando, o domínio da função não foi definido.
 Nestes casos, o usual é tomar por domínio o maior subconjunto de R no
 qual a fórmula faz sentido.
 E, neste caso específico, a fórmula faz sentido para todo x real.

 O gráfico de h(x) = x^(2/3) tem uma "ponta" em x = 0, de modo que  a
 derivada h'(x) não é definida na origem.

 Mas não deveria haver problema algum em x = -1.


 On Tue, Oct 29, 2019 at 4:57 AM Luiz Antonio Rodrigues <
 rodrigue...@gmail.com> wrote:

> Olá, pessoal!
> Tudo bem?
> Estou tentando descobrir os pontos  de máximo e mínimo da função:
>
> f(x)=1.5*(x)^(2/3)+x
>
> A primeira derivada se anula em x=-1.
> Mas porque -1 não pertence ao domínio da função?
> Vi isso numa calculadora gráfica.
> Eu não consigo entender isso...
> Não estou tirando a raiz cúbica de um número ao quadrado?
> Alguém pode me ajudar?
> Muito obrigado!
> Luiz
>
> --
> Esta mensagem foi verificada pelo sistema de antivírus e
> acredita-se estar livre de perigo.


 --
 Esta mensagem foi verificada pelo sistema de antivírus e
 acredita-se estar livre de perigo.
>>>
>>>
>>> --
>>> Esta mensagem foi verificada pelo sistema de antivírus e
>>> acredita-se estar livre de perigo.
>>>
>>>
>>> --
>>> Esta mensagem foi verificada pelo sistema de antivírus e
>>> acredita-se estar livre de perigo.
>>>
>>
>> --
>> Esta mensagem foi verificada pelo sistema de antivírus e
>> acredita-se estar livre de perigo.
>
>
> --
> Esta mensagem foi verificada pelo sistema de antivírus e
> acredita-se estar livre de perigo.

-- 
Esta mensagem foi verificada pelo sistema de antiv�rus e
 acredita-se estar livre de perigo.



[obm-l] Re: [obm-l] Re: [obm-l] Re: [obm-l] Re: [obm-l] Domínio de uma Função

2019-10-29 Por tôpico Luiz Antonio Rodrigues
Olá, Pedro!
Olá, Claudio!
Tudo bem?
Sim, cheguei agora há pouco nestes valores para máximo e mínimo locais.
Muito obrigado!
E você citou a minha próxima dúvida: existe um tamanho "ideal" para o
intervalo na vizinhança do ponto crítico?
Eu sei que ele não pode ser muito grande...
Mas ele pode ser bem pequeno?
Por exemplo, se o ponto crítico tiver x=5, o intervalo pode ser (4,6)?
E (4.5,5.5)?
Fiz várias pesquisas na internet e continuo confuso...
Claudio, vou seguir sua sugestão para analisar outras funções utiluzando o
Excel.
Muito obrigado pela ajuda!
Luiz



On Tue, Oct 29, 2019, 7:38 PM Pedro José  wrote:

> Boa noite.
> Não tinha conhecimento do fato citado por Ralph;
> Mas essa função tem um mínimo local em x=0 e um máximo em x=-1
> No ponto x=1 a segunda derivada é negativa, Em x=0 não existe a primeira
> derivad, tem que fazer análise da vizinhança do ponto.
>
> Saudações,
> PJMS
>
> Em ter, 29 de out de 2019 às 12:29, Claudio Buffara <
> claudio.buff...@gmail.com> escreveu:
>
>> Use uma planilha. Eu acho melhor pra analisar funções.
>>
>> Enviado do meu iPhone
>>
>> Em 29 de out de 2019, à(s) 11:23, Luiz Antonio Rodrigues <
>> rodrigue...@gmail.com> escreveu:
>>
>> 
>> Olá, Claudio!
>> Bom dia!
>> Foi assim que eu pensei também...
>> Não entendi por que a calculadora gráfica indicou domínio [0, +
>> infinito).
>> Vou verificar tudo novamente...
>> Muito obrigado pela ajuda!Â
>> Abraço!
>> Luiz
>>
>> On Tue, Oct 29, 2019, 10:49 AM Claudio Buffara 
>> wrote:
>>
>>> Estritamente falando, o domínio da função não foi definido.
>>> Nestes casos, o usual é tomar por domínio o maior subconjunto de R no
>>> qual a fórmula faz sentido.
>>> E, neste caso específico, a fórmula faz sentido para todo x real.
>>>
>>> O gráfico de h(x) = x^(2/3) tem uma "ponta" em x = 0, de modo que  a
>>> derivada h'(x) não é definida na origem.
>>>
>>> Mas não deveria haver problema algum em x = -1.
>>>
>>>
>>> On Tue, Oct 29, 2019 at 4:57 AM Luiz Antonio Rodrigues <
>>> rodrigue...@gmail.com> wrote:
>>>
 Olá, pessoal!
 Tudo bem?
 Estou tentando descobrir os pontos  de máximo e mínimo da função:

 f(x)=1.5*(x)^(2/3)+x

 A primeira derivada se anula em x=-1.
 Mas porque -1 não pertence ao domínio da função?
 Vi isso numa calculadora gráfica.
 Eu não consigo entender isso...
 Não estou tirando a raiz cúbica de um número ao quadrado?
 Alguém pode me ajudar?
 Muito obrigado!
 Luiz

 --
 Esta mensagem foi verificada pelo sistema de antivírus e
 acredita-se estar livre de perigo.
>>>
>>>
>>> --
>>> Esta mensagem foi verificada pelo sistema de antivírus e
>>> acredita-se estar livre de perigo.
>>
>>
>> --
>> Esta mensagem foi verificada pelo sistema de antivírus e
>> acredita-se estar livre de perigo.
>>
>>
>> --
>> Esta mensagem foi verificada pelo sistema de antivírus e
>> acredita-se estar livre de perigo.
>>
>
> --
> Esta mensagem foi verificada pelo sistema de antivírus e
> acredita-se estar livre de perigo.

-- 
Esta mensagem foi verificada pelo sistema de antiv�rus e
 acredita-se estar livre de perigo.



Re: [obm-l] Re: [obm-l] Re: [obm-l] Re: [obm-l] Domínio de uma Função

2019-10-29 Por tôpico Claudio Buffara
Eu uso Excel.
Muito útil pra analisar gráficos e gerar conjecturas em cálculo, teoria dos 
números e combinatória.

Abs

Enviado do meu iPhone

> Em 29 de out de 2019, à(s) 18:54, Luiz Antonio Rodrigues 
>  escreveu:
> 
> 
> Oi, Claudio!
> Tudo bem?
> Você sugere uma planilha tipo Excel ou Numbers?
> Eu nunca pensei nisso...
> Acho que é uma ideia excelente!
> 
> 
>> On Tue, Oct 29, 2019, 12:29 PM Claudio Buffara  
>> wrote:
>> Use uma planilha. Eu acho melhor pra analisar funções.
>> 
>> Enviado do meu iPhone
>> 
>>> Em 29 de out de 2019, Ã (s) 11:23, Luiz Antonio Rodrigues 
>>>  escreveu:
>>> 
>>> 
>>> Olá, Claudio!
>>> Bom dia!
>>> Foi assim que eu pensei também...
>>> Não entendi por que a calculadora gráfica indicou domínio [0, + 
>>> infinito).
>>> Vou verificar tudo novamente...
>>> Muito obrigado pela ajuda! 
>>> Abraço!
>>> Luiz
>>> 
 On Tue, Oct 29, 2019, 10:49 AM Claudio Buffara  
 wrote:
 Estritamente falando, o domínio da função não foi definido.
 Nestes casos, o usual é tomar por domínio o maior subconjunto de R 
 no qual a fórmula faz sentido.
 E, neste caso específico, a fórmula faz sentido para todo x real.
 
 O gráfico de h(x) = x^(2/3) tem uma "ponta" em x = 0, de modo que  a 
 derivada h'(x) não é definida na origem.
 
 Mas não deveria haver problema algum em x = -1.
 
 
> On Tue, Oct 29, 2019 at 4:57 AM Luiz Antonio Rodrigues 
>  wrote:
> Olá, pessoal!
> Tudo bem?
> Estou tentando descobrir os pontos  de máximo e mínimo da 
> função:
> 
> f(x)=1.5*(x)^(2/3)+x
> 
> A primeira derivada se anula em x=-1.
> Mas porque -1 não pertence ao domínio da função?
> Vi isso numa calculadora gráfica.
> Eu não consigo entender isso...
> Não estou tirando a raiz cúbica de um número ao quadrado?
> Alguém pode me ajudar?
> Muito obrigado!
> Luiz
> 
> -- 
> Esta mensagem foi verificada pelo sistema de antivírus e 
> acredita-se estar livre de perigo.
 
 -- 
 Esta mensagem foi verificada pelo sistema de antivírus e 
 acredita-se estar livre de perigo.
>>> 
>>> -- 
>>> Esta mensagem foi verificada pelo sistema de antivírus e 
>>> acredita-se estar livre de perigo.
>> 
>> -- 
>> Esta mensagem foi verificada pelo sistema de antivírus e 
>> acredita-se estar livre de perigo.
> 
> -- 
> Esta mensagem foi verificada pelo sistema de antivírus e 
> acredita-se estar livre de perigo.

-- 
Esta mensagem foi verificada pelo sistema de antiv�rus e
 acredita-se estar livre de perigo.



[obm-l] Re: [obm-l] Re: [obm-l] Re: [obm-l] Re: [obm-l] Aproximação Linear e Quadrática

2019-10-13 Por tôpico Rodrigo Ângelo
Também acho que está correto.

x=0 é ponto de inflexão de f(x)=x^3

Perto de 0 a função se parece com a função constante 0

On Sun, Oct 13, 2019, 00:00 Ralph Teixeira  wrote:

> Pois eh, para mim essas sao as respostas corretas: "0" e "0" de novo. Se
> voce usar Serie de Taylor, faz sentido! Perto de 0, x^3 fica mais bem
> aproximado pela expressao "0" do que qualquer outra funcao afim ou
> quadratica!
>
> Abraco, Ralph.
>
> On Sat, Oct 12, 2019 at 7:29 PM Luiz Antonio Rodrigues <
> rodrigue...@gmail.com> wrote:
>
>> Olá, Ralph!
>> Tudo bem?
>> Sim, eu pensei nisso...
>>
>> Para a aproximação linear eu usei:
>> L(x) ~= f(0) + f'(0)*x = 0
>>
>> Para a quadrática:
>> Q(x) ~= f(0) + f'(0)*x + (1/2)*f''(0)*x^2 = 0
>>
>> Estranho, não é?
>>
>>
>> On Sat, Oct 12, 2019, 7:09 PM Ralph Teixeira  wrote:
>>
>>> Hm, por que nao eh a resposta correta? x^3 eh BEM perto de 0 quando x eh
>>> pequeno...
>>>
>>> Abraco, Ralph.
>>>
>>> On Sat, Oct 12, 2019 at 5:15 PM Luiz Antonio Rodrigues <
>>> rodrigue...@gmail.com> wrote:
>>>
 Olá, pessoal!
 Boa tarde!
 Tudo bem?
 Preciso de uma dica.
 Estou calculando as aproximações linear e quadrática de:

 f(x)=x^3

 Nas duas eu obtive zero, usando a série de Taylor, que não é a resposta
 correta.
 Alguém tem alguma ideia?
 Muito obrigado!
 Luiz

 --
 Esta mensagem foi verificada pelo sistema de antivírus e
 acredita-se estar livre de perigo.
>>>
>>>
>>> --
>>> Esta mensagem foi verificada pelo sistema de antivírus e
>>> acredita-se estar livre de perigo.
>>
>>
>> --
>> Esta mensagem foi verificada pelo sistema de antivírus e
>> acredita-se estar livre de perigo.
>
>
> --
> Esta mensagem foi verificada pelo sistema de antivírus e
> acredita-se estar livre de perigo.

-- 
Esta mensagem foi verificada pelo sistema de antiv�rus e
 acredita-se estar livre de perigo.



[obm-l] Re: [obm-l] Re: [obm-l] Re: [obm-l] Re: [obm-l] Números primos

2019-08-29 Por tôpico Ralph Teixeira
Exato, 6 é um número pequeno com "muitos" divisores, então é um bom ponto
de partida...

Claro, a gente podia continuar analisando o problema e achando mais e mais
restrições (módulo 12... módulo 15... módulo 120...)... Mas, em algum
momento, você tem que partir para tentar uns números e ver o que acontece,
senão não fecha nunca. :D

On Thu, Aug 29, 2019 at 1:02 PM Claudio Buffara 
wrote:

> Acho que apenas o fato de que, apesar de existirem 6 restos possíveis ao
> se dividir um inteiro por 6, os primos maiores que 3 deixam apenas resto 1
> ou resto 5 (== -1).
>
>
> On Thu, Aug 29, 2019 at 12:42 PM Carlos Monteiro <
> cacacarlosalberto1...@gmail.com> wrote:
>
>> Valeu!
>> Tem alguma motivação para a congruência mod 6?
>>
>>
>> Em qui, 29 de ago de 2019 12:12, Ralph Teixeira 
>> escreveu:
>>
>>> Resposta curta: 3, 7 e 13 servem.
>>>
>>> Resposta longa:
>>> Sejam p1>> p1=2, porque então a soma seria par.
>>> Afirmo que p1=3. De fato, caso contrário, todos eles deixariam resto 1
>>> ou -1 (hm, eu devia dizer 5, mas vou escrever -1 mesmo) na divisão por 6.
>>> Mas então seus quadrados deixariam resto 1 na divisão por 6, e a soma dos
>>> quadrados deixaria resto 3, absurdo.
>>> Note que p2 e p3 têm que deixar o mesmo resto (1 ou -1) na divisão por 6
>>> (caso contrário, p2+p3=6a+1+6b-1 seria divisível por 6, então 3+p2+p3 seria
>>> divisível por 3).
>>> Então a gente quer coisas do tipo {3,6a+1,6b+1} ou {3,6a-1,6b-1}. Isto
>>> me leva a tentar
>>> {3,5,11} -- soma 19, soma dos quadrados 155; Quebrei a cara.
>>> {3,7,13} -- soma 23, soma dos quadrados 227. Ambos primos! Funcionou!
>>>
>>> Abraço, Ralph.
>>>
>>> On Thu, Aug 29, 2019 at 11:35 AM Carlos Monteiro <
>>> cacacarlosalberto1...@gmail.com> wrote:
>>>
 Encontre três números primos distintos dois a dois tais que sua soma e
 a soma dos seus quadrados são números primos também.

 --
 Esta mensagem foi verificada pelo sistema de antivírus e
 acredita-se estar livre de perigo.
>>>
>>>
>>> --
>>> Esta mensagem foi verificada pelo sistema de antivírus e
>>> acredita-se estar livre de perigo.
>>
>>
>> --
>> Esta mensagem foi verificada pelo sistema de antivírus e
>> acredita-se estar livre de perigo.
>
>
> --
> Esta mensagem foi verificada pelo sistema de antivírus e
> acredita-se estar livre de perigo.

-- 
Esta mensagem foi verificada pelo sistema de antiv�rus e
 acredita-se estar livre de perigo.



[obm-l] Re: [obm-l] Re: [obm-l] Re: [obm-l] Re: [obm-l] Aproximação Linear

2019-08-25 Por tôpico Luiz Antonio Rodrigues
Olá, Claudio!
Sim!
Foi exatamente isso que aconteceu comigo!
Muito obrigado pela ajuda!

On Sun, Aug 25, 2019, 1:27 PM Claudio Buffara 
wrote:

> Fico feliz de ter podido ajudar!
>
> Infelizmente, os livros de cálculo focam quase que exclusivamente na noção
> de derivada como a inclinação da reta tangente ao gráfico da função.
> Obviamente isso está correto, mas é apenas uma forma de ver a derivada, e
> que não é facilmente generalizável pra 2 ou mais dimensões.
> Um outro ponto de vista, que às vezes é mais útil, especialmente no R^n, é
> entender a derivada como uma transformação linear que aproxima a função na
> vizinhança de um ponto, com um erro que tende a zero mais rapidamente do
> que o erro na determinação do ponto do domínio.
>
>
> --
> Esta mensagem foi verificada pelo sistema de antivírus e
> acredita-se estar livre de perigo.

-- 
Esta mensagem foi verificada pelo sistema de antiv�rus e
 acredita-se estar livre de perigo.



[obm-l] Re: [obm-l] Re: [obm-l] Re: [obm-l] Re: [obm-l] Re: [obm-l] Re: [obm-l] Re: [obm-l] Recorrência

2019-08-04 Por tôpico Arthur Queiroz
Sim. Corrigindo:
G(n+1) = [G(1)]^(2^n)
G(n) = [G(1)]^[2^(n-1)] = [3^2]^[2^(n-1)] = 3^(2^n)
O resto está correto, eu acredito.

Em qui, 1 de ago de 2019 07:55, Caio Costa  escreveu:

> Seria G(n+1) = [G(1)]^(2^n)?
>
> On Wed, Jul 31, 2019, 9:24 PM Arthur Queiroz 
> wrote:
>
>> Complementando, dá pra achar o termo geral assim:
>> N(n+1) = 2*N(n)^2 + 2*N(n)
>> Multiplicando os dois lados por dois e adicionando um:
>> 2*N(n+1) + 1= 4*N(n)^2+4*N(n)+1
>> Fatorando o lado direito:
>> 2*N(n+1) + 1 = (2*N(n)+1)^2
>> Agora, sendo G(n) = 2*N(n)+1, teremos que:
>> G(n+1) = G(n)^2 = ((G(n-1)^2)^2 = ... = G(1)^(2^(n-1))
>> Só que G(1) = 2*N(1)+1 = 2*4+1 = 9 = 3^2, logo G(n+1) = G(1)^(2^(n-1)) =
>> (3^2)^(2^(n-1)) = 3^(2^n)
>> Voltando, N(n), pela equação anterior, é igual a (G(n)-1)/2
>> Logo, N(n) = (3^(2^n)-1)/2
>> Por fim, a resposta do problema é N(2013)/(N(2013)+1) =
>> (3^(2^2013)-1)/(3^(2^2013) + 1)
>>
>> Em qua, 31 de jul de 2019 às 20:20, Claudio Buffara <
>> claudio.buff...@gmail.com> escreveu:
>>
>>> x(0) = 2 ==> x(1) = 4/5 ==> x(2) = 40/41 ==> x(3) = 3800/3801
>>>
>>> Em geral, se x(n) = a/b (a e b inteiros primos entre si) ==> x(n+1) =
>>> 2ab/(a^2+b^2) < 1.
>>> Além disso, olhando os primeiros termos, parece que a^2 + b^2 = 2ab + 1
>>> ==> (a - b)^2 = 1 ==> b = a + 1
>>> E, de fato, se x(n) = a/(a+1), então x(n+1) = 2a(a+1)/(2a^2+2a+1)
>>>
>>> Assim, a sequência de numeradores será:
>>> N(1) = 4,
>>> N(2) = 2*4*(4+1) = 40
>>> N(3) = 2*40*(40+1) = 3800
>>> ...
>>> N(n+1) = 2*N(n)*(N(n)+1)
>>>
>>> De bate pronto não vejo uma fórmula fechada pra esta recorrência, mas o
>>> número desejado é N(2013)/(N(2013)+1), e N(2013) é um inteiro gigantesco.
>>>
>>>
>>>
>>> On Wed, Jul 31, 2019 at 7:59 PM Claudio Buffara <
>>> claudio.buff...@gmail.com> wrote:
>>>
 Exatamente isso!

 On Wed, Jul 31, 2019 at 7:38 PM Caio Costa  wrote:

> não vai dar 1, mas vai dar um número muito próximo de 1 (valor exato).
> O que eles estão dizendo (pelo que entendi) é que a diferença para 1 é tão
> pequena, desprezível, que não será percebida pelo mero visor da
> calculadora, que normalmente tem precisão de até 8 casas decimais.
>
> Att,
>
> Caio Costa
>
> Em qua, 31 de jul de 2019 às 18:43, Pedro José 
> escreveu:
>
>> Boa noite!
>> Não consegui perceber como vocês chegaram ao valor.
>> Com respeito, Cláudio, o enunciado fala em número que a mesma coisa
>> que valor. O número é a ideia e não a representação, portanto 
>> 1,0 =
>> 1 = I (representação romana) = 0,
>> Mas se puderem me ajudar e detalhar melhor como dá 1. Agradeço.
>>
>> Saudações,
>> PJMS
>>
>>
>> Em qua, 31 de jul de 2019 às 12:31, Claudio Buffara <
>> claudio.buff...@gmail.com> escreveu:
>>
>>> A questão não pede o valor de x(2013) (supondo que x(0) = 2)
>>> A questão pode o número que aparecerá no visor da calculadora.
>>> Neste caso, será 1,0 (numa calculadora com 9 casas decimais
>>> após a vírgula).
>>>
>>>
>>> Enviado do meu iPhone
>>>
>>> Em 31 de jul de 2019, à(s) 10:50, Rodrigo Ângelo <
>>> drigo.ang...@gmail.com> escreveu:
>>>
>>> Parece muito o método de Newton-Raphson pra encontrar zero de
>>> função, nesse caso, começando em 2 acho que converge pra raíz 
>>> positiva
>>> de x - 1/x que é 1
>>>
>>> Atenciosamente,
>>> Rodrigo de Castro Ângelo
>>>
>>>
>>> Em qua, 31 de jul de 2019 Ã s 09:08, Carlos Monteiro <
>>> cacacarlosalberto1...@gmail.com> escreveu:
>>>
 Luca tem uma calculadora com um único botão. Se um número x
 está na tela da calculadora e apertamos seu único botão, o número 
 x é
 substituído pelo número (2x)/(x^2 + 1). Dado que, inicialmente, o
 número 2 está na tela da calculadora, qual número aparecerá após
 apertarmos 2013 vezes seu botão.

 --
 Esta mensagem foi verificada pelo sistema de antivírus e
 acredita-se estar livre de perigo.
>>>
>>>
>>> --
>>> Esta mensagem foi verificada pelo sistema de antivírus e
>>> acredita-se estar livre de perigo.
>>>
>>>
>>> --
>>> Esta mensagem foi verificada pelo sistema de antivírus e
>>> acredita-se estar livre de perigo.
>>>
>>
>> --
>> Esta mensagem foi verificada pelo sistema de antivírus e
>> acredita-se estar livre de perigo.
>
>
> --
> Esta mensagem foi verificada pelo sistema de antivírus e
> acredita-se estar livre de perigo.


>>> --
>>> Esta mensagem foi verificada pelo sistema de antivírus e
>>> acredita-se estar livre de perigo.
>>
>>
>> --
>> Esta mensagem foi verificada pelo sistema de antivírus e
>> acredita-se estar livre de perigo.
>
>
> --
> Esta mensagem foi verificada pelo sistema de antivírus e
> acredita-se estar livre de perigo.

-- 
Esta mensagem foi 

[obm-l] Re: [obm-l] Re: [obm-l] Re: [obm-l] Re: [obm-l] Re: [obm-l] Re: [obm-l] Recorrência

2019-08-01 Por tôpico Caio Costa
Seria G(n+1) = [G(1)]^(2^n)?

On Wed, Jul 31, 2019, 9:24 PM Arthur Queiroz  wrote:

> Complementando, dá pra achar o termo geral assim:
> N(n+1) = 2*N(n)^2 + 2*N(n)
> Multiplicando os dois lados por dois e adicionando um:
> 2*N(n+1) + 1= 4*N(n)^2+4*N(n)+1
> Fatorando o lado direito:
> 2*N(n+1) + 1 = (2*N(n)+1)^2
> Agora, sendo G(n) = 2*N(n)+1, teremos que:
> G(n+1) = G(n)^2 = ((G(n-1)^2)^2 = ... = G(1)^(2^(n-1))
> Só que G(1) = 2*N(1)+1 = 2*4+1 = 9 = 3^2, logo G(n+1) = G(1)^(2^(n-1)) =
> (3^2)^(2^(n-1)) = 3^(2^n)
> Voltando, N(n), pela equação anterior, é igual a (G(n)-1)/2
> Logo, N(n) = (3^(2^n)-1)/2
> Por fim, a resposta do problema é N(2013)/(N(2013)+1) =
> (3^(2^2013)-1)/(3^(2^2013) + 1)
>
> Em qua, 31 de jul de 2019 às 20:20, Claudio Buffara <
> claudio.buff...@gmail.com> escreveu:
>
>> x(0) = 2 ==> x(1) = 4/5 ==> x(2) = 40/41 ==> x(3) = 3800/3801
>>
>> Em geral, se x(n) = a/b (a e b inteiros primos entre si) ==> x(n+1) =
>> 2ab/(a^2+b^2) < 1.
>> Além disso, olhando os primeiros termos, parece que a^2 + b^2 = 2ab + 1
>> ==> (a - b)^2 = 1 ==> b = a + 1
>> E, de fato, se x(n) = a/(a+1), então x(n+1) = 2a(a+1)/(2a^2+2a+1)
>>
>> Assim, a sequência de numeradores será:
>> N(1) = 4,
>> N(2) = 2*4*(4+1) = 40
>> N(3) = 2*40*(40+1) = 3800
>> ...
>> N(n+1) = 2*N(n)*(N(n)+1)
>>
>> De bate pronto não vejo uma fórmula fechada pra esta recorrência, mas o
>> número desejado é N(2013)/(N(2013)+1), e N(2013) é um inteiro gigantesco.
>>
>>
>>
>> On Wed, Jul 31, 2019 at 7:59 PM Claudio Buffara <
>> claudio.buff...@gmail.com> wrote:
>>
>>> Exatamente isso!
>>>
>>> On Wed, Jul 31, 2019 at 7:38 PM Caio Costa  wrote:
>>>
 não vai dar 1, mas vai dar um número muito próximo de 1 (valor exato).
 O que eles estão dizendo (pelo que entendi) é que a diferença para 1 é tão
 pequena, desprezível, que não será percebida pelo mero visor da
 calculadora, que normalmente tem precisão de até 8 casas decimais.

 Att,

 Caio Costa

 Em qua, 31 de jul de 2019 às 18:43, Pedro José 
 escreveu:

> Boa noite!
> Não consegui perceber como vocês chegaram ao valor.
> Com respeito, Cláudio, o enunciado fala em número que a mesma coisa
> que valor. O número é a ideia e não a representação, portanto 1,0 
> =
> 1 = I (representação romana) = 0,
> Mas se puderem me ajudar e detalhar melhor como dá 1. Agradeço.
>
> Saudações,
> PJMS
>
>
> Em qua, 31 de jul de 2019 às 12:31, Claudio Buffara <
> claudio.buff...@gmail.com> escreveu:
>
>> A questão não pede o valor de x(2013) (supondo que x(0) = 2)
>> A questão pode o número que aparecerá no visor da calculadora.
>> Neste caso, será 1,0 (numa calculadora com 9 casas decimais
>> após a vírgula).
>>
>>
>> Enviado do meu iPhone
>>
>> Em 31 de jul de 2019, à(s) 10:50, Rodrigo Ângelo <
>> drigo.ang...@gmail.com> escreveu:
>>
>> Parece muito o método de Newton-Raphson pra encontrar zero de
>> função, nesse caso, começando em 2 acho que converge pra raíz 
>> positiva
>> de x - 1/x que é 1
>>
>> Atenciosamente,
>> Rodrigo de Castro Ângelo
>>
>>
>> Em qua, 31 de jul de 2019 Ã s 09:08, Carlos Monteiro <
>> cacacarlosalberto1...@gmail.com> escreveu:
>>
>>> Luca tem uma calculadora com um único botão. Se um número x está
>>> na tela da calculadora e apertamos seu único botão, o número x é
>>> substituído pelo número (2x)/(x^2 + 1). Dado que, inicialmente, o
>>> número 2 está na tela da calculadora, qual número aparecerá após
>>> apertarmos 2013 vezes seu botão.
>>>
>>> --
>>> Esta mensagem foi verificada pelo sistema de antivírus e
>>> acredita-se estar livre de perigo.
>>
>>
>> --
>> Esta mensagem foi verificada pelo sistema de antivírus e
>> acredita-se estar livre de perigo.
>>
>>
>> --
>> Esta mensagem foi verificada pelo sistema de antivírus e
>> acredita-se estar livre de perigo.
>>
>
> --
> Esta mensagem foi verificada pelo sistema de antivírus e
> acredita-se estar livre de perigo.


 --
 Esta mensagem foi verificada pelo sistema de antivírus e
 acredita-se estar livre de perigo.
>>>
>>>
>> --
>> Esta mensagem foi verificada pelo sistema de antivírus e
>> acredita-se estar livre de perigo.
>
>
> --
> Esta mensagem foi verificada pelo sistema de antivírus e
> acredita-se estar livre de perigo.

-- 
Esta mensagem foi verificada pelo sistema de antiv�rus e
 acredita-se estar livre de perigo.



[obm-l] Re: [obm-l] Re: [obm-l] Re: [obm-l] Re: [obm-l] Re: [obm-l] Recorrência

2019-07-31 Por tôpico Arthur Queiroz
Complementando, dá pra achar o termo geral assim:
N(n+1) = 2*N(n)^2 + 2*N(n)
Multiplicando os dois lados por dois e adicionando um:
2*N(n+1) + 1= 4*N(n)^2+4*N(n)+1
Fatorando o lado direito:
2*N(n+1) + 1 = (2*N(n)+1)^2
Agora, sendo G(n) = 2*N(n)+1, teremos que:
G(n+1) = G(n)^2 = ((G(n-1)^2)^2 = ... = G(1)^(2^(n-1))
Só que G(1) = 2*N(1)+1 = 2*4+1 = 9 = 3^2, logo G(n+1) = G(1)^(2^(n-1)) =
(3^2)^(2^(n-1)) = 3^(2^n)
Voltando, N(n), pela equação anterior, é igual a (G(n)-1)/2
Logo, N(n) = (3^(2^n)-1)/2
Por fim, a resposta do problema é N(2013)/(N(2013)+1) =
(3^(2^2013)-1)/(3^(2^2013) + 1)

Em qua, 31 de jul de 2019 às 20:20, Claudio Buffara <
claudio.buff...@gmail.com> escreveu:

> x(0) = 2 ==> x(1) = 4/5 ==> x(2) = 40/41 ==> x(3) = 3800/3801
>
> Em geral, se x(n) = a/b (a e b inteiros primos entre si) ==> x(n+1) =
> 2ab/(a^2+b^2) < 1.
> Além disso, olhando os primeiros termos, parece que a^2 + b^2 = 2ab + 1
> ==> (a - b)^2 = 1 ==> b = a + 1
> E, de fato, se x(n) = a/(a+1), então x(n+1) = 2a(a+1)/(2a^2+2a+1)
>
> Assim, a sequência de numeradores será:
> N(1) = 4,
> N(2) = 2*4*(4+1) = 40
> N(3) = 2*40*(40+1) = 3800
> ...
> N(n+1) = 2*N(n)*(N(n)+1)
>
> De bate pronto não vejo uma fórmula fechada pra esta recorrência, mas o
> número desejado é N(2013)/(N(2013)+1), e N(2013) é um inteiro gigantesco.
>
>
>
> On Wed, Jul 31, 2019 at 7:59 PM Claudio Buffara 
> wrote:
>
>> Exatamente isso!
>>
>> On Wed, Jul 31, 2019 at 7:38 PM Caio Costa  wrote:
>>
>>> não vai dar 1, mas vai dar um número muito próximo de 1 (valor exato). O
>>> que eles estão dizendo (pelo que entendi) é que a diferença para 1 é tão
>>> pequena, desprezível, que não será percebida pelo mero visor da
>>> calculadora, que normalmente tem precisão de até 8 casas decimais.
>>>
>>> Att,
>>>
>>> Caio Costa
>>>
>>> Em qua, 31 de jul de 2019 às 18:43, Pedro José 
>>> escreveu:
>>>
 Boa noite!
 Não consegui perceber como vocês chegaram ao valor.
 Com respeito, Cláudio, o enunciado fala em número que a mesma coisa que
 valor. O número é a ideia e não a representação, portanto 1,0 = 1 =
 I (representação romana) = 0,
 Mas se puderem me ajudar e detalhar melhor como dá 1. Agradeço.

 Saudações,
 PJMS


 Em qua, 31 de jul de 2019 às 12:31, Claudio Buffara <
 claudio.buff...@gmail.com> escreveu:

> A questão não pede o valor de x(2013) (supondo que x(0) = 2)
> A questão pode o número que aparecerá no visor da calculadora.
> Neste caso, será 1,0 (numa calculadora com 9 casas decimais
> após a vírgula).
>
>
> Enviado do meu iPhone
>
> Em 31 de jul de 2019, à(s) 10:50, Rodrigo Ângelo <
> drigo.ang...@gmail.com> escreveu:
>
> Parece muito o método de Newton-Raphson pra encontrar zero de
> função, nesse caso, começando em 2 acho que converge pra raíz positiva
> de x - 1/x que é 1
>
> Atenciosamente,
> Rodrigo de Castro Ângelo
>
>
> Em qua, 31 de jul de 2019 Ã s 09:08, Carlos Monteiro <
> cacacarlosalberto1...@gmail.com> escreveu:
>
>> Luca tem uma calculadora com um único botão. Se um número x está
>> na tela da calculadora e apertamos seu único botão, o número x é
>> substituído pelo número (2x)/(x^2 + 1). Dado que, inicialmente, o
>> número 2 está na tela da calculadora, qual número aparecerá após
>> apertarmos 2013 vezes seu botão.
>>
>> --
>> Esta mensagem foi verificada pelo sistema de antivírus e
>> acredita-se estar livre de perigo.
>
>
> --
> Esta mensagem foi verificada pelo sistema de antivírus e
> acredita-se estar livre de perigo.
>
>
> --
> Esta mensagem foi verificada pelo sistema de antivírus e
> acredita-se estar livre de perigo.
>

 --
 Esta mensagem foi verificada pelo sistema de antivírus e
 acredita-se estar livre de perigo.
>>>
>>>
>>> --
>>> Esta mensagem foi verificada pelo sistema de antivírus e
>>> acredita-se estar livre de perigo.
>>
>>
> --
> Esta mensagem foi verificada pelo sistema de antivírus e
> acredita-se estar livre de perigo.

-- 
Esta mensagem foi verificada pelo sistema de antiv�rus e
 acredita-se estar livre de perigo.



[obm-l] Re: [obm-l] Re: [obm-l] Re: [obm-l] Re: [obm-l] Recorrência

2019-07-31 Por tôpico Claudio Buffara
x(0) = 2 ==> x(1) = 4/5 ==> x(2) = 40/41 ==> x(3) = 3800/3801

Em geral, se x(n) = a/b (a e b inteiros primos entre si) ==> x(n+1) =
2ab/(a^2+b^2) < 1.
Além disso, olhando os primeiros termos, parece que a^2 + b^2 = 2ab + 1 ==>
(a - b)^2 = 1 ==> b = a + 1
E, de fato, se x(n) = a/(a+1), então x(n+1) = 2a(a+1)/(2a^2+2a+1)

Assim, a sequência de numeradores será:
N(1) = 4,
N(2) = 2*4*(4+1) = 40
N(3) = 2*40*(40+1) = 3800
...
N(n+1) = 2*N(n)*(N(n)+1)

De bate pronto não vejo uma fórmula fechada pra esta recorrência, mas o
número desejado é N(2013)/(N(2013)+1), e N(2013) é um inteiro gigantesco.



On Wed, Jul 31, 2019 at 7:59 PM Claudio Buffara 
wrote:

> Exatamente isso!
>
> On Wed, Jul 31, 2019 at 7:38 PM Caio Costa  wrote:
>
>> não vai dar 1, mas vai dar um número muito próximo de 1 (valor exato). O
>> que eles estão dizendo (pelo que entendi) é que a diferença para 1 é tão
>> pequena, desprezível, que não será percebida pelo mero visor da
>> calculadora, que normalmente tem precisão de até 8 casas decimais.
>>
>> Att,
>>
>> Caio Costa
>>
>> Em qua, 31 de jul de 2019 às 18:43, Pedro José 
>> escreveu:
>>
>>> Boa noite!
>>> Não consegui perceber como vocês chegaram ao valor.
>>> Com respeito, Cláudio, o enunciado fala em número que a mesma coisa que
>>> valor. O número é a ideia e não a representação, portanto 1,0 = 1 =
>>> I (representação romana) = 0,
>>> Mas se puderem me ajudar e detalhar melhor como dá 1. Agradeço.
>>>
>>> Saudações,
>>> PJMS
>>>
>>>
>>> Em qua, 31 de jul de 2019 às 12:31, Claudio Buffara <
>>> claudio.buff...@gmail.com> escreveu:
>>>
 A questão não pede o valor de x(2013) (supondo que x(0) = 2)
 A questão pode o número que aparecerá no visor da calculadora.
 Neste caso, será 1,0 (numa calculadora com 9 casas decimais
 após a vírgula).


 Enviado do meu iPhone

 Em 31 de jul de 2019, à(s) 10:50, Rodrigo Ângelo <
 drigo.ang...@gmail.com> escreveu:

 Parece muito o método de Newton-Raphson pra encontrar zero de
 função, nesse caso, começando em 2 acho que converge pra raíz positiva
 de x - 1/x que é 1

 Atenciosamente,
 Rodrigo de Castro Ângelo


 Em qua, 31 de jul de 2019 Ã s 09:08, Carlos Monteiro <
 cacacarlosalberto1...@gmail.com> escreveu:

> Luca tem uma calculadora com um único botão. Se um número x está
> na tela da calculadora e apertamos seu único botão, o número x é
> substituído pelo número (2x)/(x^2 + 1). Dado que, inicialmente, o
> número 2 está na tela da calculadora, qual número aparecerá após
> apertarmos 2013 vezes seu botão.
>
> --
> Esta mensagem foi verificada pelo sistema de antivírus e
> acredita-se estar livre de perigo.


 --
 Esta mensagem foi verificada pelo sistema de antivírus e
 acredita-se estar livre de perigo.


 --
 Esta mensagem foi verificada pelo sistema de antivírus e
 acredita-se estar livre de perigo.

>>>
>>> --
>>> Esta mensagem foi verificada pelo sistema de antivírus e
>>> acredita-se estar livre de perigo.
>>
>>
>> --
>> Esta mensagem foi verificada pelo sistema de antivírus e
>> acredita-se estar livre de perigo.
>
>

-- 
Esta mensagem foi verificada pelo sistema de antiv�rus e
 acredita-se estar livre de perigo.



[obm-l] Re: [obm-l] Re: [obm-l] Re: [obm-l] Re: [obm-l] Recorrência

2019-07-31 Por tôpico Claudio Buffara
Exatamente isso!

On Wed, Jul 31, 2019 at 7:38 PM Caio Costa  wrote:

> não vai dar 1, mas vai dar um número muito próximo de 1 (valor exato). O
> que eles estão dizendo (pelo que entendi) é que a diferença para 1 é tão
> pequena, desprezível, que não será percebida pelo mero visor da
> calculadora, que normalmente tem precisão de até 8 casas decimais.
>
> Att,
>
> Caio Costa
>
> Em qua, 31 de jul de 2019 às 18:43, Pedro José 
> escreveu:
>
>> Boa noite!
>> Não consegui perceber como vocês chegaram ao valor.
>> Com respeito, Cláudio, o enunciado fala em número que a mesma coisa que
>> valor. O número é a ideia e não a representação, portanto 1,0 = 1 =
>> I (representação romana) = 0,
>> Mas se puderem me ajudar e detalhar melhor como dá 1. Agradeço.
>>
>> Saudações,
>> PJMS
>>
>>
>> Em qua, 31 de jul de 2019 às 12:31, Claudio Buffara <
>> claudio.buff...@gmail.com> escreveu:
>>
>>> A questão não pede o valor de x(2013) (supondo que x(0) = 2)
>>> A questão pode o número que aparecerá no visor da calculadora.
>>> Neste caso, será 1,0 (numa calculadora com 9 casas decimais após
>>> a vírgula).
>>>
>>>
>>> Enviado do meu iPhone
>>>
>>> Em 31 de jul de 2019, à(s) 10:50, Rodrigo Ângelo 
>>> escreveu:
>>>
>>> Parece muito o método de Newton-Raphson pra encontrar zero de função,
>>> nesse caso, começando em 2 acho que converge pra raíz positiva de x - 1/x
>>> que é 1
>>>
>>> Atenciosamente,
>>> Rodrigo de Castro Ângelo
>>>
>>>
>>> Em qua, 31 de jul de 2019 Ã s 09:08, Carlos Monteiro <
>>> cacacarlosalberto1...@gmail.com> escreveu:
>>>
 Luca tem uma calculadora com um único botão. Se um número x está na
 tela da calculadora e apertamos seu único botão, o número x é
 substituído pelo número (2x)/(x^2 + 1). Dado que, inicialmente, o
 número 2 está na tela da calculadora, qual número aparecerá após
 apertarmos 2013 vezes seu botão.

 --
 Esta mensagem foi verificada pelo sistema de antivírus e
 acredita-se estar livre de perigo.
>>>
>>>
>>> --
>>> Esta mensagem foi verificada pelo sistema de antivírus e
>>> acredita-se estar livre de perigo.
>>>
>>>
>>> --
>>> Esta mensagem foi verificada pelo sistema de antivírus e
>>> acredita-se estar livre de perigo.
>>>
>>
>> --
>> Esta mensagem foi verificada pelo sistema de antivírus e
>> acredita-se estar livre de perigo.
>
>
> --
> Esta mensagem foi verificada pelo sistema de antivírus e
> acredita-se estar livre de perigo.

-- 
Esta mensagem foi verificada pelo sistema de antiv�rus e
 acredita-se estar livre de perigo.



[obm-l] Re: [obm-l] Re: [obm-l] Re: [obm-l] Re: [obm-l] Combinatória

2019-07-21 Por tôpico Anderson Torres
Em sex, 14 de jun de 2019 às 10:05, Caio Costa  escreveu:
>
> A resposta é o coeficiente de x^15 no polinômio de grau infinito 
> (1+x+x^2+x^3)^n, com n natural indo para infinito. Faz sentido tal afirmação?

Não faz não. Por que um natural indo ao infinito teria alguma coisa a ver aqui?

>
> Em sex, 14 de jun de 2019 às 08:34, Vinícius Raimundo 
>  escreveu:
>>
>> Obrigado
>>
>> Tinha pensado em recorrência, mas não achei a correta
>> Alguém conhece um material bom para o estudo deste assunto?
>>
>> Em qui, 13 de jun de 2019 às 18:41, Claudio Buffara 
>>  escreveu:
>>>
>>> Chame isso de a(15).
>>> Vale a recorrência a(n) = a(n-1) + a(n-2) + a(n-3),  com a(1) = 1, a(2) = 2 
>>> e a(3) = 4.
>>> Isso porque você pode chegar ao n-ésimo degrau a partir do (n-1)-ésimo, 
>>> (n-2)-ésimo ou (n-3)-ésimo degrau.
>>> E você pode chegar ao (n-1)-ésimo de a(n-1) maneiras, ao (n-2)-ésimo de 
>>> a(n-2) maneiras, e ao (n-3) ésimo de a(n-3) maneiras.
>>>
>>> Daí, com uma planilha...
>>> a(4) = 4+2+1 = 7
>>> a(5) = 7+4+2 = 13
>>> ...
>>> a(15) = 5768.
>>>
>>>
>>> On Thu, Jun 13, 2019 at 6:03 PM Vinícius Raimundo  
>>> wrote:

 Pedro tem que descer uma escada com 15 degraus. Porém, ele só pode descer 
 1, 2 ou 3 degraus de cada vez
 De quantas maneiras ele pode fazer isso?

 --
 Esta mensagem foi verificada pelo sistema de antivírus e
 acredita-se estar livre de perigo.
>>>
>>>
>>> --
>>> Esta mensagem foi verificada pelo sistema de antivírus e
>>> acredita-se estar livre de perigo.
>>
>>
>> --
>> Esta mensagem foi verificada pelo sistema de antivírus e
>> acredita-se estar livre de perigo.
>
>
> --
> Esta mensagem foi verificada pelo sistema de antivírus e
> acredita-se estar livre de perigo.

-- 
Esta mensagem foi verificada pelo sistema de antiv�rus e
 acredita-se estar livre de perigo.


=
Instru��es para entrar na lista, sair da lista e usar a lista em
http://www.mat.puc-rio.br/~obmlistas/obm-l.html
=


[obm-l] Re: [obm-l] Re: [obm-l] Re: [obm-l] Re: [obm-l] Re: [obm-l] teoria dos números curiosidade

2019-07-04 Por tôpico Israel Meireles Chrisostomo
muito obrigado!!!

Em qui, 4 de jul de 2019 às 09:13, Claudio Buffara <
claudio.buff...@gmail.com> escreveu:

> Considere o seguinte algoritmo:
> Dada a/b (acho q precisa ser entre 0 e 1), tome o menor n1 tal que 1/n1 <=
> a/b.
> Daí, tome o menor n2 tal que 1/n2 <= a/b - 1/n1.
> Daí tome o menor n3 tal que 1/n3 <= a/b - 1/n1 - 1/n2
> Etc...
> Esse processo eventualmente para (quando uma desigualdade <= se torna uma
> igualdade), com:
> a/b = 1/n1 + 1/n2 + 1/n3 + ... + 1/nk, para algum k.
> Resta saber se produz uma fração egípcia (ou seja, se n1 < n2 < n3 < ...)
> e se a fração egípcia resultante é a menor possível.
> Vou pensar melhor é tentar simular alguns casos numa planilha.
>
>
> Enviado do meu iPhone
>
> Em 3 de jul de 2019, à(s) 22:11, Bernardo Freitas Paulo da Costa <
> bernardo...@gmail.com> escreveu:
>
> > On Wed, Jul 3, 2019 at 8:34 PM Claudio Buffara
> >  wrote:
> >> Infinitas.
> >> Basta usar recursivamente a relação  1/n = 1/(n+1) + 1/(n(n+1)), que
> cada vez você obtém uma representação mais longa.
> >> 1/2 = 1/3 + 1/6 = 1/3 + 1/7 + 1/42 = 1/3 + 1/7 + 1/43 + 1/1806 = ...
> >
> > Mais difícil, talvez, seria calcular qual o menor número de termos
> > necessários para representar p/q :)  Será que isso é NP completo?
> >
> > Abraços,
> > --
> > Bernardo Freitas Paulo da Costa
> >
> > --
> > Esta mensagem foi verificada pelo sistema de antivírus e
> > acredita-se estar livre de perigo.
> >
> >
> > =
> > Instruções para entrar na lista, sair da lista e usar a lista em
> > http://www.mat.puc-rio.br/~obmlistas/obm-l.html
> > =
>
> --
> Esta mensagem foi verificada pelo sistema de antivírus e
>  acredita-se estar livre de perigo.
>
>
> =
> Instru�ões para entrar na lista, sair da lista e usar a lista em
> http://www.mat.puc-rio.br/~obmlistas/obm-l.html
> =
>


-- 
Israel Meireles Chrisostomo

-- 
Esta mensagem foi verificada pelo sistema de antiv�rus e
 acredita-se estar livre de perigo.



Re: [obm-l] Re: [obm-l] Re: [obm-l] Re: [obm-l] Re: [obm-l] teoria dos números curiosidade

2019-07-04 Por tôpico Claudio Buffara
Considere o seguinte algoritmo:
Dada a/b (acho q precisa ser entre 0 e 1), tome o menor n1 tal que 1/n1 <= a/b.
Daí, tome o menor n2 tal que 1/n2 <= a/b - 1/n1.
Daí tome o menor n3 tal que 1/n3 <= a/b - 1/n1 - 1/n2
Etc...
Esse processo eventualmente para (quando uma desigualdade <= se torna uma 
igualdade), com:
a/b = 1/n1 + 1/n2 + 1/n3 + ... + 1/nk, para algum k.
Resta saber se produz uma fração egípcia (ou seja, se n1 < n2 < n3 < ...) e se 
a fração egípcia resultante é a menor possível.
Vou pensar melhor é tentar simular alguns casos numa planilha.


Enviado do meu iPhone

Em 3 de jul de 2019, à(s) 22:11, Bernardo Freitas Paulo da Costa 
 escreveu:

> On Wed, Jul 3, 2019 at 8:34 PM Claudio Buffara
>  wrote:
>> Infinitas.
>> Basta usar recursivamente a relação  1/n = 1/(n+1) + 1/(n(n+1)), que cada 
>> vez você obtém uma representação mais longa.
>> 1/2 = 1/3 + 1/6 = 1/3 + 1/7 + 1/42 = 1/3 + 1/7 + 1/43 + 1/1806 = ...
> 
> Mais difícil, talvez, seria calcular qual o menor número de termos
> necessários para representar p/q :)  Será que isso é NP completo?
> 
> Abraços,
> --
> Bernardo Freitas Paulo da Costa
> 
> -- 
> Esta mensagem foi verificada pelo sistema de antivírus e
> acredita-se estar livre de perigo.
> 
> 
> =
> Instruções para entrar na lista, sair da lista e usar a lista em
> http://www.mat.puc-rio.br/~obmlistas/obm-l.html
> =

-- 
Esta mensagem foi verificada pelo sistema de antiv�rus e
 acredita-se estar livre de perigo.


=
Instru��es para entrar na lista, sair da lista e usar a lista em
http://www.mat.puc-rio.br/~obmlistas/obm-l.html
=


[obm-l] Re: [obm-l] Re: [obm-l] Re: [obm-l] Re: [obm-l] teoria dos números curiosidade

2019-07-03 Por tôpico Bernardo Freitas Paulo da Costa
On Wed, Jul 3, 2019 at 8:34 PM Claudio Buffara
 wrote:
> Infinitas.
> Basta usar recursivamente a relação  1/n = 1/(n+1) + 1/(n(n+1)), que cada vez 
> você obtém uma representação mais longa.
> 1/2 = 1/3 + 1/6 = 1/3 + 1/7 + 1/42 = 1/3 + 1/7 + 1/43 + 1/1806 = ...

Mais difícil, talvez, seria calcular qual o menor número de termos
necessários para representar p/q :)  Será que isso é NP completo?

Abraços,
--
Bernardo Freitas Paulo da Costa

-- 
Esta mensagem foi verificada pelo sistema de antiv�rus e
 acredita-se estar livre de perigo.


=
Instru��es para entrar na lista, sair da lista e usar a lista em
http://www.mat.puc-rio.br/~obmlistas/obm-l.html
=


[obm-l] Re: [obm-l] Re: [obm-l] Re: [obm-l] Re: [obm-l] Re: teoria dos números

2019-07-01 Por tôpico Israel Meireles Chrisostomo
7 /4=(8-1) /4=2-1 /4  desculpem-me, troque natural por inteiro e sigam o
enunciado, desde já agradeço muito pela sua atenção


Livre
de vírus. www.avast.com
.
<#DAB4FAD8-2DD7-40BB-A1B8-4E2AA1F9FDF2>

Em seg, 1 de jul de 2019 às 11:59, Rodrigo Ângelo 
escreveu:

> Olá,
>
> Não consegui escrever 7/4 na forma k + 1/q, com k e q naturais.
>
> Atenciosamente,
> Rodrigo de Castro Ângelo
>
>
> Em dom, 30 de jun de 2019 às 20:41, Israel Meireles Chrisostomo <
> israelmchrisost...@gmail.com> escreveu:
>
>> Desde já agradeço
>>
>>
>> 
>>  Livre
>> de vírus. www.avg.com
>> .
>>
>> <#m_7250998230468987111_m_-3581101848330391677_DAB4FAD8-2DD7-40BB-A1B8-4E2AA1F9FDF2>
>>
>> Em dom, 30 de jun de 2019 às 18:54, Pedro José 
>> escreveu:
>>
>>> Boa noite!
>>>
>>> Continuo achando que não vele sempre.
>>>
>>> seja s/t um racional em sua forma reduzida com s e t inteiros, ou seja,
>>> (s,t)=1 e mais com t primo.
>>> s/t= k+1/q, com k e q naturais.
>>>
>>> sq -kqt= t posso obter uma equação em s e k com xs - yk = t. Por Bézout
>>> (x,y) | t mas x=q e y=kq ==> (x,y)=q e q|t. Como q é primo, q =1 ou q =t.
>>>
>>> Se q=1 temos que s/t = k+1 implicando que t|s, absurdo pois (s,t)=1 por
>>> hipóteste.
>>>
>>> Se q=t temos que (s-1)/t = k e não será atendido sempre. E.g, falha para
>>> s=5 e t= 3. E falha para quando é menor que 1,e;g, s=2 e t = 3 1/3=k, com k
>>> inteiro.
>>>
>>> Mas novamente aguardo por alguém com mais embasamento, para confirmar.
>>>
>>> Saudações,
>>> PJMS.
>>>
>>>
>>>
>>>
>>>
>>>
>>> Em dom, 30 de jun de 2019 às 16:25, Israel Meireles Chrisostomo <
>>> israelmchrisost...@gmail.com> escreveu:
>>>
 E tmbm toda fração racional inexata(cuja divisão não seja exata) menor
 do que 1 pode ser escrito na forma  k-1/q  onde k e q são naturais?



 
  Livre
 de vírus. www.avast.com
 .

 <#m_7250998230468987111_m_-3581101848330391677_m_-2724749205857227574_m_-787305250869872_DAB4FAD8-2DD7-40BB-A1B8-4E2AA1F9FDF2>

 Em dom, 30 de jun de 2019 às 15:11, Israel Meireles Chrisostomo <
 israelmchrisost...@gmail.com> escreveu:

> Toda fração racional inexata(cuja divisão não seja exata) mair do que
> 1 pode ser escrito na forma  k+1/q  onde k e q são naturais?
>
> --
> Israel Meireles Chrisostomo
>
>
> 
>  Livre
> de vírus. www.avast.com
> .
>
> <#m_7250998230468987111_m_-3581101848330391677_m_-2724749205857227574_m_-787305250869872_m_6303873557306665532_DAB4FAD8-2DD7-40BB-A1B8-4E2AA1F9FDF2>
>


 --
 Israel Meireles Chrisostomo

 --
 Esta mensagem foi verificada pelo sistema de antivírus e
 acredita-se estar livre de perigo.
>>>
>>>
>>> --
>>> Esta mensagem foi verificada pelo sistema de antivírus e
>>> acredita-se estar livre de perigo.
>>
>>
>>
>> --
>> Israel Meireles Chrisostomo
>>
>> --
>> Esta mensagem foi verificada pelo sistema de antivírus e
>> acredita-se estar livre de perigo.
>
>
> --
> Esta mensagem foi verificada pelo sistema de antivírus e
> acredita-se estar livre de perigo.



-- 
Israel Meireles Chrisostomo

-- 
Esta mensagem foi verificada pelo sistema de antiv�rus e
 acredita-se estar livre de perigo.



[obm-l] Re: [obm-l] Re: [obm-l] Re: [obm-l] Re: teoria dos números

2019-07-01 Por tôpico Rodrigo Ângelo
Olá,

Não consegui escrever 7/4 na forma k + 1/q, com k e q naturais.

Atenciosamente,
Rodrigo de Castro Ângelo


Em dom, 30 de jun de 2019 às 20:41, Israel Meireles Chrisostomo <
israelmchrisost...@gmail.com> escreveu:

> Desde já agradeço
>
>
> 
>  Livre
> de vírus. www.avg.com
> .
> <#m_-3581101848330391677_DAB4FAD8-2DD7-40BB-A1B8-4E2AA1F9FDF2>
>
> Em dom, 30 de jun de 2019 às 18:54, Pedro José 
> escreveu:
>
>> Boa noite!
>>
>> Continuo achando que não vele sempre.
>>
>> seja s/t um racional em sua forma reduzida com s e t inteiros, ou seja,
>> (s,t)=1 e mais com t primo.
>> s/t= k+1/q, com k e q naturais.
>>
>> sq -kqt= t posso obter uma equação em s e k com xs - yk = t. Por Bézout
>> (x,y) | t mas x=q e y=kq ==> (x,y)=q e q|t. Como q é primo, q =1 ou q =t.
>>
>> Se q=1 temos que s/t = k+1 implicando que t|s, absurdo pois (s,t)=1 por
>> hipóteste.
>>
>> Se q=t temos que (s-1)/t = k e não será atendido sempre. E.g, falha para
>> s=5 e t= 3. E falha para quando é menor que 1,e;g, s=2 e t = 3 1/3=k, com k
>> inteiro.
>>
>> Mas novamente aguardo por alguém com mais embasamento, para confirmar.
>>
>> Saudações,
>> PJMS.
>>
>>
>>
>>
>>
>>
>> Em dom, 30 de jun de 2019 às 16:25, Israel Meireles Chrisostomo <
>> israelmchrisost...@gmail.com> escreveu:
>>
>>> E tmbm toda fração racional inexata(cuja divisão não seja exata) menor
>>> do que 1 pode ser escrito na forma  k-1/q  onde k e q são naturais?
>>>
>>>
>>>
>>> 
>>>  Livre
>>> de vírus. www.avast.com
>>> .
>>>
>>> <#m_-3581101848330391677_m_-2724749205857227574_m_-787305250869872_DAB4FAD8-2DD7-40BB-A1B8-4E2AA1F9FDF2>
>>>
>>> Em dom, 30 de jun de 2019 às 15:11, Israel Meireles Chrisostomo <
>>> israelmchrisost...@gmail.com> escreveu:
>>>
 Toda fração racional inexata(cuja divisão não seja exata) mair do que 1
 pode ser escrito na forma  k+1/q  onde k e q são naturais?

 --
 Israel Meireles Chrisostomo


 
  Livre
 de vírus. www.avast.com
 .

 <#m_-3581101848330391677_m_-2724749205857227574_m_-787305250869872_m_6303873557306665532_DAB4FAD8-2DD7-40BB-A1B8-4E2AA1F9FDF2>

>>>
>>>
>>> --
>>> Israel Meireles Chrisostomo
>>>
>>> --
>>> Esta mensagem foi verificada pelo sistema de antivírus e
>>> acredita-se estar livre de perigo.
>>
>>
>> --
>> Esta mensagem foi verificada pelo sistema de antivírus e
>> acredita-se estar livre de perigo.
>
>
>
> --
> Israel Meireles Chrisostomo
>
> --
> Esta mensagem foi verificada pelo sistema de antivírus e
> acredita-se estar livre de perigo.

-- 
Esta mensagem foi verificada pelo sistema de antiv�rus e
 acredita-se estar livre de perigo.



[obm-l] Re: [obm-l] Re: [obm-l] Re: [obm-l] Re: [obm-l] Re: [obm-l] Pontuação de Campeonato

2019-06-07 Por tôpico Mauricio de Araujo
entendi, obrigado!!

Att,
__
Mauricio de Araujo



Em sex, 7 de jun de 2019 às 10:53, Ralph Teixeira 
escreveu:

> Pois é, por serem 4 rebaixados, a chave é olhar com carinho para o 17o
> time, que é o primeiro rebaixado. Por isso tem que olhar 17 times naquela
> conta!
>
> Por exemplo, o pior caso possível, em que seu time vai bem pra caramba mas
> rebaixa assim mesmo, é o caso em que tem 17 times bons (incluindo o seu) e
> 3 times horrorosos, que dão seus pontos para os outros 17 -- vide o exemplo
> que explicitei agora há pouco.
>
> Abraço, Ralph.
>
> On Fri, Jun 7, 2019 at 9:44 AM Mauricio de Araujo <
> mauricio.de.ara...@gmail.com> wrote:
>
>> Não seria, 16  ao invés de 17 a ser considerado nas contas? Afinal "4"
>> times descem...
>>
>> Att,
>> __
>> Mauricio de Araujo
>> 
>>
>>
>> Em sex, 7 de jun de 2019 às 07:44, Jeferson Almir <
>> jefersonram...@gmail.com> escreveu:
>>
>>> Valeu Ralph !!
>>> Só pra terminar eu precisaria exibir uma tabela que 33 pontos não é
>>> suficiente??? Abraço.
>>>
>>> Em sex, 7 de jun de 2019 às 00:22, Ralph Teixeira 
>>> escreveu:
>>>
 RESPOSTA: 34 pontos.

 Quando o campeonato termina, os 17 melhores times jogaram 17x16/2=17x8
 partidas entre si, mais 17x3 partidas com os ultimos 3 times. Assim, esses
 17 "melhores" times tem acesso a, no maximo, 17x11x3 pontos, ou seja, 33
 pontos cada um na media (no maximo!).

 Isso significa que, se voce faz 34 pontos, com certeza escapa do
 rebaixamento (eh impossivel que os 17 primeiros times tenham >=34 pontos
 cada).

 Isto dito, com MUITO azar pode ser que algum time com 33 pontos seja
 rebaixado. Basta imaginar que os 17 primeiros ganham TODOS os jogos dos 3
 outros, e entre si cada um dos 17 primeiros ganha 8 e perde 8 (para mostrar
 categoricamente que isso eh possivel, ponha os 17 times num circulo e
 imagine que cada time ganha dos 8 mais proximos no sentido horario, e perde
 dos outros 8). Entao voce tem uma tabela onde 17 times terminam com (cada
 um) 11 vitorias e 8 derrotas, ou seja, 33 pontos cada, e um deles seria
 rebaixado (em algum criterio de desempate).

 Abraco, Ralph.

 On Thu, Jun 6, 2019 at 10:44 PM Jeferson Almir <
 jefersonram...@gmail.com> wrote:

> Qual a pontuação mínima de um campeonato com 20 times  para que um
> time fique livre do rebaixamento( 4 últimos times  descem ) sabendo
> que cada time joga com todos os outros somente uma única vez??. E que
> vitória vale 3 pontos empate vale 1 ponto.
>
> --
> Esta mensagem foi verificada pelo sistema de antivírus e
> acredita-se estar livre de perigo.


 --
 Esta mensagem foi verificada pelo sistema de antivírus e
 acredita-se estar livre de perigo.
>>>
>>>
>>> --
>>> Esta mensagem foi verificada pelo sistema de antivírus e
>>> acredita-se estar livre de perigo.
>>
>>
>> --
>> Esta mensagem foi verificada pelo sistema de antivírus e
>> acredita-se estar livre de perigo.
>
>
> --
> Esta mensagem foi verificada pelo sistema de antivírus e
> acredita-se estar livre de perigo.

-- 
Esta mensagem foi verificada pelo sistema de antiv�rus e
 acredita-se estar livre de perigo.



[obm-l] Re: [obm-l] Re: [obm-l] Re: [obm-l] Re: [obm-l] Pontuação de Campeonato

2019-06-07 Por tôpico Ralph Teixeira
Pois é, por serem 4 rebaixados, a chave é olhar com carinho para o 17o
time, que é o primeiro rebaixado. Por isso tem que olhar 17 times naquela
conta!

Por exemplo, o pior caso possível, em que seu time vai bem pra caramba mas
rebaixa assim mesmo, é o caso em que tem 17 times bons (incluindo o seu) e
3 times horrorosos, que dão seus pontos para os outros 17 -- vide o exemplo
que explicitei agora há pouco.

Abraço, Ralph.

On Fri, Jun 7, 2019 at 9:44 AM Mauricio de Araujo <
mauricio.de.ara...@gmail.com> wrote:

> Não seria, 16  ao invés de 17 a ser considerado nas contas? Afinal "4"
> times descem...
>
> Att,
> __
> Mauricio de Araujo
> 
>
>
> Em sex, 7 de jun de 2019 às 07:44, Jeferson Almir <
> jefersonram...@gmail.com> escreveu:
>
>> Valeu Ralph !!
>> Só pra terminar eu precisaria exibir uma tabela que 33 pontos não é
>> suficiente??? Abraço.
>>
>> Em sex, 7 de jun de 2019 às 00:22, Ralph Teixeira 
>> escreveu:
>>
>>> RESPOSTA: 34 pontos.
>>>
>>> Quando o campeonato termina, os 17 melhores times jogaram 17x16/2=17x8
>>> partidas entre si, mais 17x3 partidas com os ultimos 3 times. Assim, esses
>>> 17 "melhores" times tem acesso a, no maximo, 17x11x3 pontos, ou seja, 33
>>> pontos cada um na media (no maximo!).
>>>
>>> Isso significa que, se voce faz 34 pontos, com certeza escapa do
>>> rebaixamento (eh impossivel que os 17 primeiros times tenham >=34 pontos
>>> cada).
>>>
>>> Isto dito, com MUITO azar pode ser que algum time com 33 pontos seja
>>> rebaixado. Basta imaginar que os 17 primeiros ganham TODOS os jogos dos 3
>>> outros, e entre si cada um dos 17 primeiros ganha 8 e perde 8 (para mostrar
>>> categoricamente que isso eh possivel, ponha os 17 times num circulo e
>>> imagine que cada time ganha dos 8 mais proximos no sentido horario, e perde
>>> dos outros 8). Entao voce tem uma tabela onde 17 times terminam com (cada
>>> um) 11 vitorias e 8 derrotas, ou seja, 33 pontos cada, e um deles seria
>>> rebaixado (em algum criterio de desempate).
>>>
>>> Abraco, Ralph.
>>>
>>> On Thu, Jun 6, 2019 at 10:44 PM Jeferson Almir 
>>> wrote:
>>>
 Qual a pontuação mínima de um campeonato com 20 times  para que um time
 fique livre do rebaixamento( 4 últimos times  descem ) sabendo que
 cada time joga com todos os outros somente uma única vez??. E que vitória
 vale 3 pontos empate vale 1 ponto.

 --
 Esta mensagem foi verificada pelo sistema de antivírus e
 acredita-se estar livre de perigo.
>>>
>>>
>>> --
>>> Esta mensagem foi verificada pelo sistema de antivírus e
>>> acredita-se estar livre de perigo.
>>
>>
>> --
>> Esta mensagem foi verificada pelo sistema de antivírus e
>> acredita-se estar livre de perigo.
>
>
> --
> Esta mensagem foi verificada pelo sistema de antivírus e
> acredita-se estar livre de perigo.

-- 
Esta mensagem foi verificada pelo sistema de antiv�rus e
 acredita-se estar livre de perigo.



[obm-l] Re: [obm-l] Re: [obm-l] Re: [obm-l] Re: [obm-l] Re: [obm-l] Soluções inteiras

2019-05-18 Por tôpico matematica10complicada
Thank you 

Em sex, 17 de mai de 2019 19:47, Pedro José  escreveu:

> Boa noite!
> Corrigi de orelhada, devido a paridade e a solução (21,23), aue
> encontrara. Quando dispor de um tempo, tentarei compreender. Mas pelo visto
> é mais fácil apontar que existe uma infinidade de soluções, do que achá-las
> propriamente. Não se gera uma fórmula para as soluções. Se compreendi, pelo
> menos, um pouco da explicação.
>
> Grato,
> PJMS
>
> Em sex, 17 de mai de 2019 19:01, Ralph Teixeira  escreveu:
>
>> Oops, sim, eu errei, voce consertou, era y=6a+p e x=5a+p. Tambem poderia
>> ser y=6a-p e x=5a-p, mas entao x vai ser negativo, o que pode ser obtido
>> diretamente das solucoes positivas trocando sinais.
>>
>> Na pratica, a ideia eh a seguinte: tome (11+2raiz(30))^n para varios
>> valores de n.
>>
>> Por exemplo, para n=2, temos:
>> (11+2raiz(30))^2=241+44raiz(30)
>> Eu afirmo que p=241 e a=44 tambem servem -- confira que p^2-30a^2=1 de
>> novo!
>> Colocando isto na quadratica do y, voce acha y, e depois acha x:
>> y=6a+p=505 e x=y-a=461
>> (Confira que este cara serve! Tambem tem as solucoes trocando os sinais
>> de x e y, mas nao vou falar muito delas, vou me concentrar nas positivas,
>> as outras vem por tais trocas de sinal.)
>>
>> Para n=3:
>> (11+2raiz(30))^3=5291+966raiz(30). Entao p=5291, a=966 servem, levando a
>> y=6a+p=11087 e x=y-a=10121
>>
>> Para cada n, voce terah uma escolha de p e a, e portanto uma escolha de x
>> e y... Ou seja, o problema tem infinitas solucoes!
>>
>> (Sim, o metodo vao sempre gerar p=impar e a=par, entao todas as solucoes
>> serao x=5a+p=impar e y=6a+p=impar)
>>
>> As respostas que faltam -- (A) POR QUE isso gera solucoes? (B) Esta ideia
>> ACHA TODAS as solucoes (bom, com as devidas trocas de sinal que sempre
>> existem)?
>>
>> ---///---
>> (A) POR QUE gera solucoes?
>>
>> Lema: Seja m um numero natural positivo que NAO EH quadrado perfeito.
>> Considere a Equacao de Pell p^2-m.a^2=1 (normalmente o pessoal usa x e y,
>> mas vou usar p e a para ficar parecido com minha notacao ali em cima). Se
>> p=p0 e a=a0 eh uma solucao, entao p=pn e a=an tambem eh, onde pn e an sao
>> inteiros determinados pela formula
>> (p0+a0.raiz(m))^n=pn+an.raiz(m).
>>
>> Demonstracao: Fatorando, vem que (p0+a0.raiz(m)).(p0-a0.raiz(m))=1.
>>
>> Elevando os dois lados a potencia n, vem (p0+a0.raiz(m))^n .
>> (p0-a0.raiz(m))^n =1.
>>
>> Mas o primeiro fator do produto eh exatamente pn+raiz(m).an (pela nossa
>> definicao de an e pn), e nao eh dificil ver que, se m nao eh quadrado
>> perfeito, o segundo fator tem de ser exatamente o "conjugado" pn-raiz(m).an
>> (abra o binomio de Newton se necessario para enxergar isso).
>>
>> Portanto, temos (pn+an.raiz(m)).(pn-an.raiz(m))=1, ou seja pn^2-m.an^2=1
>> tambem!
>>
>> ---///---
>>
>> Repito, esse lema mostra que o processo GERA solucoes, mas falta mostrar
>> (B): que existe alguma especie de "solucao fundamental" que gera TODAS as
>> outras por este processo... Bom, a resposta eh SIM, esta solucao
>> "fundamental" existe, e eu **acho** que neste caso eh (11,2)... mas para
>> mostrar isso, veja o artigo da Eureka, acho que este E-mail ficou muito
>> comprdo... :D
>>
>> Abraco, Ralph.
>>
>> On Fri, May 17, 2019 at 6:05 PM Pedro José  wrote:
>>
>>> Boa tarde!
>>> Se fizer s=x^2 e t=y^2 temos 6s-5t=1; cuja solução é s=5a+1 e t=6a+1,
>>> com a >=0. Então, x e y não deveriam ser ímpares?
>>> As soluções que achei:
>>> (-1,-1);(-1,1);(1,-1) e (1,1) essa no lápis. para a=0
>>> (-21,-23);(-21,23);(21,-23) e (21,23) com auxílio do Excel para a=88.
>>>
>>> Não sei se há mais soluções. Porém creio que as soluções são em 2Z+1.
>>>
>>> Se fosse:
>>> y=6a+p
>>> x=5a+p
>>> (p,a)=(11,2) daria a solução (x,y) = (21,23)
>>>
>>> Não consegui alcançar seu pensamento. Mas creio que pela solução da
>>> equação diofantina, tanto x como y deveriam ser ímpares.
>>>
>>> Saudações,
>>> PJMS
>>>
>>>
>>>
>>> Em sex, 17 de mai de 2019 às 14:02, Ralph Teixeira 
>>> escreveu:
>>>
 Escreva x=y-a com a inteiro. Ficamos com y^2-12ay+6a^2-1=0.

 Pense nisso como uma quadrática em y. Para haver soluções inteiras, o
 discriminante tem que ser quadrado perfeito:

 D = 144a^2 -4 (6a^2-1) = 120a^2+4 = 4p^2 (tem que ser par, por isso já
 coloquei o 4)
 30a^2+1=p^2
 p^2-30a^2=1

 Isso é uma Equação de Pell, cuja teoria não é difícil, mas está bem
 além das congruências... Veja o artigo do Caminha na Eureka 7, por exemplo:
 https://www.obm.org.br/content/uploads/2017/01/eureka7.pdf

 Em suma, você acha uma solução fundamental (acho que é (p,a)=(11,2)
 neste caso) e gerar as outras olhando para
 (11+2raiz(30))^n  (para cada n=0,1,2,..., a parte inteira disso dá um
 possível p, o coeficiente de raiz(30) dá um possível a).

 Enfim, encontrados p e a, teremos:
 y=6a+-2p
 x=5a+-2p

 Ou seja, creio haver infinitas soluções!

 Abraço, Ralph.

 On Fri, May 17, 2019 at 7:25 AM 

[obm-l] Re: [obm-l] Re: [obm-l] Re: [obm-l] Re: [obm-l] Soluções inteiras

2019-05-17 Por tôpico Pedro José
Boa noite!
Corrigi de orelhada, devido a paridade e a solução (21,23), aue encontrara.
Quando dispor de um tempo, tentarei compreender. Mas pelo visto é mais
fácil apontar que existe uma infinidade de soluções, do que achá-las
propriamente. Não se gera uma fórmula para as soluções. Se compreendi, pelo
menos, um pouco da explicação.

Grato,
PJMS

Em sex, 17 de mai de 2019 19:01, Ralph Teixeira  Oops, sim, eu errei, voce consertou, era y=6a+p e x=5a+p. Tambem poderia
> ser y=6a-p e x=5a-p, mas entao x vai ser negativo, o que pode ser obtido
> diretamente das solucoes positivas trocando sinais.
>
> Na pratica, a ideia eh a seguinte: tome (11+2raiz(30))^n para varios
> valores de n.
>
> Por exemplo, para n=2, temos:
> (11+2raiz(30))^2=241+44raiz(30)
> Eu afirmo que p=241 e a=44 tambem servem -- confira que p^2-30a^2=1 de
> novo!
> Colocando isto na quadratica do y, voce acha y, e depois acha x:
> y=6a+p=505 e x=y-a=461
> (Confira que este cara serve! Tambem tem as solucoes trocando os sinais de
> x e y, mas nao vou falar muito delas, vou me concentrar nas positivas, as
> outras vem por tais trocas de sinal.)
>
> Para n=3:
> (11+2raiz(30))^3=5291+966raiz(30). Entao p=5291, a=966 servem, levando a
> y=6a+p=11087 e x=y-a=10121
>
> Para cada n, voce terah uma escolha de p e a, e portanto uma escolha de x
> e y... Ou seja, o problema tem infinitas solucoes!
>
> (Sim, o metodo vao sempre gerar p=impar e a=par, entao todas as solucoes
> serao x=5a+p=impar e y=6a+p=impar)
>
> As respostas que faltam -- (A) POR QUE isso gera solucoes? (B) Esta ideia
> ACHA TODAS as solucoes (bom, com as devidas trocas de sinal que sempre
> existem)?
>
> ---///---
> (A) POR QUE gera solucoes?
>
> Lema: Seja m um numero natural positivo que NAO EH quadrado perfeito.
> Considere a Equacao de Pell p^2-m.a^2=1 (normalmente o pessoal usa x e y,
> mas vou usar p e a para ficar parecido com minha notacao ali em cima). Se
> p=p0 e a=a0 eh uma solucao, entao p=pn e a=an tambem eh, onde pn e an sao
> inteiros determinados pela formula
> (p0+a0.raiz(m))^n=pn+an.raiz(m).
>
> Demonstracao: Fatorando, vem que (p0+a0.raiz(m)).(p0-a0.raiz(m))=1.
>
> Elevando os dois lados a potencia n, vem (p0+a0.raiz(m))^n .
> (p0-a0.raiz(m))^n =1.
>
> Mas o primeiro fator do produto eh exatamente pn+raiz(m).an (pela nossa
> definicao de an e pn), e nao eh dificil ver que, se m nao eh quadrado
> perfeito, o segundo fator tem de ser exatamente o "conjugado" pn-raiz(m).an
> (abra o binomio de Newton se necessario para enxergar isso).
>
> Portanto, temos (pn+an.raiz(m)).(pn-an.raiz(m))=1, ou seja pn^2-m.an^2=1
> tambem!
>
> ---///---
>
> Repito, esse lema mostra que o processo GERA solucoes, mas falta mostrar
> (B): que existe alguma especie de "solucao fundamental" que gera TODAS as
> outras por este processo... Bom, a resposta eh SIM, esta solucao
> "fundamental" existe, e eu **acho** que neste caso eh (11,2)... mas para
> mostrar isso, veja o artigo da Eureka, acho que este E-mail ficou muito
> comprdo... :D
>
> Abraco, Ralph.
>
> On Fri, May 17, 2019 at 6:05 PM Pedro José  wrote:
>
>> Boa tarde!
>> Se fizer s=x^2 e t=y^2 temos 6s-5t=1; cuja solução é s=5a+1 e t=6a+1, com
>> a >=0. Então, x e y não deveriam ser ímpares?
>> As soluções que achei:
>> (-1,-1);(-1,1);(1,-1) e (1,1) essa no lápis. para a=0
>> (-21,-23);(-21,23);(21,-23) e (21,23) com auxílio do Excel para a=88.
>>
>> Não sei se há mais soluções. Porém creio que as soluções são em 2Z+1.
>>
>> Se fosse:
>> y=6a+p
>> x=5a+p
>> (p,a)=(11,2) daria a solução (x,y) = (21,23)
>>
>> Não consegui alcançar seu pensamento. Mas creio que pela solução da
>> equação diofantina, tanto x como y deveriam ser ímpares.
>>
>> Saudações,
>> PJMS
>>
>>
>>
>> Em sex, 17 de mai de 2019 às 14:02, Ralph Teixeira 
>> escreveu:
>>
>>> Escreva x=y-a com a inteiro. Ficamos com y^2-12ay+6a^2-1=0.
>>>
>>> Pense nisso como uma quadrática em y. Para haver soluções inteiras, o
>>> discriminante tem que ser quadrado perfeito:
>>>
>>> D = 144a^2 -4 (6a^2-1) = 120a^2+4 = 4p^2 (tem que ser par, por isso já
>>> coloquei o 4)
>>> 30a^2+1=p^2
>>> p^2-30a^2=1
>>>
>>> Isso é uma Equação de Pell, cuja teoria não é difícil, mas está bem além
>>> das congruências... Veja o artigo do Caminha na Eureka 7, por exemplo:
>>> https://www.obm.org.br/content/uploads/2017/01/eureka7.pdf
>>>
>>> Em suma, você acha uma solução fundamental (acho que é (p,a)=(11,2)
>>> neste caso) e gerar as outras olhando para
>>> (11+2raiz(30))^n  (para cada n=0,1,2,..., a parte inteira disso dá um
>>> possível p, o coeficiente de raiz(30) dá um possível a).
>>>
>>> Enfim, encontrados p e a, teremos:
>>> y=6a+-2p
>>> x=5a+-2p
>>>
>>> Ou seja, creio haver infinitas soluções!
>>>
>>> Abraço, Ralph.
>>>
>>> On Fri, May 17, 2019 at 7:25 AM matematica10complicada <
>>> profdouglaso.del...@gmail.com> wrote:
>>>
 Olá meus caros, gostaria de uma ajuda sem usar congruência para
 resolver e achar todos os inteiros da equação
  6x^2-5y^2=1.


 Obrigado e grande 

[obm-l] Re: [obm-l] Re: [obm-l] Re: [obm-l] Re: [obm-l] Re: [obm-l] Re: [obm-l] Teoria dos números

2019-05-04 Por tôpico Pedro José
Bom dia!
Obrigado!
Encontrei uma demonstração, mas não tive bagavem para enrender. Vou ler as
publicações.

Saudações,
PJMS

Em sáb, 4 de mai de 2019 11:57, Anderson Torres <
torres.anderson...@gmail.com escreveu:

> Em seg, 29 de abr de 2019 às 16:38, Pedro José 
> escreveu:
> >
> > Boa tarde!
> > Pelo menos consegui descobrir que se um inteiro z >= não puder ser
> escrito da forma z=4^k (8m+7), com m,k >=0 e m,k inteiros então ele pode
> ser representado por uma soma de três parcelas, todas quadrados perfeitos.
> > Já a demonstração, não consegui compreender.
> >
>
> Essa é a parte chata. Mas tem paper pra caramba!
>
> https://en.wikipedia.org/wiki/Legendre%27s_three-square_theorem
> Legendre's three-square theorem - Wikipedia
> https://www.sciencedirect.com/science/article/pii/0022314X74900249
> A new proof of the three squares theorem - ScienceDirect
> https://brilliant.org/wiki/fermats-sum-of-two-squares-theorem/
> Sum of Squares Theorems | Brilliant Math & Science Wiki
>
> https://mathoverflow.net/questions/223939/proving-legendres-sum-of-3-squares-theorem-via-geometry-of-numbers
> nt.number theory - Proving Legendre's Sum of 3 Squares Theorem via
> Geometry of Numbers - MathOverflow
> https://core.ac.uk/download/pdf/82306476.pdf
> PII: 0022-314X(74)90024-9 - 82306476.pdf
>
> https://www.ams.org/journals/proc/1957-008-02/S0002-9939-1957-0085275-8/S0002-9939-1957-0085275-8.pdf
> S0002-9939-1957-0085275-8.pdf
> http://pollack.uga.edu/finding3squares-6.pdf
> finding3squares-6.pdf
> https://arxiv.org/pdf/0812.0540.pdf
> () - 0812.0540.pdf
>
>
>
> > Saudações,
> > PJMS
> >
> > Em seg, 29 de abr de 2019 às 14:14,  escreveu:
> >>
> >>
> >> Em 29 de abr de 2019 11:37, Pedro José  escreveu:
> >>
> >> Bom dia!
> >>
> >> Gostei desse problema. Fiz um montão de exemplos com números que não
> podem ser escritos como 4^n(8n+7) e todos puderam ser escritos como a soma
> de três quadrados.
> >> Vale para todos? Se sim, alguém poderia indicar uma demonstração?
> >>
> >> Saudações,
> >> PJMS
> >>
> >> Em dom, 7 de abr de 2019 às 16:16, Pedro José 
> escreveu:
> >>
> >> Boa tarde!
> >> Fiquei na dúvida se algoritmo valia para demonstração. Mas salvo engano
> para demonstração de quais números aceitam raízes primitivas usa-se
> algoritmo.
> >> Mas, agora com mais calma, poderia ter usado indução.
> >> 1) Foi provado que não vale para n=0.
> >> 2) Supondo que não vale para n, não valeria para n+1, por absurdo.
> Pois, se valesse, teria que valer para n.
> >> Creio que teria ficado mais elegante.
> >>
> >> Saudações,
> >> PJMS
> >>
> >>
> >> Em dom, 7 de abr de 2019 às 07:41, matematica10complicada <
> profdouglaso.del...@gmail.com> escreveu:
> >>
> >> Obrigado irmão. Está correto sim.
> >> Douglas O.
> >>
> >> Em qui, 4 de abr de 2019 às 19:44, Pedro José 
> escreveu:
> >>
> >> Boa noite!
> >> Estou mal, mesmo. Ao invés de nenhum li qualquer. Tinha simulado dois,
> três, quatro e deram fora, já iria questionar.
> >> Mas vamos lá:
> >> 0^2 = 0 mod8; 1^2 = 1 mod8; 2^2 = 4 mod8 3^2= 1 mod8; 4^2 = 0 mod 8;
> 5^2 = 1 mod 8 6^2 = 4 mod 8 e 7^2 = 1 mod8;
> >> Portanto o quadrado de um número, ou dá 0 ou da 1 ou 4 na equivalência
> mod8.
> >>
> >> Caso n=0 ==> x=8k+7= 7 mod8. Como mod conserva a soma, não há como
> somar 3 parcelas do conjunto, mesmo com repetição, {0,1,4} e obter 7. Então
> n>0
> >>
> >> Para n>0
> >> x = 4^n*(8K+7) ==> x pertence a 2 |N seja x = a^2 + b^2 + c^2 com a, b,
> c pertencentes a |N - {0}. teríamos que ter a,b,c pares ou um deles par e
> dois ímpares.
> >> mas 4 | x ==> x= 0 mod4. Mas se w pertence a 2|N + 1 ==> w^2 = 1 mod4.
> e se y pertence a 2 |N ==> y^2 = 0 mod 4. Como temos dois ímpares e um par
> e como a soma se conserva temos que x = 2 mod4, absurdo. Portanto só sobra
> a, b, c pares Se a,b,c pares podemos escrevê-los como a= 2s; b=2t e c=2u
> com s,t,u naturais.
> >> x = a^2+b^2+c^2= 4(s^2+t^2+u^2) ==> x1 = 4^(n-1) * (8m+7) = s^2+t^2+u^2
> e vale o mesmo raciocínio de que s,t,u são pares e poderão ser escritos
> como s=2f; t=2g; u= 2h, com f, g, h naturais e seguir nesse algoritmo até
> que tenhamos xj=4^0(8m+7)= p^2+q^2+r^2, absurdo. Pois, já vimos que n= 0
> não atende.
> >>
> >> Espero estar correto.
> >>
> >> Saudações.
> >>
> >>
> >>
> >>
> >>
> >> Em qua, 3 de abr de 2019 às 15:36, matematica10complicada <
> profdouglaso.del...@gmail.com> escreveu:
> >>
> >> Mostre que nenhum número da forma (4^n)(8k+7) , com n e k naturais pode
> ser escrito como soma de 3 tres quadrados
> >>
> >> Douglas Oliveira
> >>
> >> --
> >> Esta mensagem foi verificada pelo sistema de antivírus e
> >> acredita-se estar livre de perigo.
> >>
> >>
> >> --
> >> Esta mensagem foi verificada pelo sistema de antivírus e
> >> acredita-se estar livre de perigo.
> >>
> >>
> >> --
> >> Esta mensagem foi verificada pelo sistema de antivírus e
> >> acredita-se estar livre de perigo.
> >>
> >>
> >> --
> >> Esta mensagem foi verificada pelo sistema de antiv�rus e
> >> acredita-se estar livre de perigo.
> >>
> >>
> >>
> >> 

[obm-l] Re: [obm-l] Re: [obm-l] Re: [obm-l] Re: [obm-l] Re: [obm-l] Teoria dos números

2019-05-04 Por tôpico Anderson Torres
Em seg, 29 de abr de 2019 às 16:38, Pedro José  escreveu:
>
> Boa tarde!
> Pelo menos consegui descobrir que se um inteiro z >= não puder ser escrito da 
> forma z=4^k (8m+7), com m,k >=0 e m,k inteiros então ele pode ser 
> representado por uma soma de três parcelas, todas quadrados perfeitos.
> Já a demonstração, não consegui compreender.
>

Essa é a parte chata. Mas tem paper pra caramba!

https://en.wikipedia.org/wiki/Legendre%27s_three-square_theorem
Legendre's three-square theorem - Wikipedia
https://www.sciencedirect.com/science/article/pii/0022314X74900249
A new proof of the three squares theorem - ScienceDirect
https://brilliant.org/wiki/fermats-sum-of-two-squares-theorem/
Sum of Squares Theorems | Brilliant Math & Science Wiki
https://mathoverflow.net/questions/223939/proving-legendres-sum-of-3-squares-theorem-via-geometry-of-numbers
nt.number theory - Proving Legendre's Sum of 3 Squares Theorem via
Geometry of Numbers - MathOverflow
https://core.ac.uk/download/pdf/82306476.pdf
PII: 0022-314X(74)90024-9 - 82306476.pdf
https://www.ams.org/journals/proc/1957-008-02/S0002-9939-1957-0085275-8/S0002-9939-1957-0085275-8.pdf
S0002-9939-1957-0085275-8.pdf
http://pollack.uga.edu/finding3squares-6.pdf
finding3squares-6.pdf
https://arxiv.org/pdf/0812.0540.pdf
() - 0812.0540.pdf



> Saudações,
> PJMS
>
> Em seg, 29 de abr de 2019 às 14:14,  escreveu:
>>
>>
>> Em 29 de abr de 2019 11:37, Pedro José  escreveu:
>>
>> Bom dia!
>>
>> Gostei desse problema. Fiz um montão de exemplos com números que não podem 
>> ser escritos como 4^n(8n+7) e todos puderam ser escritos como a soma de três 
>> quadrados.
>> Vale para todos? Se sim, alguém poderia indicar uma demonstração?
>>
>> Saudações,
>> PJMS
>>
>> Em dom, 7 de abr de 2019 às 16:16, Pedro José  escreveu:
>>
>> Boa tarde!
>> Fiquei na dúvida se algoritmo valia para demonstração. Mas salvo engano para 
>> demonstração de quais números aceitam raízes primitivas usa-se algoritmo.
>> Mas, agora com mais calma, poderia ter usado indução.
>> 1) Foi provado que não vale para n=0.
>> 2) Supondo que não vale para n, não valeria para n+1, por absurdo. Pois, se 
>> valesse, teria que valer para n.
>> Creio que teria ficado mais elegante.
>>
>> Saudações,
>> PJMS
>>
>>
>> Em dom, 7 de abr de 2019 às 07:41, matematica10complicada 
>>  escreveu:
>>
>> Obrigado irmão. Está correto sim.
>> Douglas O.
>>
>> Em qui, 4 de abr de 2019 às 19:44, Pedro José  escreveu:
>>
>> Boa noite!
>> Estou mal, mesmo. Ao invés de nenhum li qualquer. Tinha simulado dois, três, 
>> quatro e deram fora, já iria questionar.
>> Mas vamos lá:
>> 0^2 = 0 mod8; 1^2 = 1 mod8; 2^2 = 4 mod8 3^2= 1 mod8; 4^2 = 0 mod 8; 5^2 = 1 
>> mod 8 6^2 = 4 mod 8 e 7^2 = 1 mod8;
>> Portanto o quadrado de um número, ou dá 0 ou da 1 ou 4 na equivalência  mod8.
>>
>> Caso n=0 ==> x=8k+7= 7 mod8. Como mod conserva a soma, não há como somar 3 
>> parcelas do conjunto, mesmo com repetição, {0,1,4} e obter 7. Então n>0
>>
>> Para n>0
>> x = 4^n*(8K+7) ==> x pertence a 2 |N seja x = a^2 + b^2 + c^2 com a, b, c 
>> pertencentes a |N - {0}. teríamos que ter a,b,c pares ou um deles par e dois 
>> ímpares.
>> mas 4 | x ==> x= 0 mod4. Mas se w pertence a 2|N + 1 ==> w^2 = 1 mod4. e se 
>> y pertence a 2 |N ==> y^2 = 0 mod 4. Como temos dois ímpares e um par e como 
>> a soma se conserva temos que x = 2 mod4, absurdo. Portanto só sobra a, b, c 
>> pares Se a,b,c pares podemos escrevê-los como a= 2s; b=2t e c=2u com s,t,u 
>> naturais.
>> x = a^2+b^2+c^2= 4(s^2+t^2+u^2) ==> x1 = 4^(n-1) * (8m+7) = s^2+t^2+u^2 e 
>> vale o mesmo raciocínio de que s,t,u são pares e poderão ser escritos como 
>> s=2f; t=2g; u= 2h, com f, g, h naturais e seguir nesse algoritmo até que 
>> tenhamos xj=4^0(8m+7)= p^2+q^2+r^2, absurdo. Pois, já vimos que n= 0 não 
>> atende.
>>
>> Espero estar correto.
>>
>> Saudações.
>>
>>
>>
>>
>>
>> Em qua, 3 de abr de 2019 às 15:36, matematica10complicada 
>>  escreveu:
>>
>> Mostre que nenhum número da forma (4^n)(8k+7) , com n e k naturais pode ser 
>> escrito como soma de 3 tres quadrados
>>
>> Douglas Oliveira
>>
>> --
>> Esta mensagem foi verificada pelo sistema de antivírus e
>> acredita-se estar livre de perigo.
>>
>>
>> --
>> Esta mensagem foi verificada pelo sistema de antivírus e
>> acredita-se estar livre de perigo.
>>
>>
>> --
>> Esta mensagem foi verificada pelo sistema de antivírus e
>> acredita-se estar livre de perigo.
>>
>>
>> --
>> Esta mensagem foi verificada pelo sistema de antiv�rus e
>> acredita-se estar livre de perigo.
>>
>>
>>
>> --
>> Esta mensagem foi verificada pelo sistema de antivírus e
>> acredita-se estar livre de perigo.
>
>
> --
> Esta mensagem foi verificada pelo sistema de antivírus e
> acredita-se estar livre de perigo.

-- 
Esta mensagem foi verificada pelo sistema de antiv�rus e
 acredita-se estar livre de perigo.


=
Instru��es para entrar na lista, sair da lista e usar a lista em

[obm-l] Re: [obm-l] Re: [obm-l] Re: [obm-l] Re: [obm-l] Re: [obm-l] Re: [obm-l] Polinômio Inteiro

2019-05-02 Por tôpico Jeferson Almir
Por nada Pedro !! E sen1º é um  número algébrico . Abraço.

Em qui, 2 de mai de 2019 às 10:52, Pedro José 
escreveu:

> Bom dia!
> Jeferson,
> obrigado! Pensava, na verdade tinha certeza que sen 1grau era
> transcendente.
> Fui até pesquisar o teorema d*e *Lindemann-Weierstrass*, *que nem me
> recordava o nome, mas é para sen1, mas não um grau e sim radiano.
> Falha de armazenamento na memória.
>
> Sds,
> PJMS
>
>
>
> Em qua, 1 de mai de 2019 às 06:46, Jeferson Almir <
> jefersonram...@gmail.com> escreveu:
>
>> Puxa Raph mais uma vez muito obrigado!!
>>
>> Em ter, 30 de abr de 2019 às 19:17, Ralph Teixeira 
>> escreveu:
>>
>>> Oi, Jeferson.
>>>
>>> Sua ideia funciona: comece com P(x,y)=(y+ix)^180+1. Como voce disse,
>>> P(s,c)=0 onde c=cos1º e s=sin1º.
>>>
>>> Agora olhemos para a parte real deste polinomio: ateh dah para escrever
>>> explicitamente, mas eu vou me limitar a dizer que eh algo do tipo
>>> R(x,y)=SOMA(a_k*y^(2k)*x^(180-2k))+1 onde os a_k sao coeficientes inteiros
>>> (que incluem os sinais negativos que porventura venham das potencias pares
>>> de i).
>>>
>>> Enfim, o truque eh perceber que todas as potencias de y ali sao pares.
>>> Ou seja, trocando y^2 por 1-x^2 em todos os termos, voce vai ficar com um
>>> polinomio P(x) que satisfaz o que voce quer.
>>>
>>> Abraco, Ralph.
>>>
>>> On Tue, Apr 30, 2019 at 6:02 PM Jeferson Almir 
>>> wrote:
>>>
 Eu estou tentando através do binômio de Newton obter tal polinômio
 pegando a parte real do número complexo. Sen1º não é transcende.

 Em ter, 30 de abr de 2019 às 17:35, Pedro José 
 escreveu:

> Boa tarde!
> Não compreendi
> sen1º é um número transcendente, ou não??
>
> Sds,
> PJMS
>
>
> Em ter, 30 de abr de 2019 às 14:30, Jeferson Almir <
> jefersonram...@gmail.com> escreveu:
>
>> Mostre que existe um polinômio P(x) de coeficientes inteiros que
>> possui sen1º como raiz de P(x).
>>
>>
>> Eu tentei usar a forma exponencial de números complexos (Euler)
>>  e^(i.pi/180) = cos1º + isen1º e depois elevando 180 e pegando a parte 
>> real
>> do complexo mas ainda não consegui .
>>
>> --
>> Esta mensagem foi verificada pelo sistema de antivírus e
>> acredita-se estar livre de perigo.
>
>
> --
> Esta mensagem foi verificada pelo sistema de antivírus e
> acredita-se estar livre de perigo.


 --
 Esta mensagem foi verificada pelo sistema de antivírus e
 acredita-se estar livre de perigo.
>>>
>>>
>>> --
>>> Esta mensagem foi verificada pelo sistema de antivírus e
>>> acredita-se estar livre de perigo.
>>
>>
>> --
>> Esta mensagem foi verificada pelo sistema de antivírus e
>> acredita-se estar livre de perigo.
>
>
> --
> Esta mensagem foi verificada pelo sistema de antivírus e
> acredita-se estar livre de perigo.

-- 
Esta mensagem foi verificada pelo sistema de antiv�rus e
 acredita-se estar livre de perigo.



[obm-l] Re: [obm-l] Re: [obm-l] Re: [obm-l] Re: [obm-l] Re: [obm-l] Polinômio Inteiro

2019-05-02 Por tôpico Pedro José
Bom dia!
Jeferson,
obrigado! Pensava, na verdade tinha certeza que sen 1grau era transcendente.
Fui até pesquisar o teorema d*e *Lindemann-Weierstrass*, *que nem me
recordava o nome, mas é para sen1, mas não um grau e sim radiano.
Falha de armazenamento na memória.

Sds,
PJMS



Em qua, 1 de mai de 2019 às 06:46, Jeferson Almir 
escreveu:

> Puxa Raph mais uma vez muito obrigado!!
>
> Em ter, 30 de abr de 2019 às 19:17, Ralph Teixeira 
> escreveu:
>
>> Oi, Jeferson.
>>
>> Sua ideia funciona: comece com P(x,y)=(y+ix)^180+1. Como voce disse,
>> P(s,c)=0 onde c=cos1º e s=sin1º.
>>
>> Agora olhemos para a parte real deste polinomio: ateh dah para escrever
>> explicitamente, mas eu vou me limitar a dizer que eh algo do tipo
>> R(x,y)=SOMA(a_k*y^(2k)*x^(180-2k))+1 onde os a_k sao coeficientes inteiros
>> (que incluem os sinais negativos que porventura venham das potencias pares
>> de i).
>>
>> Enfim, o truque eh perceber que todas as potencias de y ali sao pares. Ou
>> seja, trocando y^2 por 1-x^2 em todos os termos, voce vai ficar com um
>> polinomio P(x) que satisfaz o que voce quer.
>>
>> Abraco, Ralph.
>>
>> On Tue, Apr 30, 2019 at 6:02 PM Jeferson Almir 
>> wrote:
>>
>>> Eu estou tentando através do binômio de Newton obter tal polinômio
>>> pegando a parte real do número complexo. Sen1º não é transcende.
>>>
>>> Em ter, 30 de abr de 2019 às 17:35, Pedro José 
>>> escreveu:
>>>
 Boa tarde!
 Não compreendi
 sen1º é um número transcendente, ou não??

 Sds,
 PJMS


 Em ter, 30 de abr de 2019 às 14:30, Jeferson Almir <
 jefersonram...@gmail.com> escreveu:

> Mostre que existe um polinômio P(x) de coeficientes inteiros que
> possui sen1º como raiz de P(x).
>
>
> Eu tentei usar a forma exponencial de números complexos (Euler)
>  e^(i.pi/180) = cos1º + isen1º e depois elevando 180 e pegando a parte 
> real
> do complexo mas ainda não consegui .
>
> --
> Esta mensagem foi verificada pelo sistema de antivírus e
> acredita-se estar livre de perigo.


 --
 Esta mensagem foi verificada pelo sistema de antivírus e
 acredita-se estar livre de perigo.
>>>
>>>
>>> --
>>> Esta mensagem foi verificada pelo sistema de antivírus e
>>> acredita-se estar livre de perigo.
>>
>>
>> --
>> Esta mensagem foi verificada pelo sistema de antivírus e
>> acredita-se estar livre de perigo.
>
>
> --
> Esta mensagem foi verificada pelo sistema de antivírus e
> acredita-se estar livre de perigo.

-- 
Esta mensagem foi verificada pelo sistema de antiv�rus e
 acredita-se estar livre de perigo.



[obm-l] Re: [obm-l] Re: [obm-l] Re: [obm-l] Re: [obm-l] Polinômio Inteiro

2019-05-01 Por tôpico Jeferson Almir
Puxa Raph mais uma vez muito obrigado!!

Em ter, 30 de abr de 2019 às 19:17, Ralph Teixeira 
escreveu:

> Oi, Jeferson.
>
> Sua ideia funciona: comece com P(x,y)=(y+ix)^180+1. Como voce disse,
> P(s,c)=0 onde c=cos1º e s=sin1º.
>
> Agora olhemos para a parte real deste polinomio: ateh dah para escrever
> explicitamente, mas eu vou me limitar a dizer que eh algo do tipo
> R(x,y)=SOMA(a_k*y^(2k)*x^(180-2k))+1 onde os a_k sao coeficientes inteiros
> (que incluem os sinais negativos que porventura venham das potencias pares
> de i).
>
> Enfim, o truque eh perceber que todas as potencias de y ali sao pares. Ou
> seja, trocando y^2 por 1-x^2 em todos os termos, voce vai ficar com um
> polinomio P(x) que satisfaz o que voce quer.
>
> Abraco, Ralph.
>
> On Tue, Apr 30, 2019 at 6:02 PM Jeferson Almir 
> wrote:
>
>> Eu estou tentando através do binômio de Newton obter tal polinômio
>> pegando a parte real do número complexo. Sen1º não é transcende.
>>
>> Em ter, 30 de abr de 2019 às 17:35, Pedro José 
>> escreveu:
>>
>>> Boa tarde!
>>> Não compreendi
>>> sen1º é um número transcendente, ou não??
>>>
>>> Sds,
>>> PJMS
>>>
>>>
>>> Em ter, 30 de abr de 2019 às 14:30, Jeferson Almir <
>>> jefersonram...@gmail.com> escreveu:
>>>
 Mostre que existe um polinômio P(x) de coeficientes inteiros que possui
 sen1º como raiz de P(x).


 Eu tentei usar a forma exponencial de números complexos (Euler)
  e^(i.pi/180) = cos1º + isen1º e depois elevando 180 e pegando a parte real
 do complexo mas ainda não consegui .

 --
 Esta mensagem foi verificada pelo sistema de antivírus e
 acredita-se estar livre de perigo.
>>>
>>>
>>> --
>>> Esta mensagem foi verificada pelo sistema de antivírus e
>>> acredita-se estar livre de perigo.
>>
>>
>> --
>> Esta mensagem foi verificada pelo sistema de antivírus e
>> acredita-se estar livre de perigo.
>
>
> --
> Esta mensagem foi verificada pelo sistema de antivírus e
> acredita-se estar livre de perigo.

-- 
Esta mensagem foi verificada pelo sistema de antiv�rus e
 acredita-se estar livre de perigo.



[obm-l] Re: [obm-l] Re: [obm-l] Re: [obm-l] Re: [obm-l] Teoria dos números

2019-04-29 Por tôpico Pedro José
Boa tarde!
Pelo menos consegui descobrir que se um inteiro z >= não puder ser escrito
da forma z=4^k (8m+7), com m,k >=0 e m,k inteiros então ele pode ser
representado por uma soma de três parcelas, todas quadrados perfeitos.
Já a demonstração, não consegui compreender.

Saudações,
PJMS

Em seg, 29 de abr de 2019 às 14:14,  escreveu:

>
> Em 29 de abr de 2019 11:37, Pedro José  escreveu:
>
> Bom dia!
>
> Gostei desse problema. Fiz um montão de exemplos com números que não podem
> ser escritos como 4^n(8n+7) e todos puderam ser escritos como a soma de
> três quadrados.
> Vale para todos? Se sim, alguém poderia indicar uma demonstração?
>
> Saudações,
> PJMS
>
> Em dom, 7 de abr de 2019 às 16:16, Pedro José 
> escreveu:
>
> Boa tarde!
> Fiquei na dúvida se algoritmo valia para demonstração. Mas salvo engano
> para demonstração de quais números aceitam raízes primitivas usa-se
> algoritmo.
> Mas, agora com mais calma, poderia ter usado indução.
> 1) Foi provado que não vale para n=0.
> 2) Supondo que não vale para n, não valeria para n+1, por absurdo. Pois,
> se valesse, teria que valer para n.
> Creio que teria ficado mais elegante.
>
> Saudações,
> PJMS
>
>
> Em dom, 7 de abr de 2019 às 07:41, matematica10complicada <
> profdouglaso.del...@gmail.com> escreveu:
>
> Obrigado irmão. Está correto sim.
> Douglas O.
>
> Em qui, 4 de abr de 2019 às 19:44, Pedro José 
> escreveu:
>
> Boa noite!
> Estou mal, mesmo. Ao invés de nenhum li qualquer. Tinha simulado dois,
> três, quatro e deram fora, já iria questionar.
> Mas vamos lá:
> 0^2 = 0 mod8; 1^2 = 1 mod8; 2^2 = 4 mod8 3^2= 1 mod8; 4^2 = 0 mod 8; 5^2 =
> 1 mod 8 6^2 = 4 mod 8 e 7^2 = 1 mod8;
> Portanto o quadrado de um número, ou dá 0 ou da 1 ou 4 na equivalência
> mod8.
>
> Caso n=0 ==> x=8k+7= 7 mod8. Como mod conserva a soma, não há como somar 3
> parcelas do conjunto, mesmo com repetição, {0,1,4} e obter 7. Então n>0
>
> Para n>0
> x = 4^n*(8K+7) ==> x pertence a 2 |N seja x = a^2 + b^2 + c^2 com a, b, c
> pertencentes a |N - {0}. teríamos que ter a,b,c pares ou um deles par e
> dois ímpares.
> mas 4 | x ==> x= 0 mod4. Mas se w pertence a 2|N + 1 ==> w^2 = 1 mod4. e
> se y pertence a 2 |N ==> y^2 = 0 mod 4. Como temos dois ímpares e um par e
> como a soma se conserva temos que x = 2 mod4, absurdo. Portanto só sobra a,
> b, c pares Se a,b,c pares podemos escrevê-los como a= 2s; b=2t e c=2u com
> s,t,u naturais.
> x = a^2+b^2+c^2= 4(s^2+t^2+u^2) ==> x1 = 4^(n-1) * (8m+7) = s^2+t^2+u^2 e
> vale o mesmo raciocínio de que s,t,u são pares e poderão ser escritos como
> s=2f; t=2g; u= 2h, com f, g, h naturais e seguir nesse algoritmo até que
> tenhamos xj=4^0(8m+7)= p^2+q^2+r^2, absurdo. Pois, já vimos que n= 0 não
> atende.
>
> Espero estar correto.
>
> Saudações.
>
>
>
>
>
> Em qua, 3 de abr de 2019 às 15:36, matematica10complicada <
> profdouglaso.del...@gmail.com> escreveu:
>
> Mostre que nenhum número da forma (4^n)(8k+7) , com n e k naturais pode
> ser escrito como soma de 3 tres quadrados
>
> Douglas Oliveira
>
> --
> Esta mensagem foi verificada pelo sistema de antivírus e
> acredita-se estar livre de perigo.
>
>
> --
> Esta mensagem foi verificada pelo sistema de antivírus e
> acredita-se estar livre de perigo.
>
>
> --
> Esta mensagem foi verificada pelo sistema de antivírus e
> acredita-se estar livre de perigo.
>
>
> --
> Esta mensagem foi verificada pelo sistema de antiv�rus e
> acredita-se estar livre de perigo.
>
>
>
> --
> Esta mensagem foi verificada pelo sistema de antivírus e
> acredita-se estar livre de perigo.

-- 
Esta mensagem foi verificada pelo sistema de antiv�rus e
 acredita-se estar livre de perigo.



Re: [obm-l] Re: [obm-l] Re: [obm-l] Re: [obm-l] Teoria dos números

2019-04-29 Por tôpico prmanso
Em 29 de abr de 2019 11:37, Pedro José  escreveu:Bom dia!Gostei desse problema. Fiz um montão de exemplos com números que não podem ser escritos como 4^n(8n+7) e todos puderam ser escritos como a soma de três quadrados.Vale para todos? Se sim, alguém poderia indicar uma demonstração?Saudações,PJMSEm dom, 7 de abr de 2019 às 16:16, Pedro José  escreveu:Boa tarde!Fiquei na dúvida se algoritmo valia para demonstração. Mas salvo engano para demonstração de quais números aceitam raízes primitivas usa-se algoritmo.Mas, agora com mais calma, poderia ter usado indução.1) Foi provado que não vale para n=0.2) Supondo que não vale para n, não valeria para n+1, por absurdo. Pois, se valesse, teria que valer para n.Creio que teria ficado mais elegante.Saudações,PJMSEm dom, 7 de abr de 2019 às 07:41, matematica10complicada  escreveu:Obrigado irmão. Está correto sim. Douglas O.Em qui, 4 de abr de 2019 às 19:44, Pedro José  escreveu:Boa noite!Estou mal, mesmo. Ao invés de nenhum li qualquer. Tinha simulado dois, três, quatro e deram fora, já iria questionar.Mas vamos lá:0^2 = 0 mod8; 1^2 = 1 mod8; 2^2 = 4 mod8 3^2= 1 mod8; 4^2 = 0 mod 8; 5^2 = 1 mod 8 6^2 = 4 mod 8 e 7^2 = 1 mod8;Portanto o quadrado de um número, ou dá 0 ou da 1 ou 4 na equivalência  mod8.Caso n=0 ==> x=8k+7= 7 mod8. Como mod conserva a soma, não há como somar 3 parcelas do conjunto, mesmo com repetição, {0,1,4} e obter 7. Então n>0Para n>0x = 4^n*(8K+7) ==> x pertence a 2 |N seja x = a^2 + b^2 + c^2 com a, b, c pertencentes a |N - {0}. teríamos que ter a,b,c pares ou um deles par e dois ímpares.mas 4 | x ==> x= 0 mod4. Mas se w pertence a 2|N + 1 ==> w^2 = 1 mod4. e se y pertence a 2 |N ==> y^2 = 0 mod 4. Como temos dois ímpares e um par e como a soma se conserva temos que x = 2 mod4, absurdo. Portanto só sobra a, b, c pares Se a,b,c pares podemos escrevê-los como a= 2s; b=2t e c=2u com s,t,u naturais.x = a^2+b^2+c^2= 4(s^2+t^2+u^2) ==> x1 = 4^(n-1) * (8m+7) = s^2+t^2+u^2 e vale o mesmo raciocínio de que s,t,u são pares e poderão ser escritos como s=2f; t=2g; u= 2h, com f, g, h naturais e seguir nesse algoritmo até que tenhamos xj=4^0(8m+7)= p^2+q^2+r^2, absurdo. Pois, já vimos que n= 0 não atende.Espero estar correto.Saudações. Em qua, 3 de abr de 2019 às 15:36, matematica10complicada  escreveu:Mostre que nenhum número da forma (4^n)(8k+7) , com n e k naturais pode ser escrito como soma de 3 tres quadradosDouglas Oliveira
--
Esta mensagem foi verificada pelo sistema de antivírus e 
 acredita-se estar livre de perigo.


--
Esta mensagem foi verificada pelo sistema de antivírus e 
 acredita-se estar livre de perigo.


--
Esta mensagem foi verificada pelo sistema de antivírus e 
 acredita-se estar livre de perigo.



--
Esta mensagem foi verificada pelo sistema de antiv�rus e 
 acredita-se estar livre de perigo.

--
Esta mensagem foi verificada pelo sistema de antivírus e 
 acredita-se estar livre de perigo.




[obm-l] Re: [obm-l] Re: [obm-l] Re: [obm-l] Re: [obm-l] Re: [obm-l] Valor maximo da expressão.

2018-11-29 Por tôpico Pedro José
Boa tarde!
Obrigado.
Devia ter pensado um pouco mais. Essa técnica, já havia percebido na
demonstração de que o triângulo órtico é o que tem menor perímetro, dentre
os inscritos em um triângulo acutângulo.
Da próxima vez, espero me aperceber desse artifício.

Grato,
PJMS.

Em qui, 29 de nov de 2018 às 12:45, Claudio Buffara <
claudio.buff...@gmail.com> escreveu:

> Eu resolvi o problema pra CD paralelo a AB.
>
> Daí mostrei que, de todos os CD com um dado comprimento, o que produz o
> PCD de maior área é justamente o CD paralelo a AB.
>
> Isso prova que, pra achar o PCD de área máxima, basta procurar dentre
> aqueles em que CD é paralelo a AB, que foi o que fiz antes.
>
> Enviado do meu iPhone
>
> Em 29 de nov de 2018, à(s) 10:56, Pedro José 
> escreveu:
>
> Bom dia!
> Cláudio,
> só não compreendi porque você afirma que CD tem o comprimento fixo.
>
> Saudações,
> PJMS
>
> Em qua, 28 de nov de 2018 Ã s 20:38, Claudio Buffara <
> claudio.buff...@gmail.com> escreveu:
>
>> *Dentre todos os segmentos CD com um dado comprimento, o que produz o
>> triângulo PCD de maior área é justamente aquele que é paralelo ao
>> diâmetro AB.*
>>
>> Suponhamos que a altura de PAB em relação a AB seja h e a altura de PCB
>> em relação a CD seja k.
>> Assim, área(PCD) = (1/2)*CD*k.  Â
>> Como CD tem comprimento fixo, área(PCD) será máxima quando k for
>> máximo.
>> Os triângulos PAB e PCD são semelhantes (ângulos inscritos, etc...)
>> ==> h/AB = k/CD ==> k = (CD/AB)*h  Â
>> Mas AB também é fixo (= diâmetro), de modo que k é um múltiplo fixo
>> de h ==> área(PCD) será máxima quando h for máximo.
>>
>> Seja Q o pé da altura de PAB em relação a AB ==> PQ = h.
>> AQ = h*ctg(PAB) = h*ctg(A)
>> QB = h*ctg(PBA) = h*ctg(B) ==> AB = AQÂ + QB = h*(ctg(A)Â + ctg(B)) ==> h
>> = AB/(ctg(A)Â + ctg(B)).
>> Logo, h será máximo quando ctg(A) + ctg(B) for mínimo.
>>
>> Quando a corda CD (de comprimento constante) se desloca, CAB + DBA = A +
>> B permanece constante (digamos, igual a M).
>> ctg(A)Â + ctg(B) = cos(A)/sen(A)Â + cos(B)/sen(B) = (cos(A)sen(B)Â +
>> cos(B)sen(A))/(sen(A)sen(B)) = sen(A+B)/(sen(A)sen(B)) =
>> sen(M)/(sen(A)sen(B)
>>
>> Logo, ctg(A) + ctg(B) será mínimo quando sen(A)sen(B) for máximo.
>> sen(A)sen(B) = (cos(A-B) - cos(A+B))/2 = (1/2)*cos(A-B)Â + (1/2)*cos(M)
>> será máximo quando cos(A-B) = 1, ou seja, quando A = B.
>> Finalmente, A = B <==> CD é paralelo a AB.
>>
>> []s,
>> Claudio.
>>
>>
>>
>>
>>
>>
>> On Wed, Nov 28, 2018 at 3:57 PM Claudio Buffara <
>> claudio.buff...@gmail.com> wrote:
>>
>>> Eu só tratei do caso em que CD é paralelo a AB.
>>> Chame a medida do ângulo PAB (= PCB) de t (de modo que QCO mede 2t,
>>> onde O = ponto médio de AB = centro do círculo).
>>> Então, se não errei na álgebra ou na trigonometria, a área de PDC é
>>> igual a:
>>> Area(t) = r^2*(sen(t) - 2*sen^3(t)), onde t varia entre 0 e pi/4.
>>> Derivando Area(t) em relação a t e igualando a zero, achei sen(t) =
>>> 1/raiz(6).
>>> Substituindo este valor de t, achei que Area(1/raiz(6)) = r^2*raiz(6)/9.
>>>
>>> O que eu não provei, mas que parece ser verdade, é que, dentre todos
>>> os segmentos CD com um dado comprimento, o que produz o triângulo PCD de
>>> maior área é justamente aquele que é paralelo ao diâmetro AB.
>>>
>>> []s,
>>> Claudio.
>>>
>>>
>>>
>>> On Tue, Nov 27, 2018 at 5:42 PM matematica10complicada <
>>> profdouglaso.del...@gmail.com> wrote:
>>>
 Opa Claudio, obrigado pela sua analise, vou te passar a questão e ai
 de repente podemos chegar a uma conclusão melhor.

 PROBLEMA:

 Num semicírculo de raio "r" e diametro AB, inscreve-se um
 quadrilátero ABCD, sendo P o ponto de encontro das diagonais AC e BD,
 determine a area máxima do triangulo CPD.

 Valeu pela ajuda.

 O.Douglas

 Em ter, 27 de nov de 2018 14:02, Claudio Buffara <
 claudio.buff...@gmail.com escreveu:

> Não existe, pois quando x e y tendem a Pi/2 pela esquerda,Â
> sen(x)sen(y) tende a sen^2(Pi/2) = 1Â
> eÂ
> x+y tende a Pi pela esquerda ==> tan^2(x+y) tende a zero pela direita.
> Logo, o quociente tende a +infinito.
>
> On Tue, Nov 27, 2018 at 12:27 AM matematica10complicada <
> profdouglaso.del...@gmail.com> wrote:
>
>> Ola meus caros!
>>
>> Preciso de uma ajuda no seguinte problema:
>>
>> Encontrar o valor maximo deÂ
>>
>> [Sen(x).sen(y)]\[tg(x+y)]^2 , onde x e y sao agudos.
>>
>> Obrigado desde já.Â
>>
>> Douglas Oliveira.
>>
>> --
>> Esta mensagem foi verificada pelo sistema de antivírus e
>> acredita-se estar livre de perigo.
>
>
> --
> Esta mensagem foi verificada pelo sistema de antivírus e
> acredita-se estar livre de perigo.


 --
 Esta mensagem foi verificada pelo sistema de antivírus e
 acredita-se estar livre de perigo.
>>>
>>>
>> --
>> Esta mensagem foi verificada pelo sistema 

Re: [obm-l] Re: [obm-l] Re: [obm-l] Re: [obm-l] Re: [obm-l] Valor maximo da expressão.

2018-11-29 Por tôpico Claudio Buffara
Eu resolvi o problema pra CD paralelo a AB.

Daí mostrei que, de todos os CD com um dado comprimento, o que produz o PCD de 
maior área é justamente o CD paralelo a AB.

Isso prova que, pra achar o PCD de área máxima, basta procurar dentre aqueles 
em que CD é paralelo a AB, que foi o que fiz antes.

Enviado do meu iPhone

Em 29 de nov de 2018, à(s) 10:56, Pedro José  escreveu:

> Bom dia!
> Cláudio,
> só não compreendi porque você afirma que CD tem o comprimento fixo.
> 
> Saudações,
> PJMS
> 
> Em qua, 28 de nov de 2018 Ã s 20:38, Claudio Buffara 
>  escreveu:
>> Dentre todos os segmentos CD com um dado comprimento, o que produz o 
>> triângulo PCD de maior área é justamente aquele que é paralelo ao 
>> diâmetro AB.
>> 
>> Suponhamos que a altura de PAB em relação a AB seja h e a altura de PCB em 
>> relação a CD seja k.
>> Assim, área(PCD) = (1/2)*CD*k.   
>> Como CD tem comprimento fixo, área(PCD) será máxima quando k for máximo.
>> Os triângulos PAB e PCD são semelhantes (ângulos inscritos, etc...) ==> 
>> h/AB = k/CD ==> k = (CD/AB)*h   
>> Mas AB também é fixo (= diâmetro), de modo que k é um múltiplo fixo de 
>> h ==> área(PCD) será máxima quando h for máximo.
>> 
>> Seja Q o pé da altura de PAB em relação a AB ==> PQ = h.
>> AQ = h*ctg(PAB) = h*ctg(A)
>> QB = h*ctg(PBA) = h*ctg(B) ==> AB = AQÂ + QB = h*(ctg(A)Â + ctg(B)) ==> h = 
>> AB/(ctg(A)Â + ctg(B)).
>> Logo, h será máximo quando ctg(A) + ctg(B) for mínimo.
>> 
>> Quando a corda CD (de comprimento constante) se desloca, CAB + DBA = A + B 
>> permanece constante (digamos, igual a M).
>> ctg(A)Â + ctg(B) = cos(A)/sen(A)Â + cos(B)/sen(B) = (cos(A)sen(B)Â + 
>> cos(B)sen(A))/(sen(A)sen(B)) = sen(A+B)/(sen(A)sen(B)) = sen(M)/(sen(A)sen(B)
>> 
>> Logo, ctg(A) + ctg(B) será mínimo quando sen(A)sen(B) for máximo.
>> sen(A)sen(B) = (cos(A-B) - cos(A+B))/2 = (1/2)*cos(A-B)Â + (1/2)*cos(M) 
>> será máximo quando cos(A-B) = 1, ou seja, quando A = B.
>> Finalmente, A = B <==> CD é paralelo a AB.
>> 
>> []s,
>> Claudio.
>> 
>> 
>> 
>> 
>> 
>> 
>>> On Wed, Nov 28, 2018 at 3:57 PM Claudio Buffara  
>>> wrote:
>>> Eu só tratei do caso em que CD é paralelo a AB.
>>> Chame a medida do ângulo PAB (= PCB) de t (de modo que QCO mede 2t, onde O 
>>> = ponto médio de AB = centro do círculo).
>>> Então, se não errei na álgebra ou na trigonometria, a área de PDC é 
>>> igual a:
>>> Area(t) = r^2*(sen(t) - 2*sen^3(t)), onde t varia entre 0 e pi/4.
>>> Derivando Area(t) em relação a t e igualando a zero, achei sen(t) = 
>>> 1/raiz(6).
>>> Substituindo este valor de t, achei que Area(1/raiz(6)) = r^2*raiz(6)/9.
>>> 
>>> O que eu não provei, mas que parece ser verdade, é que, dentre todos os 
>>> segmentos CD com um dado comprimento, o que produz o triângulo PCD de 
>>> maior área é justamente aquele que é paralelo ao diâmetro AB.
>>> 
>>> []s,
>>> Claudio.
>>> 
>>> 
>>> 
 On Tue, Nov 27, 2018 at 5:42 PM matematica10complicada 
  wrote:
 Opa Claudio, obrigado pela sua analise, vou te passar a questão e ai de 
 repente podemos chegar a uma conclusão melhor.
 
 PROBLEMA:
 
 Num semicírculo de raio "r" e diametro AB, inscreve-se um quadrilátero 
 ABCD, sendo P o ponto de encontro das diagonais AC e BD, determine a area 
 máxima do triangulo CPD.
 
 Valeu pela ajuda.
 
 O.Douglas
 
 Em ter, 27 de nov de 2018 14:02, Claudio Buffara 
  Não existe, pois quando x e y tendem a Pi/2 pela esquerda, 
> sen(x)sen(y) tende a sen^2(Pi/2) = 1Â 
> e 
> x+y tende a Pi pela esquerda ==> tan^2(x+y) tende a zero pela direita.
> Logo, o quociente tende a +infinito.
> 
>> On Tue, Nov 27, 2018 at 12:27 AM matematica10complicada 
>>  wrote:
>> Ola meus caros!
>> 
>> Preciso de uma ajuda no seguinte problema:
>> 
>> Encontrar o valor maximo de 
>> 
>> [Sen(x).sen(y)]\[tg(x+y)]^2 , onde x e y sao agudos.
>> 
>> Obrigado desde já. 
>> 
>> Douglas Oliveira.
>> 
>> -- 
>> Esta mensagem foi verificada pelo sistema de antivírus e 
>> acredita-se estar livre de perigo.
> 
> -- 
> Esta mensagem foi verificada pelo sistema de antivírus e 
> acredita-se estar livre de perigo.
 
 -- 
 Esta mensagem foi verificada pelo sistema de antivírus e 
 acredita-se estar livre de perigo.
>> 
>> -- 
>> Esta mensagem foi verificada pelo sistema de antivírus e 
>> acredita-se estar livre de perigo.
> 
> -- 
> Esta mensagem foi verificada pelo sistema de antivírus e 
> acredita-se estar livre de perigo.

-- 
Esta mensagem foi verificada pelo sistema de antiv�rus e
 acredita-se estar livre de perigo.



[obm-l] Re: [obm-l] Re: [obm-l] Re: [obm-l] Re: [obm-l] Valor maximo da expressão.

2018-11-29 Por tôpico Pedro José
Bom dia!
Cláudio,
só não compreendi porque você afirma que CD tem o comprimento fixo.

Saudações,
PJMS

Em qua, 28 de nov de 2018 às 20:38, Claudio Buffara <
claudio.buff...@gmail.com> escreveu:

> *Dentre todos os segmentos CD com um dado comprimento, o que produz o
> triângulo PCD de maior área é justamente aquele que é paralelo ao diâmetro
> AB.*
>
> Suponhamos que a altura de PAB em relação a AB seja h e a altura de PCB em
> relação a CD seja k.
> Assim, área(PCD) = (1/2)*CD*k.
> Como CD tem comprimento fixo, área(PCD) será máxima quando k for máximo.
> Os triângulos PAB e PCD são semelhantes (ângulos inscritos, etc...) ==>
> h/AB = k/CD ==> k = (CD/AB)*h
> Mas AB também é fixo (= diâmetro), de modo que k é um múltiplo fixo de h
> ==> área(PCD) será máxima quando h for máximo.
>
> Seja Q o pé da altura de PAB em relação a AB ==> PQ = h.
> AQ = h*ctg(PAB) = h*ctg(A)
> QB = h*ctg(PBA) = h*ctg(B) ==> AB = AQ + QB = h*(ctg(A) + ctg(B)) ==> h =
> AB/(ctg(A) + ctg(B)).
> Logo, h será máximo quando ctg(A) + ctg(B) for mínimo.
>
> Quando a corda CD (de comprimento constante) se desloca, CAB + DBA = A + B
> permanece constante (digamos, igual a M).
> ctg(A) + ctg(B) = cos(A)/sen(A) + cos(B)/sen(B) = (cos(A)sen(B) +
> cos(B)sen(A))/(sen(A)sen(B)) = sen(A+B)/(sen(A)sen(B)) =
> sen(M)/(sen(A)sen(B)
>
> Logo, ctg(A) + ctg(B) será mínimo quando sen(A)sen(B) for máximo.
> sen(A)sen(B) = (cos(A-B) - cos(A+B))/2 = (1/2)*cos(A-B) + (1/2)*cos(M)
> será máximo quando cos(A-B) = 1, ou seja, quando A = B.
> Finalmente, A = B <==> CD é paralelo a AB.
>
> []s,
> Claudio.
>
>
>
>
>
>
> On Wed, Nov 28, 2018 at 3:57 PM Claudio Buffara 
> wrote:
>
>> Eu só tratei do caso em que CD é paralelo a AB.
>> Chame a medida do ângulo PAB (= PCB) de t (de modo que QCO mede 2t, onde
>> O = ponto médio de AB = centro do círculo).
>> Então, se não errei na álgebra ou na trigonometria, a área de PDC é igual
>> a:
>> Area(t) = r^2*(sen(t) - 2*sen^3(t)), onde t varia entre 0 e pi/4.
>> Derivando Area(t) em relação a t e igualando a zero, achei sen(t) =
>> 1/raiz(6).
>> Substituindo este valor de t, achei que Area(1/raiz(6)) = r^2*raiz(6)/9.
>>
>> O que eu não provei, mas que parece ser verdade, é que, dentre todos os
>> segmentos CD com um dado comprimento, o que produz o triângulo PCD de maior
>> área é justamente aquele que é paralelo ao diâmetro AB.
>>
>> []s,
>> Claudio.
>>
>>
>>
>> On Tue, Nov 27, 2018 at 5:42 PM matematica10complicada <
>> profdouglaso.del...@gmail.com> wrote:
>>
>>> Opa Claudio, obrigado pela sua analise, vou te passar a questão e ai de
>>> repente podemos chegar a uma conclusão melhor.
>>>
>>> PROBLEMA:
>>>
>>> Num semicírculo de raio "r" e diametro AB, inscreve-se um quadrilátero
>>> ABCD, sendo P o ponto de encontro das diagonais AC e BD, determine a area
>>> máxima do triangulo CPD.
>>>
>>> Valeu pela ajuda.
>>>
>>> O.Douglas
>>>
>>> Em ter, 27 de nov de 2018 14:02, Claudio Buffara <
>>> claudio.buff...@gmail.com escreveu:
>>>
 Não existe, pois quando x e y tendem a Pi/2 pela esquerda,
 sen(x)sen(y) tende a sen^2(Pi/2) = 1
 e
 x+y tende a Pi pela esquerda ==> tan^2(x+y) tende a zero pela direita.
 Logo, o quociente tende a +infinito.

 On Tue, Nov 27, 2018 at 12:27 AM matematica10complicada <
 profdouglaso.del...@gmail.com> wrote:

> Ola meus caros!
>
> Preciso de uma ajuda no seguinte problema:
>
> Encontrar o valor maximo de
>
> [Sen(x).sen(y)]\[tg(x+y)]^2 , onde x e y sao agudos.
>
> Obrigado desde já.
>
> Douglas Oliveira.
>
> --
> Esta mensagem foi verificada pelo sistema de antivírus e
> acredita-se estar livre de perigo.


 --
 Esta mensagem foi verificada pelo sistema de antivírus e
 acredita-se estar livre de perigo.
>>>
>>>
>>> --
>>> Esta mensagem foi verificada pelo sistema de antivírus e
>>> acredita-se estar livre de perigo.
>>
>>
> --
> Esta mensagem foi verificada pelo sistema de antivírus e
> acredita-se estar livre de perigo.

-- 
Esta mensagem foi verificada pelo sistema de antiv�rus e
 acredita-se estar livre de perigo.



[obm-l] Re: [obm-l] Re: [obm-l] Re: [obm-l] Re: [obm-l] Valor maximo da expressão.

2018-11-28 Por tôpico Pedro José
Boa tarde!
Não percebera a restrição que AB está sobre o diâmetro. Julgue ser um
quadrilátero qualquer.
Bola fora.
Saudações,
PJMS

Em Qua, 28 de nov de 2018 17:22, Pedro José  escreveu:

> Boa tarde!
> Pensei assim, o triângulo inscrito no semicírculo que tem a maior área é o
> que tem a hipotenusa igual ao diâmetro e a altura igual ao r, cuja área
> será r^2.
> Então posso arbitrar o ponto C numa extremidade do diâmetro, o ponto D,
> tal que a projeção ortogonal de D sobre o diâmetro dê o centro do
> semicírculo.
> Seja M o ponto diametralmente oposto a C. Não há como P ser M, mas posso
> aproximá-lo de M o quanto quiser. Assim não haverá um máximo, mas um
> limitante, área < r^2.
> Procede?
> Saudações,
> PJMS.
>
> Em Qua, 28 de nov de 2018 16:06, Claudio Buffara <
> claudio.buff...@gmail.com> escreveu:
>
>> Eu só tratei do caso em que CD é paralelo a AB.
>> Chame a medida do ângulo PAB (= PCB) de t (de modo que QCO mede 2t, onde
>> O = ponto médio de AB = centro do círculo).
>> Então, se não errei na álgebra ou na trigonometria, a área de PDC é igual
>> a:
>> Area(t) = r^2*(sen(t) - 2*sen^3(t)), onde t varia entre 0 e pi/4.
>> Derivando Area(t) em relação a t e igualando a zero, achei sen(t) =
>> 1/raiz(6).
>> Substituindo este valor de t, achei que Area(1/raiz(6)) = r^2*raiz(6)/9.
>>
>> O que eu não provei, mas que parece ser verdade, é que, dentre todos os
>> segmentos CD com um dado comprimento, o que produz o triângulo PCD de maior
>> área é justamente aquele que é paralelo ao diâmetro AB.
>>
>> []s,
>> Claudio.
>>
>>
>>
>> On Tue, Nov 27, 2018 at 5:42 PM matematica10complicada <
>> profdouglaso.del...@gmail.com> wrote:
>>
>>> Opa Claudio, obrigado pela sua analise, vou te passar a questão e ai de
>>> repente podemos chegar a uma conclusão melhor.
>>>
>>> PROBLEMA:
>>>
>>> Num semicírculo de raio "r" e diametro AB, inscreve-se um quadrilátero
>>> ABCD, sendo P o ponto de encontro das diagonais AC e BD, determine a area
>>> máxima do triangulo CPD.
>>>
>>> Valeu pela ajuda.
>>>
>>> O.Douglas
>>>
>>> Em ter, 27 de nov de 2018 14:02, Claudio Buffara <
>>> claudio.buff...@gmail.com escreveu:
>>>
 Não existe, pois quando x e y tendem a Pi/2 pela esquerda,
 sen(x)sen(y) tende a sen^2(Pi/2) = 1
 e
 x+y tende a Pi pela esquerda ==> tan^2(x+y) tende a zero pela direita.
 Logo, o quociente tende a +infinito.

 On Tue, Nov 27, 2018 at 12:27 AM matematica10complicada <
 profdouglaso.del...@gmail.com> wrote:

> Ola meus caros!
>
> Preciso de uma ajuda no seguinte problema:
>
> Encontrar o valor maximo de
>
> [Sen(x).sen(y)]\[tg(x+y)]^2 , onde x e y sao agudos.
>
> Obrigado desde já.
>
> Douglas Oliveira.
>
> --
> Esta mensagem foi verificada pelo sistema de antivírus e
> acredita-se estar livre de perigo.


 --
 Esta mensagem foi verificada pelo sistema de antivírus e
 acredita-se estar livre de perigo.
>>>
>>>
>>> --
>>> Esta mensagem foi verificada pelo sistema de antivírus e
>>> acredita-se estar livre de perigo.
>>
>>
>> --
>> Esta mensagem foi verificada pelo sistema de antivírus e
>> acredita-se estar livre de perigo.
>
>

-- 
Esta mensagem foi verificada pelo sistema de antiv�rus e
 acredita-se estar livre de perigo.



[obm-l] Re: [obm-l] Re: [obm-l] Re: [obm-l] Re: [obm-l] Valor maximo da expressão.

2018-11-28 Por tôpico Pedro José
Boa tarde!
Pensei assim, o triângulo inscrito no semicírculo que tem a maior área é o
que tem a hipotenusa igual ao diâmetro e a altura igual ao r, cuja área
será r^2.
Então posso arbitrar o ponto C numa extremidade do diâmetro, o ponto D, tal
que a projeção ortogonal de D sobre o diâmetro dê o centro do semicírculo.
Seja M o ponto diametralmente oposto a C. Não há como P ser M, mas posso
aproximá-lo de M o quanto quiser. Assim não haverá um máximo, mas um
limitante, área < r^2.
Procede?
Saudações,
PJMS.

Em Qua, 28 de nov de 2018 16:06, Claudio Buffara 
escreveu:

> Eu só tratei do caso em que CD é paralelo a AB.
> Chame a medida do ângulo PAB (= PCB) de t (de modo que QCO mede 2t, onde O
> = ponto médio de AB = centro do círculo).
> Então, se não errei na álgebra ou na trigonometria, a área de PDC é igual
> a:
> Area(t) = r^2*(sen(t) - 2*sen^3(t)), onde t varia entre 0 e pi/4.
> Derivando Area(t) em relação a t e igualando a zero, achei sen(t) =
> 1/raiz(6).
> Substituindo este valor de t, achei que Area(1/raiz(6)) = r^2*raiz(6)/9.
>
> O que eu não provei, mas que parece ser verdade, é que, dentre todos os
> segmentos CD com um dado comprimento, o que produz o triângulo PCD de maior
> área é justamente aquele que é paralelo ao diâmetro AB.
>
> []s,
> Claudio.
>
>
>
> On Tue, Nov 27, 2018 at 5:42 PM matematica10complicada <
> profdouglaso.del...@gmail.com> wrote:
>
>> Opa Claudio, obrigado pela sua analise, vou te passar a questão e ai de
>> repente podemos chegar a uma conclusão melhor.
>>
>> PROBLEMA:
>>
>> Num semicírculo de raio "r" e diametro AB, inscreve-se um quadrilátero
>> ABCD, sendo P o ponto de encontro das diagonais AC e BD, determine a area
>> máxima do triangulo CPD.
>>
>> Valeu pela ajuda.
>>
>> O.Douglas
>>
>> Em ter, 27 de nov de 2018 14:02, Claudio Buffara <
>> claudio.buff...@gmail.com escreveu:
>>
>>> Não existe, pois quando x e y tendem a Pi/2 pela esquerda,
>>> sen(x)sen(y) tende a sen^2(Pi/2) = 1
>>> e
>>> x+y tende a Pi pela esquerda ==> tan^2(x+y) tende a zero pela direita.
>>> Logo, o quociente tende a +infinito.
>>>
>>> On Tue, Nov 27, 2018 at 12:27 AM matematica10complicada <
>>> profdouglaso.del...@gmail.com> wrote:
>>>
 Ola meus caros!

 Preciso de uma ajuda no seguinte problema:

 Encontrar o valor maximo de

 [Sen(x).sen(y)]\[tg(x+y)]^2 , onde x e y sao agudos.

 Obrigado desde já.

 Douglas Oliveira.

 --
 Esta mensagem foi verificada pelo sistema de antivírus e
 acredita-se estar livre de perigo.
>>>
>>>
>>> --
>>> Esta mensagem foi verificada pelo sistema de antivírus e
>>> acredita-se estar livre de perigo.
>>
>>
>> --
>> Esta mensagem foi verificada pelo sistema de antivírus e
>> acredita-se estar livre de perigo.
>
>
> --
> Esta mensagem foi verificada pelo sistema de antivírus e
> acredita-se estar livre de perigo.

-- 
Esta mensagem foi verificada pelo sistema de antiv�rus e
 acredita-se estar livre de perigo.



[obm-l] Re: [obm-l] Re: [obm-l] Re: [obm-l] Re: [obm-l] Valor maximo da expressão.

2018-11-28 Por tôpico matematica10complicada
Estou desconfiado do hexagono , mas ainda nao conclui. Tentei achar
primeiro a area em funcao dos 3 arcos e depois usar uma especie de
desigualdade tipo Jensen.

Douglas Oliveira.

Em qua, 28 de nov de 2018 15:06, Claudio Buffara  Eu só tratei do caso em que CD é paralelo a AB.
> Chame a medida do ângulo PAB (= PCB) de t (de modo que QCO mede 2t, onde O
> = ponto médio de AB = centro do círculo).
> Então, se não errei na álgebra ou na trigonometria, a área de PDC é igual
> a:
> Area(t) = r^2*(sen(t) - 2*sen^3(t)), onde t varia entre 0 e pi/4.
> Derivando Area(t) em relação a t e igualando a zero, achei sen(t) =
> 1/raiz(6).
> Substituindo este valor de t, achei que Area(1/raiz(6)) = r^2*raiz(6)/9.
>
> O que eu não provei, mas que parece ser verdade, é que, dentre todos os
> segmentos CD com um dado comprimento, o que produz o triângulo PCD de maior
> área é justamente aquele que é paralelo ao diâmetro AB.
>
> []s,
> Claudio.
>
>
>
> On Tue, Nov 27, 2018 at 5:42 PM matematica10complicada <
> profdouglaso.del...@gmail.com> wrote:
>
>> Opa Claudio, obrigado pela sua analise, vou te passar a questão e ai de
>> repente podemos chegar a uma conclusão melhor.
>>
>> PROBLEMA:
>>
>> Num semicírculo de raio "r" e diametro AB, inscreve-se um quadrilátero
>> ABCD, sendo P o ponto de encontro das diagonais AC e BD, determine a area
>> máxima do triangulo CPD.
>>
>> Valeu pela ajuda.
>>
>> O.Douglas
>>
>> Em ter, 27 de nov de 2018 14:02, Claudio Buffara <
>> claudio.buff...@gmail.com escreveu:
>>
>>> Não existe, pois quando x e y tendem a Pi/2 pela esquerda,
>>> sen(x)sen(y) tende a sen^2(Pi/2) = 1
>>> e
>>> x+y tende a Pi pela esquerda ==> tan^2(x+y) tende a zero pela direita.
>>> Logo, o quociente tende a +infinito.
>>>
>>> On Tue, Nov 27, 2018 at 12:27 AM matematica10complicada <
>>> profdouglaso.del...@gmail.com> wrote:
>>>
 Ola meus caros!

 Preciso de uma ajuda no seguinte problema:

 Encontrar o valor maximo de

 [Sen(x).sen(y)]\[tg(x+y)]^2 , onde x e y sao agudos.

 Obrigado desde já.

 Douglas Oliveira.

 --
 Esta mensagem foi verificada pelo sistema de antivírus e
 acredita-se estar livre de perigo.
>>>
>>>
>>> --
>>> Esta mensagem foi verificada pelo sistema de antivírus e
>>> acredita-se estar livre de perigo.
>>
>>
>> --
>> Esta mensagem foi verificada pelo sistema de antivírus e
>> acredita-se estar livre de perigo.
>
>
> --
> Esta mensagem foi verificada pelo sistema de antivírus e
> acredita-se estar livre de perigo.

-- 
Esta mensagem foi verificada pelo sistema de antiv�rus e
 acredita-se estar livre de perigo.



[obm-l] Re: [obm-l] Re: [obm-l] Re: [obm-l] Re: [obm-l] Teoria dos números

2018-11-28 Por tôpico Pedro José
Boa tarde!
Perdoem-me pela insistência.
Mas outra forma de pensar.
Se k>0, e se a>b e se pensarmos em duas soluções positivas logicamente
estamos assumindo que a seja máximo. Pois, se existe a1 solução e a1>=a
então a1.a=b^2-k>b^2, absurdo.
Portanto quando dizemos que a>b, estamos escolhendo conscientemente a
solução máxima. Então não é ocaso de pegar uma solução hipotética, supor
que é solução mínima  e provar que existe uma menor, gerando absurdo.
Saudações,
PJMS

Em Qua, 28 de nov de 2018 15:42, Pedro José  escreveu:

> Boa tarde!
> Preciso de ajuda.
> Após pensar mais um pouco é bem razoável que dada uma solução hipotética e
> se consiga provar que há uma menor, que seja um absurdo. Absurdo no
> sentido, que não há solução. Gostaria até que me sugerissem material
> didático sobre o tópico.
> Não obstante existe solução para a1 e
> x inteiro.
> Então há um paradoxo. Que por um lado se a é solução para a 0 Mas quando se achou a foi feita uma restrição, SPG, que a >=b e após
> estudar o caso a=b, ficamos com a restrição a>b, que é usada para provar
> que a1b. Só que:
> a1=(b^2-k)/ab.
> Então esse é o ponto a1 mesmo sendo maior que zero,  não é solução pois
> a1 solução. A prova para a  Por favor, alguém poderia opinar.
> Saudações,
> PJMS
>
> Em Seg, 26 de nov de 2018 01:59, Pedro José 
> escreveu:
>
>> Bom dia!
>> Refiro-me a solução recomendada por Israel.
>> A princípio eu encrenquei com a solução. Pois, sem nenhuma caracterização
>> definiu que a era mínimo. Então achei que a solução para a > capenga. Mas como não houve nenhuma crítica, julguei ser cisma minha. Mas
>> depois me veio o pensamento, usando a técnica usada na resolução sempre que
>> houver duas soluções(digo duas mesmo, distintas) haveria um absurdo. Pois
>> ele supôs que a era mínimo e provou que a1, solução, a1> como ele não usou nenhum argumento para supor que a era mínimo, apenas
>> arbitrou, poderia ter arbitrado que a era máximo e se a1>a, também seria
>> absurdo.
>> Aí, encrenquei mesmo com a soluçao e achei essa família de soluções para
>> a> K=x^2; b=x^3 e a =x^5-x, que para x>1, xinteiro, atende a> k=(a^2+b^2)/(ab+1); continua dando um quadrado perfeito, mas se não fosse?
>> A linha de argumento da solução,  desprezou essa possibilidade.
>> Preciso ajuda, estou correto ou errado?
>> Grato,
>> PJMS
>>
>> Em Seg, 27 de ago de 2018 11:01, Pedro José 
>> escreveu:
>>
>>> Bom dia!
>>>
>>> Linda solução pela simplicidade de ferramentas utilizadas.
>>> Todavia, creio eu que não foi de bom alvitre utilizar a imagem de um
>>> matemático famoso e divulgar que ele só ganhou um ponto na questão.
>>> A mensagem, não explícita, mas é uma mensagem:"Ele não resolveu mas eu
>>> sim."
>>> As condições de resolução são totalmente diversas. Inclusive, devido às
>>> questões anteriores ele já pode ter chegado a essa com o ponterinho do
>>> relógio pendurado.
>>> A solução do problema, mesmo tardia, sem a carga emocional que uma IMO
>>> deve impor aos seus participantes, ainda é carregada de méritos, e na minha
>>> visão, essa em particular, com uma beleza maior por ser sutil e singela.
>>> Mas usar a imagem da "fera", não obstante não ser dono da verdade, foi
>>> bola fora.
>>>
>>> Saudações,
>>> PJMS
>>>
>>>
>>> Em qui, 23 de ago de 2018 às 15:50, Israel Meireles Chrisostomo <
>>> israelmchrisost...@gmail.com> escreveu:
>>>
 Assista a esse vídeo:
 https://www.youtube.com/watch?v=Cy3Vyl-jxpk

 Em qui, 23 de ago de 2018 às 14:09, Daniel Quevedo 
 escreveu:

> Blz não sabia q era de lá, vou consultar. Obrigado!
>
> Em qui, 23 de ago de 2018 às 10:30, Claudio Buffara <
> claudio.buff...@gmail.com> escreveu:
>
>> Esse é clássico. Foi o problema 6 da IMO de 1988 e é usualmente
>> considerado o problema mais difícil proposto numa IMO, pelo menos até
>> aquela data.
>>
>> Um bom ponto de partida pode ser este:
>> https://en.wikipedia.org/wiki/Vieta_jumping
>> Ou então: https://mks.mff.cuni.cz/kalva/imo.html
>>
>> []s,
>> Claudio.
>>
>>
>>
>> 2018-08-23 9:57 GMT-03:00 Daniel Quevedo :
>>
>>> Sejam a e b inteiros estritamente positivos tais que (ab + 1) é um
>>> divisor de (a^2 + b^2). Sobre o número  (a^2 +b^2)/(ab +1) podemos 
>>> afirmar
>>> que é um quadrado perfeito:
>>> A) se, e só se, a e b também o forem.
>>> B) se, e só se, a e b tiverem acreana paridade
>>> C) se, e só se, a e b tiverem paridades distintas
>>> D) somente para um número finito de valores de a e b
>>> E) sempre
>>>
>>> R: e
>>>
>> --
>>> Fiscal: Daniel Quevedo
>>>
>>
>>> --
>>> Esta mensagem foi verificada pelo sistema de antivírus e
>>> acredita-se estar livre de perigo.
>>
>>
>>
>> --
>> Esta mensagem foi verificada pelo sistema de antivírus e
>> acredita-se estar livre de perigo.
>
> --
> Fiscal: Daniel Quevedo
>
> --
> Esta mensagem foi verificada pelo 

[obm-l] Re: [obm-l] Re: [obm-l] Re: [obm-l] Re: [obm-l] Teoria dos números

2018-11-28 Por tôpico Pedro José
Boa tarde!
Preciso de ajuda.
Após pensar mais um pouco é bem razoável que dada uma solução hipotética e
se consiga provar que há uma menor, que seja um absurdo. Absurdo no
sentido, que não há solução. Gostaria até que me sugerissem material
didático sobre o tópico.
Não obstante existe solução para a1 e
x inteiro.
Então há um paradoxo. Que por um lado se a é solução para a=b e após
estudar o caso a=b, ficamos com a restrição a>b, que é usada para provar
que a1b. Só que:
a1=(b^2-k)/ab.
Então esse é o ponto a1 mesmo sendo maior que zero,  não é solução pois
a1 escreveu:

> Bom dia!
> Refiro-me a solução recomendada por Israel.
> A princípio eu encrenquei com a solução. Pois, sem nenhuma caracterização
> definiu que a era mínimo. Então achei que a solução para a  capenga. Mas como não houve nenhuma crítica, julguei ser cisma minha. Mas
> depois me veio o pensamento, usando a técnica usada na resolução sempre que
> houver duas soluções(digo duas mesmo, distintas) haveria um absurdo. Pois
> ele supôs que a era mínimo e provou que a1, solução, a1 como ele não usou nenhum argumento para supor que a era mínimo, apenas
> arbitrou, poderia ter arbitrado que a era máximo e se a1>a, também seria
> absurdo.
> Aí, encrenquei mesmo com a soluçao e achei essa família de soluções para
> a K=x^2; b=x^3 e a =x^5-x, que para x>1, xinteiro, atende a k=(a^2+b^2)/(ab+1); continua dando um quadrado perfeito, mas se não fosse?
> A linha de argumento da solução,  desprezou essa possibilidade.
> Preciso ajuda, estou correto ou errado?
> Grato,
> PJMS
>
> Em Seg, 27 de ago de 2018 11:01, Pedro José 
> escreveu:
>
>> Bom dia!
>>
>> Linda solução pela simplicidade de ferramentas utilizadas.
>> Todavia, creio eu que não foi de bom alvitre utilizar a imagem de um
>> matemático famoso e divulgar que ele só ganhou um ponto na questão.
>> A mensagem, não explícita, mas é uma mensagem:"Ele não resolveu mas eu
>> sim."
>> As condições de resolução são totalmente diversas. Inclusive, devido às
>> questões anteriores ele já pode ter chegado a essa com o ponterinho do
>> relógio pendurado.
>> A solução do problema, mesmo tardia, sem a carga emocional que uma IMO
>> deve impor aos seus participantes, ainda é carregada de méritos, e na minha
>> visão, essa em particular, com uma beleza maior por ser sutil e singela.
>> Mas usar a imagem da "fera", não obstante não ser dono da verdade, foi
>> bola fora.
>>
>> Saudações,
>> PJMS
>>
>>
>> Em qui, 23 de ago de 2018 às 15:50, Israel Meireles Chrisostomo <
>> israelmchrisost...@gmail.com> escreveu:
>>
>>> Assista a esse vídeo:
>>> https://www.youtube.com/watch?v=Cy3Vyl-jxpk
>>>
>>> Em qui, 23 de ago de 2018 às 14:09, Daniel Quevedo 
>>> escreveu:
>>>
 Blz não sabia q era de lá, vou consultar. Obrigado!

 Em qui, 23 de ago de 2018 às 10:30, Claudio Buffara <
 claudio.buff...@gmail.com> escreveu:

> Esse é clássico. Foi o problema 6 da IMO de 1988 e é usualmente
> considerado o problema mais difícil proposto numa IMO, pelo menos até
> aquela data.
>
> Um bom ponto de partida pode ser este:
> https://en.wikipedia.org/wiki/Vieta_jumping
> Ou então: https://mks.mff.cuni.cz/kalva/imo.html
>
> []s,
> Claudio.
>
>
>
> 2018-08-23 9:57 GMT-03:00 Daniel Quevedo :
>
>> Sejam a e b inteiros estritamente positivos tais que (ab + 1) é um
>> divisor de (a^2 + b^2). Sobre o número  (a^2 +b^2)/(ab +1) podemos 
>> afirmar
>> que é um quadrado perfeito:
>> A) se, e só se, a e b também o forem.
>> B) se, e só se, a e b tiverem acreana paridade
>> C) se, e só se, a e b tiverem paridades distintas
>> D) somente para um número finito de valores de a e b
>> E) sempre
>>
>> R: e
>>
> --
>> Fiscal: Daniel Quevedo
>>
>
>> --
>> Esta mensagem foi verificada pelo sistema de antivírus e
>> acredita-se estar livre de perigo.
>
>
>
> --
> Esta mensagem foi verificada pelo sistema de antivírus e
> acredita-se estar livre de perigo.

 --
 Fiscal: Daniel Quevedo

 --
 Esta mensagem foi verificada pelo sistema de antivírus e
 acredita-se estar livre de perigo.
>>>
>>>
>>>
>>> --
>>> Israel Meireles Chrisostomo
>>>
>>> --
>>> Esta mensagem foi verificada pelo sistema de antivírus e
>>> acredita-se estar livre de perigo.
>>
>>

-- 
Esta mensagem foi verificada pelo sistema de antiv�rus e
 acredita-se estar livre de perigo.



[obm-l] Re: [obm-l] Re: [obm-l] Re: [obm-l] Re: [obm-l] Teoria dos números

2018-11-25 Por tôpico Pedro José
Bom dia!
Refiro-me a solução recomendada por Israel.
A princípio eu encrenquei com a solução. Pois, sem nenhuma caracterização
definiu que a era mínimo. Então achei que a solução para a a, também seria
absurdo.
Aí, encrenquei mesmo com a soluçao e achei essa família de soluções para
a1, xinteiro, atende a escreveu:

> Bom dia!
>
> Linda solução pela simplicidade de ferramentas utilizadas.
> Todavia, creio eu que não foi de bom alvitre utilizar a imagem de um
> matemático famoso e divulgar que ele só ganhou um ponto na questão.
> A mensagem, não explícita, mas é uma mensagem:"Ele não resolveu mas eu
> sim."
> As condições de resolução são totalmente diversas. Inclusive, devido às
> questões anteriores ele já pode ter chegado a essa com o ponterinho do
> relógio pendurado.
> A solução do problema, mesmo tardia, sem a carga emocional que uma IMO
> deve impor aos seus participantes, ainda é carregada de méritos, e na minha
> visão, essa em particular, com uma beleza maior por ser sutil e singela.
> Mas usar a imagem da "fera", não obstante não ser dono da verdade, foi
> bola fora.
>
> Saudações,
> PJMS
>
>
> Em qui, 23 de ago de 2018 às 15:50, Israel Meireles Chrisostomo <
> israelmchrisost...@gmail.com> escreveu:
>
>> Assista a esse vídeo:
>> https://www.youtube.com/watch?v=Cy3Vyl-jxpk
>>
>> Em qui, 23 de ago de 2018 às 14:09, Daniel Quevedo 
>> escreveu:
>>
>>> Blz não sabia q era de lá, vou consultar. Obrigado!
>>>
>>> Em qui, 23 de ago de 2018 às 10:30, Claudio Buffara <
>>> claudio.buff...@gmail.com> escreveu:
>>>
 Esse é clássico. Foi o problema 6 da IMO de 1988 e é usualmente
 considerado o problema mais difícil proposto numa IMO, pelo menos até
 aquela data.

 Um bom ponto de partida pode ser este:
 https://en.wikipedia.org/wiki/Vieta_jumping
 Ou então: https://mks.mff.cuni.cz/kalva/imo.html

 []s,
 Claudio.



 2018-08-23 9:57 GMT-03:00 Daniel Quevedo :

> Sejam a e b inteiros estritamente positivos tais que (ab + 1) é um
> divisor de (a^2 + b^2). Sobre o número  (a^2 +b^2)/(ab +1) podemos afirmar
> que é um quadrado perfeito:
> A) se, e só se, a e b também o forem.
> B) se, e só se, a e b tiverem acreana paridade
> C) se, e só se, a e b tiverem paridades distintas
> D) somente para um número finito de valores de a e b
> E) sempre
>
> R: e
>
 --
> Fiscal: Daniel Quevedo
>

> --
> Esta mensagem foi verificada pelo sistema de antivírus e
> acredita-se estar livre de perigo.



 --
 Esta mensagem foi verificada pelo sistema de antivírus e
 acredita-se estar livre de perigo.
>>>
>>> --
>>> Fiscal: Daniel Quevedo
>>>
>>> --
>>> Esta mensagem foi verificada pelo sistema de antivírus e
>>> acredita-se estar livre de perigo.
>>
>>
>>
>> --
>> Israel Meireles Chrisostomo
>>
>> --
>> Esta mensagem foi verificada pelo sistema de antivírus e
>> acredita-se estar livre de perigo.
>
>

-- 
Esta mensagem foi verificada pelo sistema de antiv�rus e
 acredita-se estar livre de perigo.



[obm-l] Re: [obm-l] Re: [obm-l] Re: [obm-l] Re: [obm-l] Demonstração com Geometria Plana?

2018-11-24 Por tôpico Anderson Torres
Em sex, 23 de nov de 2018 às 22:47, Vanderlei Nemitz
 escreveu:
>
> Estamos aguardando o Carlos Victor...
> :)
>
> Em sex, 23 de nov de 2018 18:14, Mauricio de Araujo 
> >
>> Alguem conseguiu finalizar a demonstração?
>>
>> Em qua, 21 de nov de 2018 11:52, Vanderlei Nemitz > escreveu:
>>>
>>> Hummm...
>>> Parece que prolongando BF e DC, que se encontram num ponto Q, E é o 
>>> ortocentro do triângulo BDQ.
>>> O desenho sugere isso.
>>> Mas como mostrar isso?

Eu estava pensando em usar geometria projetiva, algo acerca de
conjugados harmônicos. Mas o máximo que consegui foi obter um ponto
médio...

>>>
>>> Em ter, 20 de nov de 2018 23:38, Carlos Victor >> escreveu:

 Oi Vanderlei,

 Uma dica : tente mostrar que o ponto E é o ortocentro de um triângulo " 
 estratégico". É muito legal que você descubra sozinho

 Abraços

 Carlos Victor

 Em 20/11/2018 17:33, Vanderlei Nemitz escreveu:

 Pessoal, o seguinte problema sai "tranquilamente" usando Geometria 
 Analítica.
 Tentei usar Geometria Plana, mas apenas girei bastante, sem concluir. Será 
 que é possível?

 Dado um ponto P situado no prolongamento do lado AB de um quadrado ABCD, 
 traçam-se as retas PC e PD. Pelo ponto E, intersecção de BC e PD, 
 conduzimos a reta AE cuja intersecção com PC é o ponto F. Provar que BF e 
 PD são perpendiculares.

 --
 Esta mensagem foi verificada pelo sistema de antivrus e
 acredita-se estar livre de perigo.


>>>
>>>
>>> --
>>> Esta mensagem foi verificada pelo sistema de antivírus e
>>> acredita-se estar livre de perigo.
>>
>>
>> --
>> Esta mensagem foi verificada pelo sistema de antivírus e
>> acredita-se estar livre de perigo.
>
>
> --
> Esta mensagem foi verificada pelo sistema de antivírus e
> acredita-se estar livre de perigo.

-- 
Esta mensagem foi verificada pelo sistema de antiv�rus e
 acredita-se estar livre de perigo.


=
Instru��es para entrar na lista, sair da lista e usar a lista em
http://www.mat.puc-rio.br/~obmlistas/obm-l.html
=


[obm-l] Re: [obm-l] Re: [obm-l] Re: [obm-l] Re: [obm-l] Demonstração com Geometria Plana?

2018-11-24 Por tôpico Carlos Victor
 

Oi Vanderlei, vamos lá: 

Seja ABCD o quadrado de diagonais AC e BD. Sejam os pontos P, E e F como
no enunciado. Tracemos a reta que passa por A e E encontrando o
prolongamento de DC em R.Seja também Q o ponto de interseção da reta que
passa por B e F com o prolongamento de DC.Seja T a interseção da reta
que passa por Q e E com o lado AB. 

Sejam BP=z, quadrado de lado AB=L, TB=k, CQ=x e QR=y. Por semelhança de
triângulos verifique que : 

x/k =L/z e y/L =x/z donde x^2=ky. Agora por semelhança veja que 

y/AT= x/k ou seja ky=x.AT e como ky=x^2 temos que x=AT ou seja CQ=AT. 

Como CQ é paralelo a AT e congruentes, temos que o quadrilátero ACQT é
um paralelogramo e já que as diagonais do quadrado são perpendiculares
temos que QT é perpendicular a BD. 

Temos então que no triângulo BDQ, BC e QH( H é a interseção de QT com
BD); 

ou seja E é o ortocentro de BDQ; donde PD é perpendicular a BQ. 

Verifiquem se há algum erro, ok? 

Abraços 

Carlos Victor 

Em 23/11/2018 22:38, Vanderlei Nemitz escreveu: 

> Estamos aguardando o Carlos Victor... 
> :) 
> 
> Em sex, 23 de nov de 2018 18:14, Mauricio de Araujo 
>  Alguem conseguiu finalizar a demonstração? 
> 
> Em qua, 21 de nov de 2018 11:52, Vanderlei Nemitz  escreveu: 
> Hummm... 
> Parece que prolongando BF e DC, que se encontram num ponto Q, E é o 
> ortocentro do triângulo BDQ. 
> O desenho sugere isso. 
> Mas como mostrar isso? 
> 
> Em ter, 20 de nov de 2018 23:38, Carlos Victor  escreveu: 
> 
> Oi Vanderlei, 
> 
> Uma dica : tente mostrar que o ponto E é o ortocentro de um triângulo " 
> estratégico". É muito legal que você descubra sozinho 
> 
> Abraços 
> 
> Carlos Victor 
> 
> Em 20/11/2018 17:33, Vanderlei Nemitz escreveu: 
> Pessoal, o seguinte problema sai "tranquilamente" usando Geometria Analítica. 
> Tentei usar Geometria Plana, mas apenas girei bastante, sem concluir. Será 
> que é possível? 
> 
> Dado um ponto P situado no prolongamento do lado AB de um quadrado ABCD, 
> traçam-se as retas PC e PD. Pelo ponto E, intersecção de BC e PD, conduzimos 
> a reta AE cuja intersecção com PC é o ponto F. Provar que BF e PD são 
> perpendiculares. 
> -- 
> Esta mensagem foi verificada pelo sistema de antivrus e 
> acredita-se estar livre de perigo.

-- 
Esta mensagem foi verificada pelo sistema de antivírus e 
acredita-se estar livre de perigo. 
-- 
Esta mensagem foi verificada pelo sistema de antivírus e 
acredita-se estar livre de perigo. 
-- 
Esta mensagem foi verificada pelo sistema de antivrus e 
acredita-se estar livre de perigo. 
 
-- 
Esta mensagem foi verificada pelo sistema de antiv�rus e
 acredita-se estar livre de perigo.



[obm-l] Re: [obm-l] Re: [obm-l] Re: [obm-l] Re: [obm-l] Dúvida conceitual (equações)

2018-10-15 Por tôpico Bernardo Freitas Paulo da Costa
On Mon, Oct 15, 2018 at 8:07 AM Claudio Buffara
 wrote:
>
> Derivando e igualando a zero o lado esquerdo da sua equação, ficamos com:
> -2*cos(x)*sen(x) + sen(x) = 0 ==>
> sen(x) = 0  ou  cos(x) = 1/2 ==>
> x = 0 ou x = pi ou x = 2pi
> ou x = pi/3 ou x = 5pi/3.
>
> Assim, uma definição que me parece adequado para equações em geral (e não 
> necessariamente polinomiais) da forma f(x) = 0 é que uma raiz de 
> multiplicidade n é raiz de f, f’, ... , f^(n-1) mas não é raiz de f^(n).
>
> Naturalmente, se f não tiver todas as derivadas, precisaremos achar uma 
> definição diferente. Mas talvez, neste caso, nem faça sentido falar em 
> multiplicidade de uma raiz.

Essa definição funciona relativamente bem se f é analítica, porque o
comportamento local é determinado por inteiros.  Se f for apenas
diferenciável, talvez seja complicado dizer algo, como o exemplo
clássico de exp(-1/x^2).  A raiz tem multiplicidade infinita?

Enfim, existem, como você falou, boas razões para incorporar
multiplicidade (por exemplo estabilidade numérica), mas isso em geral
só faz sentido no mundo analítico, onde a noção de "grau" é dada pelas
derivadas.  Acho que mesmo no mundo C-infinito já pode haver
problemas, mas não sou especialista (nessas :D) patologias.  A questão
original, incluindo multiplicidades, pode ser resolvida simplesmente
usando as relações de Girard, que dependem de forma simples da
equação.

Vou tentar dar um exemplo que ilustra meu ponto de vista:  qual o
produto das raízes da equação x^2 - 4x + c?  "Qualquer um" dirá "c".
Mas, naturalmente, se c = 4, a única solução é x=2, e portanto (sem
usar multiplicidades) este produto seria apenas 2.  E daí a fórmula
fica muito mais complicada, com um caso especial, e descontínua.  A
grande sacada do Girard foi, justamente, propor incorporar as
multiplicidades, para simplificar as fórmulas (além, é claro, de
incluir também as soluções negativas, antes consideradas como
"absurdas" - este foi, provavelmente, o maior motivo de as pessoas
considerarem raízes negativas como algo que fazia sentido, e portanto
os números negativos também).  Mas isso não quer dizer que a equação
x^2 - 4x + 4 tenha duas soluções.  É apenas uma forma mais conveniente
de interpretar as raízes quando se pensam nas relações de Girard (e
várias outras fórmulas).  Neste sentido, acho que este tipo de questão
mais atrapalha (porque "era só para usar a fórmula") - a menos que,
justamente, se discuta *porque* falamos de multiplicidade: para que as
fórmulas fiquem mais simples (e você pode incluir "bonitas" também,
por minha conta).  Nada mais.  E esta "simplificação" do entendimento
através da simplificação das fórmulas não se justifica sempre: este
mesmo debate sobre multiplicidades leva a considerar objetos no
infinito (para que todas as retas se intersectem sempre em um ponto),
complexos (para x^2 + 1 = 0 ter raiz), etc.  Muitas vezes, é útil ter
esse entendimento unificado, onde tudo "só depende do grau".  Mas será
mesmo que se eu perguntar para você "em quantos pontos a reta x=3
corta a parábola y=x^2?" você vai dizer "2, é óbvio"?

Abraços,
-- 
Bernardo Freitas Paulo da Costa

-- 
Esta mensagem foi verificada pelo sistema de antiv�rus e
 acredita-se estar livre de perigo.


=
Instru��es para entrar na lista, sair da lista e usar a lista em
http://www.mat.puc-rio.br/~obmlistas/obm-l.html
=


Re: [obm-l] Re: [obm-l] Re: [obm-l] Re: [obm-l] Dúvida conceitual (equações)

2018-10-15 Por tôpico Claudio Buffara
Derivando e igualando a zero o lado esquerdo da sua equação, ficamos com:
-2*cos(x)*sen(x) + sen(x) = 0 ==>
sen(x) = 0  ou  cos(x) = 1/2 ==>
x = 0 ou x = pi ou x = 2pi
ou x = pi/3 ou x = 5pi/3.

Assim, uma definição que me parece adequado para equações em geral (e não 
necessariamente polinomiais) da forma f(x) = 0 é que uma raiz de multiplicidade 
n é raiz de f, f’, ... , f^(n-1) mas não é raiz de f^(n).

Naturalmente, se f não tiver todas as derivadas, precisaremos achar uma 
definição diferente. Mas talvez, neste caso, nem faça sentido falar em 
multiplicidade de uma raiz.

Enviado do meu iPhone

Em 15 de out de 2018, à(s) 08:13, Vanderlei Nemitz  
escreveu:

> Claudio:
> Eu ficaria com a mesma dúvida!
> Pensaria em apenas uma raiz.
> 
> Qual é a soma das raízes da equação (cos x)^2 - cos x + 1/4 = 0 no 
> intervalo [0, 2pi]?
> 
> Em seg, 15 de out de 2018 07:00, Claudio Buffara  
> escreveu:
>> Qual a soma das raizes de (2^x - 8)^3 = 0?
>> Se a equação acima fosse apresentada como:
>> 2^(3x) - 24*2^(2x) + 192*2^x - 512 = 0,
>> isso mudaria sua resposta?
>> 
>> Enviado do meu iPhone
>> 
>> Em 15 de out de 2018, Ã (s) 00:29, Vanderlei Nemitz  
>> escreveu:
>> 
>>> Valeu, Pedro! Tomara que mais alguém emita sua opinião.
>>> Um abraço!
>>> 
>>> Em dom, 14 de out de 2018 18:59, Pedro José  
>>> escreveu:
 Boa noite!
 Bom questionamento. Vou me posicionar na arquibancada. 
 Minha posição é controversa. Se quer se levar em conta a 
 repetição tem que se falar do produto das raízes, cada elevada a 
 sua multiplicidade. No caso de soma, cada raiz multiplicada pela 
 multiplicidade.
 Para esse exemplo, o conjunto solução é {1/2,-1} então o 
 produto é -1/2.
 Em suma, não aceito n raízes iguais, mas sim uma raiz de 
 multiplicidade n.
 Se quando queremos provar que algo é unico supomos a existência de 
 dois e provamos que são iguais. Creio que seja contraditório dois ou 
 nais iguais.
 Mas vamos observar as diversas posições, pois, creio que o assunto 
 não seja pacífico. 
 Saudações, 
 PJMS 
 
 Em Dom, 14 de out de 2018 06:33, Vanderlei Nemitz  
 escreveu:
> Bom dia!
> Na seguinte questão, que me foi apresentada por um aluno, a resposta 
> proposta é a alternativa C (1/2). Eu sempre pensei que apenas 
> considerávamos multiplicidades em equações polinomiais. Como 
> essa é uma equação exponencial, obtive a resposta B (-1/2). O 
> que é correto pensar?
> 
> O produto das raízes da equação 16.4^3x - 40.4^2x + 17.4^x - 2 = 
> 0 é igual a:
> A) 1
> B) - 0,5
> C) 0,5
> D) - 1
> E) 0
> 
> Muito obrigado!
> 
> -- 
> Esta mensagem foi verificada pelo sistema de antivírus e 
> acredita-se estar livre de perigo.
 
 -- 
 Esta mensagem foi verificada pelo sistema de antivírus e 
 acredita-se estar livre de perigo.
>>> 
>>> -- 
>>> Esta mensagem foi verificada pelo sistema de antivírus e 
>>> acredita-se estar livre de perigo.
>> 
>> -- 
>> Esta mensagem foi verificada pelo sistema de antivírus e 
>> acredita-se estar livre de perigo.
> 
> -- 
> Esta mensagem foi verificada pelo sistema de antivírus e 
> acredita-se estar livre de perigo.

-- 
Esta mensagem foi verificada pelo sistema de antiv�rus e
 acredita-se estar livre de perigo.



Re: [obm-l] Re: [obm-l] Re: [obm-l] Re: [obm-l] Dúvida conceitual (equações)

2018-10-15 Por tôpico Claudio Buffara
Pensando só como uma equação, talvez faça sentido não considerar a 
multiplicidade.

Mas, no seu exemplo, no intervalo [0,2pi], os gráficos de 
f(x) = cos(x) - 1/2 
e de
g(x) = (cos(x) - 1/2)^2
tem um comportamento bem distinto um do outro em vizinhanças de pi/3 e 5pi/3.
Por exemplo, o gráfico de f corta o eixo x em pi/3 enquanto que o de g apenas 
tangencia o eixo neste ponto.
Idem pros outros exemplos.
Isso sugere que, mesmo nestes casos, talvez seja conveniente considerar a 
multiplicidade de uma raiz.

Enviado do meu iPhone

Em 15 de out de 2018, à(s) 08:13, Vanderlei Nemitz  
escreveu:

> Claudio:
> Eu ficaria com a mesma dúvida!
> Pensaria em apenas uma raiz.
> 
> Qual é a soma das raízes da equação (cos x)^2 - cos x + 1/4 = 0 no 
> intervalo [0, 2pi]?
> 
> Em seg, 15 de out de 2018 07:00, Claudio Buffara  
> escreveu:
>> Qual a soma das raizes de (2^x - 8)^3 = 0?
>> Se a equação acima fosse apresentada como:
>> 2^(3x) - 24*2^(2x) + 192*2^x - 512 = 0,
>> isso mudaria sua resposta?
>> 
>> Enviado do meu iPhone
>> 
>> Em 15 de out de 2018, Ã (s) 00:29, Vanderlei Nemitz  
>> escreveu:
>> 
>>> Valeu, Pedro! Tomara que mais alguém emita sua opinião.
>>> Um abraço!
>>> 
>>> Em dom, 14 de out de 2018 18:59, Pedro José  
>>> escreveu:
 Boa noite!
 Bom questionamento. Vou me posicionar na arquibancada. 
 Minha posição é controversa. Se quer se levar em conta a 
 repetição tem que se falar do produto das raízes, cada elevada a 
 sua multiplicidade. No caso de soma, cada raiz multiplicada pela 
 multiplicidade.
 Para esse exemplo, o conjunto solução é {1/2,-1} então o 
 produto é -1/2.
 Em suma, não aceito n raízes iguais, mas sim uma raiz de 
 multiplicidade n.
 Se quando queremos provar que algo é unico supomos a existência de 
 dois e provamos que são iguais. Creio que seja contraditório dois ou 
 nais iguais.
 Mas vamos observar as diversas posições, pois, creio que o assunto 
 não seja pacífico. 
 Saudações, 
 PJMS 
 
 Em Dom, 14 de out de 2018 06:33, Vanderlei Nemitz  
 escreveu:
> Bom dia!
> Na seguinte questão, que me foi apresentada por um aluno, a resposta 
> proposta é a alternativa C (1/2). Eu sempre pensei que apenas 
> considerávamos multiplicidades em equações polinomiais. Como 
> essa é uma equação exponencial, obtive a resposta B (-1/2). O 
> que é correto pensar?
> 
> O produto das raízes da equação 16.4^3x - 40.4^2x + 17.4^x - 2 = 
> 0 é igual a:
> A) 1
> B) - 0,5
> C) 0,5
> D) - 1
> E) 0
> 
> Muito obrigado!
> 
> -- 
> Esta mensagem foi verificada pelo sistema de antivírus e 
> acredita-se estar livre de perigo.
 
 -- 
 Esta mensagem foi verificada pelo sistema de antivírus e 
 acredita-se estar livre de perigo.
>>> 
>>> -- 
>>> Esta mensagem foi verificada pelo sistema de antivírus e 
>>> acredita-se estar livre de perigo.
>> 
>> -- 
>> Esta mensagem foi verificada pelo sistema de antivírus e 
>> acredita-se estar livre de perigo.
> 
> -- 
> Esta mensagem foi verificada pelo sistema de antivírus e 
> acredita-se estar livre de perigo.

-- 
Esta mensagem foi verificada pelo sistema de antiv�rus e
 acredita-se estar livre de perigo.



  1   2   3   4   5   6   7   8   >